TEST BANK for Linne And Ringsruds Clinical Laboratory Science 7th Edition by Mary Louise Turgeon

Page 1

TEST BANK for Linne And Ringsruds Clinical Laboratory Science 7th Edition by Mary Louise Turgeon


Linne and Ringsrud’s Clinical Laboratory Science 7th Edition Turgeon Test Bank Chapter 01: Fundamentals of the Clinical Laboratory Turgeon: Linné & Ringsrud’s Clinical Laboratory Science, 7th Edition MULTIPLE CHOICE 1. The ASCLS professional Code of Ethics states that all laboratory professionals have a

responsibility for proper conduct toward: a. the patient. b. colleagues and the profession. c. society. d. all of the above. ANS: D

The Code of Ethics sets forth the principles and standards by which clinical laboratory professionals practice their profession. It includes duty to the patient, duty to colleagues and the profession, and duty to society. PTS: 1

DIF:

Cognitive Level: 1

REF: p. 12

2. The role of the CEO of a health care organization is to: a. implement policy and daily activities. b. set policy and guide the organization. c. manage financial affairs of the organization. d. manage the hospital information system. ANS: B

The CEO, chief executive T ofEfiS ceTr,BaA ndNK thS eE boLaL rdEoRf.trC usOteMes set policy and guide the organization. The chief operating officer, COO, is responsible for implementing policies and daily activities. The chief financial officer, CFO, manages organizational financial affairs and the chief technology officer manages the hospital information system. PTS: 1

DIF:

Cognitive Level: 1

REF: p. 7

3. Which certifying agency or agencies have been granted deemed status to act on behalf of the

federal government in matters of laboratory accreditation? a. Commission on Office Laboratory Accreditation (COLA) b. College of American Pathologists (CAP) c. The Joint Commission (TJC) d. Both b and c ANS: D

Two certifying agencies, the College of American Pathologists (CAP) and The Joint Commission (TJC), have been given deemed status to act on the federal government’s behalf. From an external source, guidelines and standards have also been set to govern safe work practices in the clinical laboratory. PTS: 1

DIF:

Cognitive Level: 2

REF: p. 9

4. The definition of a waived test is laboratory tests or procedures: a. cleared by the FDA for home use.


b. that can be performed by a preferred provider. c. that have a small reasonable risk of harm to the patient if performed incorrectly. d. that are moderately complex. ANS: A

As currently defined, waived laboratory tests or procedures are those cleared by the Food and Drug Administration (FDA) for home use that employ methodologies so simple that the likelihood of erroneous results is negligible and that pose no reasonable risk of harm to the patient if performed incorrectly. To meet the criteria for being in the provider-performed microscopy (PPM) category, procedures must be personally performed by the practitioner (defined as a physician, a midlevel practitioner under the supervision of a physician, or a dentist). Tests that have moderate- and high-complexity levels are more regulated, with some minimal personnel standards required, as well as proficiency testing and quality assurance programs. PTS: 1

DIF:

Cognitive Level: 1

REF: p. 5

5. CLIA personnel requirements specify: a. requirements of persons working in each of the testing sites where tests of

moderate or high complexity are done. b. educational requirements. c. required training and experience. d. all of the above. ANS: D

The personnel section of the CLIA regulations defines the responsibilities of persons working in each of the testing sites at which tests of moderate or high complexity are performed, along ES with the educational requirTem enTtsBaAnN dK traSiE niL ngLaEnR d .exCpOeM rience needed. Minimum education and experience needed by testing personnel to perform the specific laboratory tests on human specimens are also regulated by CLIA ’88. These job requirements are listed in the CLIA ’88 final regulations, along with their amendments published from 1992 to 1995. There are no CLIA regulations for testing personnel who work at sites performing only the waived tests or the PPM. For laboratories in which only tests of moderate complexity are performed, the minimum requirement for testing personnel is a high school diploma or equivalent, as long as there is documented evidence of an amount of training sufficient to ensure that the laboratorian has the skills necessary to collect, identify, and process the specimen, and to perform the laboratory analysis itself. For tests of the highly complex category, the personnel requirements are more stringent. Anyone who is eligible to perform highly complex tests can also perform moderate-complexity testing. PTS: 1

DIF:

Cognitive Level: 2

REF: pp. 4-5

6. The fastest growing segment of laboratory testing is: a. dry reagent chemistry. b. lab on a chip technology. c. molecular diagnostics. d. automation of blood cell counting. ANS: C

In the 21st century, molecular diagnosis is the hottest topic in the clinical laboratory.


PTS: 1

DIF:

Cognitive Level: 1

REF: p. 7

7. Informed consent upon patient’s hospital admission can encompass: a. agreement to the nature of the testing to be done. b. agreement to what will be done with laboratory test results. c. implied consent to many routine procedures that may be performed. d. all of the above. ANS: D

For laboratories, an important responsibility is obtaining informed consent from the patient. Informed consent means that the patient is aware of, understands, and agrees to the nature of the testing to be done and what will be done with the results reported. Generally, when a patient enters a hospital, there is an implied consent to the many routine procedures that will be performed while the patient is in the hospital. Venipuncture is one of the routine tests that carry this implied consent. PTS: 1

DIF:

Cognitive Level: 2

REF: p. 10

8. The federal legislation requiring strict confidentiality of patient results is: a. Clinical Laboratory Improvement Act (CLIA ’88). b. Health Insurance Portability and Accountability Act (HIPAA). c. National Certifying Agency. d. combined state laws. ANS: B

The Health Insurance Portability and Accountability Act (HIPAA) enacted in 1996 requires that any results obtained for specimens from patients must be kept strictly confidential. Any information about the patieTnE t, S inTcB luA diN ngKS thE eL tyL peEsRo. fC mO eaMsurements being done, must also be kept in confidence. Clinical Laboratory Improvement Act (CLIA ’88) regulates laboratory testing. The National Certifying Agency grants professional certification to individuals. HIPAA legislation supersedes any and all state laws regarding confidentiality. PTS: 1

DIF:

Cognitive Level: 1

REF: pp. 10-11

9. The revised CLIA ’88 and HIPAA regulations give patients: a. the right to access their own laboratory reports. b. the right to give a personal representative the patient’s completed test reports. c. the right to refuse laboratory testing. d. both a and b. ANS: D

The new rule revises CLIA ’88 and HIPAA privacy rules to require laboratories to give a patient, or a person designated by the patient or his or her “personal representative,” access to the patient’s completed test reports on the patient’s or the representative’s request. Generally, the rule requires that laboratories provide individuals with access to their laboratory test reports within 30 days of the request. PTS: 1

DIF:

Cognitive Level: 2

REF: p. 11

10. Chain of custody is defined as: a. physical transport of a specimen needed for legal reasons.


b. escorting patients who are prison inmates. c. beginning with the moment the specimen is collected and transported to the

laboratory, to the analysis itself and the reporting of the results, each step must be documented. d. documentation of quality control when reporting a patient’s laboratory results. ANS: C

Laboratory test results that could potentially be used in a court of law—at a trial or judicial hearing—must be handled in a specific manner. For evidence to be admissible, each step of the analysis, beginning with the moment the specimen is collected and transported to the laboratory, to the analysis itself and the reporting of the results, must be documented—a process known as maintaining the chain of custody. Specimens that provide alcohol levels, specimens collected from rape victims, specimens for paternity testing, and specimens submitted from the medical examiner’s cases are the usual types requiring “chain-of-custody” documentation. PTS: 1

DIF:

Cognitive Level: 1

REF: p. 11

11. Situational ethics is a system of judging acts: a. by strict principles. b. by categorical principles. c. within a context. d. by legal principles. ANS: C

Situational ethics is a system of ethics by which acts are judged within their contexts instead of by categorical principles. Hospitals have ethics committees to evaluate situational ethics cases. Individual laboratorT yE prSoT feB ssA ioNnK alS sE mL ayLnEeR ed.tC oOmMake decisions based on personal or professional values. PTS: 1

DIF:

Cognitive Level: 1

REF: p. 12

MATCHING

Match the following sections of the clinical laboratory with a representative test or assay for that section. Use an answer only once. a. hematology b. clinical chemistry c. immunohematology d. immunology and serology e. microbiology 1. 2. 3. 4. 5.

measures blood glucose and cholesterol levels counts blood cells determines blood groups screens for strep throat infections screens for viral diseases, for example, HIV

1. ANS: B REF: p. 6

PTS: 1

DIF:

Cognitive Level: 1


2. ANS: A REF: p. 6 3. ANS: C REF: p. 6 4. ANS: E REF: p. 6 5. ANS: D REF: p. 6

PTS: 1

DIF:

Cognitive Level: 1

PTS: 1

DIF:

Cognitive Level: 1

PTS: 1

DIF:

Cognitive Level: 1

PTS: 1

DIF:

Cognitive Level: 1

Match the following acronyms with its full name. Use an answer only once. a. American Society for Clinical Laboratory Science b. National Accrediting Agency for Clinical Laboratory Sciences c. American Society of Clinical Pathologists 6. ASCP 7. ASCLS 8. NAACLS 6. ANS: C REF: p. 2 7. ANS: A REF: p. 2 8. ANS: B REF: p. 2

PTS: 1

DIF:

Cognitive Level: 1

PTS: 1

DIF:

Cognitive Level: 1

PTS: 1

DIF:

Cognitive Level: 1


Chapter 02: Safety in the Clinical Laboratory Turgeon: Linné & Ringsrud’s Clinical Laboratory Science, 7th Edition MULTIPLE CHOICE 1. The newest Patient Safety Goals with application to the laboratory includes: a. improve accuracy of patient identification. b. identify analytical errors by using quality control before releasing results. c. decrease the laboratory turn-around time. d. establish an exact protocol for testing outpatient specimens. ANS: A

TJC National Patient Safety Goals: Goals Related to the Laboratory (Effective January 1, 2014) Goal 1 Improve the accuracy of patient identification. Goal 2 Improve the effectiveness of communication among caregivers. Goal 7 Reduce the risk of health care–associated infections. Reference: The Joint Commission (TJC): National Patient Safety Goals effective January 1, 2013. Goals related to a laboratory accreditation program. www.jointcommission.org/Standards. Retrieved February 1, 2014. PTS: 1

DIF:

Cognitive Level: 2

REF: p. 19

2. Which of the following legislative documents ensures that workers have safe and healthful

working conditions? a. Occupational Safety and Health Act TESTBANKSELLER.COM b. Hazard Communication Standard c. Clinical Laboratory Improvement Act (CLIA) d. Both a and b ANS: D

Both the Occupational Safety and Health Act and the Hazard Communication Standard are legislation that ensures that workers have safe and healthful working conditions. The U.S. government created a system of safeguards and regulations under the Occupational Safety and Health Act of 1970 and in 1988 expanded the Hazard Communication Standard to apply to hospital staff. PTS: 1

DIF:

Cognitive Level: 1

3. Safety in the clinical laboratory includes: a. a formal safety program. b. chemical hygiene plan. c. bloodborne pathogen plan. d. all of the above. ANS: D

REF: p. 22


The Occupational and Safety Health Agency (OSHA) and the Centers for Disease Control and Prevention (CDC) have published numerous safety standards that are applicable to clinical laboratories. Safety in the clinical laboratory includes a formal safety program, specifically mandated plans (e.g., chemical hygiene and bloodborne pathogens), and identification of various hazards (e.g., fire, electrical, chemical, and biological hazards). PTS: 1

DIF:

Cognitive Level: 1

REF: p. 22

4. Information related to hazards associated with chemicals in the workplace must be provided

in: a. b. c. d.

Laboratory Policy Manual. material safety data sheets (MSDSs). Bloodborne Pathogen Standards information. Standard Operating Procedures Manual.

ANS: B

Hazards associated with chemicals are provided in MSDSs, which describe hazards, safe handling, storage, and disposal of hazardous chemicals. The information is provided by chemical manufacturers and suppliers about each chemical and accompanies the shipment of each chemical. PTS: 1

DIF:

Cognitive Level: 1

REF: p. 23

5. Perhaps the simplest and most important step in the proper handling of any hazardous

substance is proper: a. disposal of a chemical. b. labeling of a substance. c. storage of a substance. d. wearing of heavy rubber gloves. ANS: B

Labeling may be the simplest and the single most important step in the proper handling of any hazardous substance. A label for a container should include a date and the contents of the container. When the contents of one container are transferred to another container, this information should also be transferred to the new container. OSHA recommends that all chemically hazardous material be properly labeled with the hazardous contents and severity of the material, as well as bear a hazard symbol. The labels of chemicals in the original containers must not be removed or altered. PTS: 1

DIF:

Cognitive Level: 2

REF: p. 26

6. Standard Precautions is: a. a term that has replaced the term Universal Precautions. b. the process of treating all blood and body fluids as potentially infectious. c. a safety precaution concerned with the handling of all patient specimens. d. all of the above. ANS: D

The Centers for Disease Control and Prevention (CDC) recommends safety precautions concerning the handling of all patient specimens. These are known as Standard Precautions (formerly known as “universal precautions,” or “universal blood and body fluid precautions”).


PTS: 1

DIF:

Cognitive Level: 2

REF: p. 28

7. The definition of a biohazard is: a. chemicals that can burn the skin severely. b. flammable substances. c. potentially infectious materials or agents. d. explosive liquids. ANS: C

Biohazard is a word that is posted throughout the laboratory to denote infectious materials or agents that present a risk or even a potential risk to the health of humans or animals in the laboratory. The potential risk can be either through direct infection or through the environment. Infection can occur during specimen collection, or from handling, transporting, or testing the specimen. PTS: 1

DIF:

Cognitive Level: 1

REF: p. 28

8. Infectious disease safety practices include: a. educating and training all health workers in standard precautions. b. providing disposable gloves. c. monitoring compliance with protective biosafety policies. d. all of the above. ANS: D

The purpose of the Bloodborne Pathogens Standard and the Occupational Exposure Standard is to provide a safe work environment. OSHA mandates that an employer do the following: 1. Educate and train all heTaE lthScTarBeAwNoK rkSerEsLinLsEtaRn. daCrdOM precautions and in prevention of bloodborne infections. 2. Provide proper equipment and supplies (e.g., gloves). 3. Monitor compliance with the protective biosafety policies. PTS: 1

DIF:

Cognitive Level: 2

REF: p. 30

9. An important change in the newly revised OSHA Hazard Communication Standard (HCS) is: a. labels on chemicals must be in different colors. b. new safety data sheets can be abbreviated documents. c. a single set of harmonized criteria for classifying chemicals is used. d. the chemical mixtures must be stated on the container label. ANS: C

The OSHA Hazard Communication Standard (HCS) (29 CFR 1910.1200(g)) requires that the chemical manufacturer, distributor, or importer provide safety data sheets (SDSs), formerly material safety data sheets (MSDSs), for each hazardous chemical to downstream users to communicate information on these hazards. As with the current standard, the new HCS requires chemical manufacturers and importers to evaluate the chemicals they produce or import and provide hazard information to employers and workers by putting labels on containers and preparing SDSs. The modified standard provides a single set of harmonized criteria for classifying chemicals according to their health and physical hazards and specifies hazard communication elements for labeling and SDSs. Employers must ensure that SDSs are readily accessible to employees.


The major changes to the HCS include the following: 1. Hazard classification: Chemical manufacturers and importers are required to determine the hazards of the chemicals they produce or import. Hazard classification under the new, updated standard provides specific criteria to address health and physical hazards as well as classification of chemical mixtures. 2. Labels: Chemical manufacturers and importers must provide a label that includes a signal word, pictogram, hazard statement, and precautionary statement for each hazard class and category. 3. Safety data sheets: The new SDS format requires 16 specific sections, ensuring consistency in presentation of important protection information. 4. Information and training: To facilitate understanding of the new system, the new standard requires that workers be trained by December 1, 2013, and beyond for specified requirements. The Hazard Communication Standard in 1983 gave the workers the “right to know,” but the new Globally Harmonized System gives workers the “right to understand.” PTS: 1

DIF:

Cognitive Level: 2

REF: p. 26

10. Preemployment health profiles of students and laboratory staff should include immune status

evaluation for at a minimum. a. hepatitis B b. rubella c. measles d. all of the above ANS: D

Preemployment health proT filEesSoTfBsA tuN deKnS tsEaL ndLlEabRo.raCtoOrM y staff should include immune status evaluation for hepatitis B, rubella, and measles at a minimum. PTS: 1

DIF:

Cognitive Level: 1

REF: p. 37

11. A carcinogen: a. is any substance that can cause the development of cancerous growth in living

tissue. b. is related to the spread of viral diseases. c. cannot be used in the laboratory. d. is both a and b. ANS: A

Carcinogens are any substances that cause the development of cancerous growths in living tissue; these substances are considered hazardous to people working with them in laboratories. When possible, substances that are potentially carcinogenic have been replaced by ones that are less hazardous. If necessary, with the proper safeguards in place, potentially carcinogenic substances can be used in the laboratory. Lists of potential carcinogens used in a particular laboratory must be available to all who work there. PTS: 1

DIF:

Cognitive Level: 1

12. A Class A fire is: a. ordinary combustibles. b. flammable liquids and gases.

REF: p. 41


c. electrical equipment. d. powdered metal. ANS: A

Fires are classified into five different basic types: Class A—Ordinary combustibles Class B—Flammable liquids and gases Class C—Electrical equipment Class D—Powdered metal (combustible) material Class E—Cannot be extinguished PTS: 1

DIF:

Cognitive Level: 1

REF: p. 42

13. Infectious waste includes: a. blood and blood products. b. contaminated sharps. c. microbiological waste. d. all of the above. ANS: D

OSHA has defined infectious waste as blood and blood products, contaminated sharps, pathological wastes, and microbiological wastes. PTS: 1

DIF:

Cognitive Level: 1

REF: pp. 42-43

14. Treatment for alkali or acid burns of the skin is to: a. rinse thoroughly with large amounts of running tap water. b. apply ice. c. wash thoroughly with soap and water. d. apply direct pressure. ANS: A

For the treatment of alkali or acid burns on the skin or in the mouth: Rinse thoroughly with large amounts of running tap water. If the burns are serious, consult a physician. PTS: 1

DIF:

Cognitive Level: 1

REF: p. 44

TRUE/FALSE 1. Food and drinks should not be stored or consumed in work areas. ANS: T

All laboratories need programs to minimize risks to the health and safety of employees, volunteers, and patients. Suitable physical arrangements, an acceptable work environment, and appropriate equipment need to be available to maintain safe operations. In addition to Standard Precautions, a variety of other safety practices should be adhered to in order to reduce the risk of inadvertent contamination with blood or certain body fluids. These practices include the following: 1. All devices in contact with blood that are capable of transmitting infection to the donor or recipient must be sterile and nonreusable.


2. Food and drinks should not be consumed in work areas or stored in the same area as specimens. Containers, refrigerators, or freezers used for specimens should be marked as containing a biohazard. 3. Specimens needing centrifugation should be capped and placed into a centrifuge with a sealed dome. 4. Slowly place a gauze square over and carefully open rubber-stoppered test tubes with the gauze over the stopper to minimize aerosol production (the introduction of substances into the air). 5. Use autodilutors or safety bulbs for pipetting. Pipetting by mouth of any clinical material must be strictly forbidden. PTS: 1

DIF:

Cognitive Level: 2

REF: p. 35

2. Rubber-stoppered test tubes should be opened slowly with a gauze square over the stopper. ANS: T

All laboratories need programs to minimize risks to the health and safety of employees, volunteers, and patients. Suitable physical arrangements, an acceptable work environment, and appropriate equipment need to be available to maintain safe operations. In addition to Standard Precautions, a variety of other safety practices should be adhered to in order to reduce the risk of inadvertent contamination with blood or certain body fluids. These practices include the following: 1. All devices in contact with blood that are capable of transmitting infection to the donor or recipient must be sterile and nonreusable. 2. Food and drinks should not be consumed in work areas or stored in the same area as specimens. Containers, refrigerators, or freezers used for specimens should be marked as containing a biohazardT . ESTBANKSELLER.COM 3. Specimens needing centrifugation should be capped and placed into a centrifuge with a sealed dome. 4. Slowly place a gauze square over and carefully open rubber-stoppered test tubes with the gauze over the stopper to minimize aerosol production (the introduction of substances into the air). 5. Use autodilutors or safety bulbs for pipetting. Pipetting by mouth of any clinical material must be strictly forbidden. PTS: 1

DIF:

Cognitive Level: 2

REF: p. 35

3. Contact lenses can be manipulated with gloved hands. ANS: F

All laboratories need programs to minimize risks to the health and safety of employees, volunteers, and patients. Suitable physical arrangements, an acceptable work environment, and appropriate equipment need to be available to maintain safe operations. In addition to Standard Precautions, a variety of other safety practices should be adhered to in order to reduce the risk of inadvertent contamination with blood or certain body fluids. These practices include the following: 1. All devices in contact with blood that are capable of transmitting infection to the donor or recipient must be sterile and nonreusable.


2. Food and drinks should not be consumed in work areas or stored in the same area as specimens. Containers, refrigerators, or freezers used for specimens should be marked as containing a biohazard. 3. Specimens needing centrifugation should be capped and placed into a centrifuge with a sealed dome. 4. Slowly place a gauze square over and carefully open rubber-stoppered test tubes with the gauze over the stopper to minimize aerosol production (the introduction of substances into the air). 5. Use autodilutors or safety bulbs for pipetting. Pipetting by mouth of any clinical material must be strictly forbidden. PTS: 1

DIF:

Cognitive Level: 2

REF: p. 35

4. Lipstick or makeup should not be applied in the clinical work area. ANS: T

All laboratories need programs to minimize risks to the health and safety of employees, volunteers, and patients. Suitable physical arrangements, an acceptable work environment, and appropriate equipment need to be available to maintain safe operations. In addition to Standard Precautions, a variety of other safety practices should be adhered to in order to reduce the risk of inadvertent contamination with blood or certain body fluids. These practices include the following: 1. All devices in contact with blood that are capable of transmitting infection to the donor or recipient must be sterile and nonreusable. 2. Food and drinks should not be consumed in work areas or stored in the same area as specimens. Containers, refrigerators, or freezers used for specimens should be marked as containing a biohazardT . ESTBANKSELLER.COM 3. Specimens needing centrifugation should be capped and placed into a centrifuge with a sealed dome. 4. Slowly place a gauze square over and carefully open rubber-stoppered test tubes with the gauze over the stopper to minimize aerosol production (the introduction of substances into the air). 5. Use autodilutors or safety bulbs for pipetting. Pipetting by mouth of any clinical material must be strictly forbidden. PTS: 1

DIF:

Cognitive Level: 2

REF: p. 35


Chapter 03: Quality Assessment and Quality Control in the Clinical Laboratory Turgeon: Linné & Ringsrud’s Clinical Laboratory Science, 7th Edition MULTIPLE CHOICE 1. Required regulation of clinical laboratories is achieved by the: a. Clinical Laboratory Improvement Amendments of 1988. b. The Joint Commission (TJC). c. College of American Pathologists (CAP). d. American Hospital Association (AHA). ANS: A

In 1988, the U.S. Congress enacted the Clinical Laboratory Improvement Amendments of 1988 (CLIA ‘88), which established a minimum government-regulated threshold for all aspects of clinical laboratory testing. Other accrediting organizations include The Joint Commission (TJC), the Commission on Office Laboratory Accreditation (COLA), and the College of American Pathologists (CAP). PTS: 1

DIF:

Cognitive Level: 2

REF: p. 54

2. ISO 15189 is based on: a. ISO/IEC 17025. b. ISO 9001. c. CAP 15189. d. both a and b. ANS: D

The standard, ISO 15189, is based on ISO/IEC 17025, the main standard used by testing and calibration laboratories and ISO 9001. PTS: 1

DIF:

Cognitive Level: 1

3. ISO 15189 monitors requirements such as: a. personnel. b. documents. c. laboratory equipment. d. all of the above. ANS: D

Box 3-1 ISO 15189 Overview MANAGEMENT REQUIREMENTS: Organization and management Quality management system Document control Review of contracts Examination by referral laboratories External services and supplies Advisory services Resolution of complaints Identification and control of nonconformities

REF: p. 56


Corrective action Preventive action Continual improvement Quality and technical records Internal audits Management review TECHNICAL REQUIREMENTS: Personnel Accommodation and environmental conditions Laboratory equipment Preexamination procedures Examination procedures Assuring quality of examination procedures Postexamination procedures Reporting of results From Malone B. ISO accreditation comes to America, Clin Lab News 35:1-4, 2009. PTS: 1

DIF:

Cognitive Level: 1

REF: p. 56

4. Lean principles of management focus on: a. reduction of unnecessary activities. b. describe personnel qualifications. c. measure reduction in variability. d. stress flow of ideas from top to bottom. ANS: A

Lean focuses on reducing waste, and Six Sigma, a metric and methodology that focuses on reducing variability. Lean principles of reduction of unnecessary and non-value-added activities to reduce total production time and effort can be appropriately applied in all sections of the laboratory PTS: 1

DIF:

Cognitive Level: 2

REF: pp. 56-57

5. Nonanalytical factors in quality assessment include: a. qualified personnel. b. established laboratory policies. c. quality control charts. d. both a and b. ANS: D

The competence of personnel is an important determinant of the quality of the laboratory result. Only properly certified personnel can perform nonwaived assays. CLIA ’88 requirements for laboratory personnel in regard to levels of education and experience or training must be followed for laboratories doing moderately complex or highly complex testing. Established laboratory policies are a nonanalytical factor in quality assessment. Laboratory policies should be included in a laboratory reference manual that is available to all hospital personnel. Each laboratory must have an up-to-date safety manual. PTS: 1

DIF:

Cognitive Level: 1

REF: pp. 58-59


6. An example of a preanalytical error is: a. specimen obtained from a wrong patient. b. lack of frequent preventive maintenance. c. reporting of laboratory results by phone. d. poorly trained phlebotomists. ANS: A

Examples of preanalytical errors include a specimen obtained from a wrong patient, specimen procured at the wrong time, specimen collected in the wrong tube or container, blood specimens collected in the wrong order, incorrect labeling of specimen, or improper processing of a specimen. PTS: 1

DIF:

Cognitive Level: 2

REF: p. 58

7. Quality assessment programs include: a. patient identification. b. specimen procurement. c. specimen transportation and processing procedures. d. all of the above. ANS: D

Quality assessment programs monitor test request procedures; patient identification, specimen procurement, and labeling; specimen transportation and processing procedures; laboratory personnel performance; laboratory instrumentation, reagents, and analytical test procedures; turnaround times; and the accuracy of the final result. Complete documentation of all procedures involved in obtaining the analytical result for the patient sample must be NaKtiS maintained and monitored TinEaSsTyB steAm cE mL anLnEerR. .COM PTS: 1

DIF:

Cognitive Level: 2

REF: pp. 58-59

8. For testing of moderate complexity, quality control requires: a. using fresh reagent daily. b. performing control procedures monthly. c. performing control procedures using at least two levels of control material each

day of testing. d. both a and c. ANS: C

Quality control (QC) consists of procedures used to detect errors that result from test system failure, adverse environmental conditions, and variance in operator performance, as well as the monitoring of the accuracy and precision of test performance over time. Accrediting agencies require monitoring and documentation of quality assessment records. CLIA ’88 states, “The laboratory must establish and follow written quality control procedures for monitoring and evaluating the quality of the analytical testing process of each method to assure the accuracy and reliability of patient test results and reports.” For tests of moderate complexity, traditionally CLIA states that laboratories comply with the more stringent requirements of performing and documenting control procedures using at least two levels of control material each day of testing and following the manufacturer’s instructions for quality control.


PTS: 1

DIF:

Cognitive Level: 2

REF: p. 61

9. A control specimen should be: a. carried through the entire test procedure. b. treated exactly in the same way as any patient’s specimen. c. assayed at least once a week. d. both a and b. ANS: D

A control specimen must be carried through the entire test procedure and be treated in exactly the same way as any unknown specimen and affected by any or all of the variables that affect the unknown specimen. The use of control specimens is based on the fact that repeated determinations on the same or different portions (or aliquots) of the same sample will not, as a rule, give identical values for any particular constituent. Many factors can produce variations in laboratory analyses. With a properly designed control system, it is possible to monitor testing variables. PTS: 1

DIF:

Cognitive Level: 2

REF: p. 62

10. Accuracy describes: a. how close a test result is to the true value. b. comparison of an instrument measure or reading to a known physical constant. c. how close the test results are to one another when repeated analyses of the same

material are performed. d. the purity of a substance. ANS: A

Accuracy describes how clT osEeSaTteBsA t rNesKuS ltEisLtL oE thRe . trC ueOvMalue. Calibration is the comparison of an instrument measure or reading to a known physical constant. Control represents a specimen with a known value that is similar in composition to the patient’s blood. Precision describes how close the test results are to one another when repeated analyses of the same material are performed. Standards are highly purified substances of a known composition. PTS: 1

DIF:

Cognitive Level: 2

REF: p. 62

11. Sensitivity is defined as the: a. proportion of cases with a specific disease or condition that give a positive test

result. b. proportion of cases with absence of the specific disease or condition that gives a

negative test result. c. proportion of a population that has the disease. d. number of patients with a normal test result who do not have the disease. ANS: A


The sensitivity of a test is defined as the proportion of cases with a specific disease or condition that give a positive test result. Sensitivity represents how much of a given substance is measured; the more sensitive the test, the smaller the amount of assayed substance that is measured. The specificity of a test is defined as the proportion of cases with absence of the specific disease or condition that gives a negative test result. In comparison, the prevalence of a disease is the proportion of a population that has the disease. This is in contrast to the incidence of a disease, which is the number of subjects found to have the disease within a defined time period, such as a year, in a population of 100,000. A positive predictive value for a test indicates the number of patients with an abnormal test result who have the disease, compared with all patients with an abnormal result. A negative predictive value for a test indicates the number of patients with a normal test result who do not have the disease, compared with all patients with a normal (negative) result. PTS: 1

DIF:

Cognitive Level: 2

REF: p. 63

12. One way to determine a control range for a particular analysis is to: a. assay an aliquot of the control serum with the regular batch of assays for 15 to 25

days. b. treat a control sample exactly like an unknown specimen. c. estimate the most reasonable value. d. do both a and b. ANS: B

The control range is determined by assaying an aliquot of the control serum with the regular batch of assays for 15 to 25 days. It is essential that the control sample be treated exactly like an unknown specimen. PTS: 1

DIF:

TCEogSnTitiBveALNeKveSl:E2LLER.COM

REF: p. 65

13. Sources of variance or error include: a. age of a specimen. b. procedural factors. c. physical activity level of a patient. d. all of the above. ANS: D

Sources of variation or errors can be related to sampling factors or procedural factors. Sources of variance that involve the sample include the time of day when the sample is obtained, the patient’s position (lying down or seated), the patient’s state of physical activity (in bed, ambulatory, or physically active), the interval since last eating (fasting or not), and the time interval and storage conditions between the obtaining of the specimen and its processing by the laboratory. The aging of the sample is another source of error. In addition, procedural factors involve aging of chemicals or reagents; personal bias or limited experience of the person performing the determination; and laboratory bias because of variations in standards, reagents, environment, methods, or apparatus. There may also be experimental error resulting from changes in the method used for a particular determination, changes in instruments, or changes in personnel. PTS: 1

DIF:

Cognitive Level: 2

14. Quality control can be monitored by using:

REF: p. 65


a. b. c. d.

Levey-Jennings charts. Westgard rules. bar graph information. both a and b.

ANS: D

Levey-Jennings charts are conventional in most laboratories to plot the daily control specimen values on a quality control chart. Westgard rules are commonly formulated to analyze data in control charts based on statistical methods. These rules define specific performance limits for a particular assay and can be used to detect both random and systematic errors. PTS: 1

DIF:

Cognitive Level: 2

REF: p. 65 | p. 67

15. If a violation of Westgard rules occurs, the correct action is: a. accept the results if only a warning is violated. b. reject the results when a mandatory rule is violated. c. increase the retesting range if either a warning or a mandatory rule is violated. d. all of the above. ANS: D

If a violation of a Westgard rule occurs, actions to take are: 1. accept the results of the testing run if only a warning is violated, 2. reject the results of the testing run when a mandatory rule is violated, 3. increase the retesting range for a particular assay if either a warning or a mandatory rule is violated. PTS: 1

DIF:

Cognitive Level: 2

REF: p. 67

MATCHING

Match the type of error with the appropriate example. a. active error b. latent error 1. failing to properly identify a patient prior to venipuncture 2. chronic shortage of qualified laboratory staff 1. ANS: A REF: p. 57 2. ANS: B REF: p. 57

PTS: 1

DIF: Cognitive Level: 2

PTS: 1

DIF: Cognitive Level: 2

Match the term with the correct description. a. a gradual change in the control sample results b. a sudden and sustained change in one direction in control sample values c. when random error or lack of precision increases 3. shift 4. trend or drift 5. dispersion


3. ANS: B REF: pp. 66-67 4. ANS: A REF: pp. 66-67 5. ANS: C REF: pp. 66-67

PTS: 1

DIF:

Cognitive Level: 2

PTS: 1

DIF:

Cognitive Level: 2

PTS: 1

DIF:

Cognitive Level: 2


Chapter 04: Phlebotomy: Collecting and Processing Blood Turgeon: Linné & Ringsrud’s Clinical Laboratory Science, 7th Edition MULTIPLE CHOICE 1. Quality assessment: a. requires establishing policies that maintain and control processes involving the

patient. b. requires establishing policies that maintain and control the process of laboratory

analysis of specimens. c. regulates collection, transportation, and processing of patient specimens. d. all of the above. ANS: D

The accuracy of a test result begins with the quality of the specimen received by the laboratory. This quality depends on how a specimen was collected, transported, and processed. As it applies to the clinical laboratory, quality assessment requires establishing policies that maintain and control processes involving the patient and the process of laboratory analysis of specimens. Quality assessment includes monitoring the following: • Preparation of a patient for any specimens to be collected • Collection of valid samples • Proper specimen transport • Performance of the requested laboratory analyses • Validation of test results • Recording and reporting the assay results • Transmitting test resultTs E toStT heBpAaN tieKnSt’E sL mL edEicRa. l rCeO coMrd • Documentation, maintenance, and availability of records describing quality assurance practices and quality control measures PTS: 1

DIF:

Cognitive Level: 2

2. Most mistakes in laboratory testing involve a. preanalytical (pre-evaluation) b. analytical (evaluation) c. postanalytical (post-evaluation) d. a combination of any of the above

REF: p. 76

errors.

ANS: A

More than two thirds of laboratory errors are caused by preanalytical errors (mistakes made before testing). Most of these mistakes are related to specimen collection and handling. PTS: 1

DIF:

Cognitive Level: 1

REF: p. 79

3. The relationship between patients and physicians and other health care professionals is

described in the recently published: a. patient’s bill of rights. b. patient care partnership document. c. Clinical Laboratory Improvement Amendment (CLIA) regulations. d. both a and b.


ANS: B

The delivery of health care involves a partnership between patients and physicians and other health care professionals. The American Hospital Association has developed the patient care partnership document that replaces the old patient’s bill of rights. This document stresses: • High-quality hospital care • A clean and safe environment • Patient involvement in their own care • Protection of privacy • Help when leaving the hospital • Help with billing claims PTS: 1

DIF: Cognitive Level: 1

REF: pp. 76-77

4. The first tier of infection control for health care professionals is to use a. standard b. contact c. airborne d. droplet

precautions.

ANS: A

The first tier of infection control is the practice of standard precautions. A second tier of an infection control system was developed to provide additional precautions to control the transmission of bacteria under special circumstances when standard precautions alone may not be enough. Second-tier precautions are divided into three basic categories: contact, airborne, and droplet. PTS: 1

DIF:

TCEogSnTitiBveALNeKveSl:E2LLER.COM

REF: p. 78

5. The most frequently analyzed specimen in the clinical laboratory is: a. urine. b. body fluids. c. blood. d. feces. ANS: C

Blood is the specimen most frequently analyzed in the clinical laboratory. Urine specimens and body fluids are also sent in great numbers. Fecal specimens and other miscellaneous specimens such as throat cultures and swabs from wound abscesses are also sent to the laboratory for analysis. PTS: 1

DIF:

Cognitive Level: 1

REF: p. 78

6. For a patient to be in a fasting state, he or she must abstain from food and liquids other than

water for a. 2 to 4 b. 4 to 8 c. 8 to 12 d. 12 to 24

hours.

ANS: C

The fasting state is defined as having had no food or liquid other than water for 8 to 12 hours.


PTS: 1

DIF:

Cognitive Level: 1

REF: p. 79

7. The pale, straw-colored fluid visible in clotted blood in a test tube is: a. plasma. b. serum. c. fibrinogen. d. fibrin. ANS: B

Whole blood that is allowed to clot normally in a test tube produces the pale-yellow fluid serum. When fresh whole blood is mixed with anticoagulants (substances that prevent blood clotting), the blood can be separated into plasma, a straw-colored fluid, and the cellular components: erythrocytes, leukocytes, and platelets (thrombocytes). PTS: 1

DIF:

Cognitive Level: 1

REF: p. 79

8. Environmental factors that can affect the quality of evacuated tubes can be: a. sunlight. b. ambient temperature. c. humidity. d. all of the above. ANS: D

A variety of environmental factors can affect the quality of evacuated tubes used to collect blood. These factors can then influence the published expiration dates of the evacuated tubes. PTS: 1

DIF:

TCEogSnTitiBveALNeKveSl:E1LLER.COM

REF: p. 80

9. The draw volume of blood into an evacuated tube can be affected at greater than

feet of

altitude. a. 2000 b. 5000 c. 15,000 d. 25,000 ANS: B

In situations in which blood is drawn at high altitudes (>5000 feet), the draw volume may be affected. The resulting draw volume will be lower. PTS: 1

DIF:

Cognitive Level: 1

REF: p. 80

10. The shelf life of an evacuated tube is determined by: a. stability of the additive. b. vacuum retention. c. clinical use of the additive. d. both a and b. ANS: D

Shelf life of an evacuated tube is defined by the stability of the additive and vacuum retention. PTS: 1

DIF:

Cognitive Level: 1

REF: p. 80


11. The anticoagulant of choice for blood cell counting and sizing is: a. EDTA. b. sodium citrate. c. heparin. d. sodium fluoride. ANS: A

EDTA is the anticoagulant of choice for blood cell counting and sizing because it produces less shrinkage of red blood cells and less of an increase in cell volume on standing. Sodium citrate is appropriate for coagulation studies. Heparin is used as an anticoagulant commonly used in chemistry and special chemistry testing. Sodium fluoride is used primarily for preservation of blood glucose specimens to prevent glycolysis or destruction of glucose. PTS: 1

DIF:

Cognitive Level: 1

REF: p. 82

12. The first critical step in blood collections is: a. introduction of the phlebotomist. b. a brief explanation of the procedure and tests to be performed. c. patient identification. d. a check of test requisitions for ordered tests. ANS: C

Patient identification is the critical first step in blood collection. It is necessary both to ask the patient’s name and to check the identification band that is physically attached to the patient. When the patient is unable to give his or her name, or when identification is attached to the bed or is missing, nursing personnel should be asked to physically identify the patient. Verbal identification of a patient sT hoEuSldTbBeAnN otKeS dE onLtL heEtRe. stCreOqM uisition. PTS: 1

DIF:

Cognitive Level: 2

REF: p. 85

13. Major errors in the identification of patients can include: a. absent wristband. b. erroneous information on the wristband. c. partially missing information on the wristband. d. all of the above. ANS: D

A CAP-conducted study identified six major types of wristband errors, as follows: • Absent wristband • Wrong wristband • More than one wristband with different information • Partially missing information on the wristband • Erroneous information on the wristband • Illegible information on the wristband PTS: 1

DIF:

Cognitive Level: 2

REF: p. 85

14. When collecting blood in multiple evacuated tubes, the correct order of draw should start

with: a. light blue (citrate).


b. yellow (blood cultures). c. lavender (EDTA). d. white. ANS: B

The correct order of draw of multiple evacuated tubes is: yellow, light blue, gold, light green, lavender, white, and gray. PTS: 1

DIF:

Cognitive Level: 2

REF: p. 85

15. Blood for the screening of newborns is for certain conditions such as: a. phenylketonuria. b. hypothyroidism. c. galactosemia. d. all of the above. ANS: D

Most states have passed laws requiring that newborn infants be screened for certain diseases that can result in serious abnormalities, including mental retardation, if they are not diagnosed and treated early. These diseases include phenylketonuria (PKU), galactosemia, hypothyroidism, and hemoglobinopathies. PTS: 1

DIF:

Cognitive Level: 1

16. A popular at-home point-of-care test (POCT) is for a. thyroid b. glucose c. electrolyte d. cholesterol

REF: p. 87

testing.

ANS: B

POCT for glucose is performed at home by many outpatient diabetics, using their own blood and one of several glucose measuring devices. It is important for patients with diabetes, especially those with insulin-dependent diabetes mellitus, to monitor their own blood glucose levels several times a day and to be able to adjust their dosage of insulin accordingly to maintain good glucose control. PTS: 1

DIF:

Cognitive Level: 1

REF: p. 88

17. Specimens must reach the laboratory for testing in less than: a. 30 minutes. b. 1 hour. c. 2 hours. d. 8 hours. ANS: C

Specimens should be processed to the point at which they can be properly stored so that the constituents to be measured will not be altered. Specimens collected at stations away from the testing laboratory need to guarantee that delivery will be made in less than 2 hours from collection and that specimens have been stored properly, including refrigeration, or freezing if necessary.


PTS: 1

DIF:

Cognitive Level: 1

REF: p. 90

18. The most common cause of an abnormal appearance of a collected blood specimen is: a. hemolysis. b. icterus (jaundice). c. lipemia. d. excess anticoagulant. ANS: A

Hemolysis in specimens is perhaps the most common cause of an abnormal appearance. Hemolyzed serum or plasma is unfit as a specimen for several chemistry determinations. A specimen that is hemolyzed appears red, usually clear red, because the red blood cells have been lysed and the hemoglobin has been released into the liquid portion of the blood. Icteric (yellow) serum or plasma is another specimen with an abnormal appearance. The abnormal color of the serum can interfere with photometric measurements. Lipemic plasma or serum takes on a milky white color. The presence of lipid, or fats, in serum or plasma can cause this abnormal appearance. Use of a lipemic serum specimen does not interfere with some chemical determinations but may interfere with others (e.g., triglyceride assay). PTS: 1

DIF:

Cognitive Level: 1

REF: p. 91


Chapter 05: The Microscope Turgeon: Linné & Ringsrud’s Clinical Laboratory Science, 7th Edition MULTIPLE CHOICE 1. Total magnification is the product of the magnification of the objective

_ the

magnification of the ocular. a. added to b. multiplied by c. divided by d. subtracted from ANS: B

The total magnification observed is the product of the magnifications of these two lenses. In other words, the magnification of the objective multiplied by the magnification of the ocular equals the total magnification. PTS: 1

DIF: Cognitive Level: 1

REF: p. 109

2. The definition of microscopic resolution is: a. an indication of how small individual dots can be and still be recognizable. b. an indication of how close individual objects (dots) can be and still be

recognizable. c. the point at which the appearance of two dots results in a dumbbell appearance. d. both a and b. ANS: D

Resolution indicates how small and how close individual objects (dots) can be and still be recognizable. Practically, the resolving power is the limit of usable magnification. Further magnification of two dots that are no longer resolvable would be “empty magnification” and would result in a dumbbell appearance. PTS: 1

DIF:

Cognitive Level: 2

REF: p. 106

3. The numerical aperture (NA) is: a. an index of the resolving power of a lens. b. the distance from an object being viewed to the lens. c. inscribed on each objective lens. d. both a and b. ANS: D

A term encountered in microscopy is NA. The NA of a lens can be defined as an index or measurement of the resolving power. As the NA increases, objects can be positioned closer and still be distinguished from each other; that is, the greater the NA, the greater the resolving power of a lens. The NA can also be defined as an index of the light-gathering power of a lens, a means of describing the amount of light entering the objective. Any particular lens has a constant-rated NA, and this value depends on the radius of the lens and its focal length (the distance from the object being viewed to the lens or the objective); however, decreasing the amount of light passing through a lens will decrease the actual NA. The rated NA is inscribed on each objective lens.


PTS: 1

DIF:

Cognitive Level: 1

4. The term ocular is another name for the microscopic a. objective b. eyepiece c. diaphragm d. nosepiece

REF: p. 106

that magnifies an image.

ANS: B

The ocular, or eyepiece, is a lens that magnifies the image formed by the objective. The usual magnification of the ocular is 10 (10); however, 5 and 20 oculars are also generally available. Most microscopes have two oculars and are called binocular microscopes. PTS: 1

DIF:

Cognitive Level: 2

REF: p. 106

5. The magnifying powers of the usual microscope objectives are: a. 5, 10, 20. b. 10, 20, 40. c. 10, 40, 80. d. 10, 40, 100. ANS: D

The objectives are the major part of the magnification system. There are usually three objectives on each microscope, with magnifying powers of 10, 40, and 100. PTS: 1

DIF:

Cognitive Level: 1

REF: p. 110

6. The definition of the microscopic working distance is the distance from the: a. bottom of the objective to the material being studied. b. ocular to the bottom of the objective. c. ocular to the material being studied. d. top of the objective to the material being studied. ANS: A

The working distance is the distance from the bottom of the objective to the material being studied. The greater the magnifying power of a lens, the smaller is the focal length and thus the working distance. This becomes very important when the microscope is being used, because the working distance is very short for the 40 (4 mm) and 100 (1.8 mm) objectives. For this reason, correct focusing habits are necessary to prevent damaging the objectives against the slide on the stage. PTS: 1

DIF:

Cognitive Level: 2

7. Oil needs to be removed from the: a. ocular. b. 10 objective. c. 40 objective. d. 100 objective. ANS: D

REF: p. 110


Oil must be removed from the oil-immersion (100) objective immediately after use by wiping with clean lens paper. If not removed, oil may seep inside the lens or dry on the outside surface of the objective. The high-dry (40) objective should never be used with oil; however, if this or any other objective or microscope part comes into contact with oil, it should be cleaned immediately. PTS: 1

DIF:

Cognitive Level: 2

REF: p. 111

8. The definition of Köhler illumination is the: a. strength of the transmitted light. b. light path through the microscope. c. light path passing through the condenser of the microscope. d. light path passing through the diaphragm of the microscope. ANS: B

When properly aligned, the microscope is adjusted in such a way that the light path through the microscope, from the light source to the eye of the observer, is correct. This is referred to as Köhler illumination. PTS: 1

DIF:

Cognitive Level: 1

REF: p. 112

9. The most common type of microscope illumination system used in the clinical laboratory is: a. brightfield. b. phase contrast. c. polarizing. d. fluorescent. ANS: A

With few exceptions, brightfield illumination has been the primary type of microscope illumination system used in the routine clinical laboratory. Other illumination systems are also available, including phase-contrast, interference-contrast, polarizing, darkfield, fluorescence, and electron microscopy. PTS: 1

DIF:

Cognitive Level: 2

REF: p. 114

10. The phase-contrast microscope differs from the brightfield microscope because of changes in: a. the objective. b. the condenser. c. the oculars. d. both a and b. ANS: D

The phase-contrast microscope is basically a brightfield microscope with changes in the objective and the condenser. An annular diaphragm, or ring, is put into (or below) the condenser. This condenser annulus is designed to let a hollow cone or “doughnut” of light pass through the condenser to the specimen. A corresponding absorption ring is fitted into the objective. Each phase objective must have a corresponding condenser annulus. Use of each phase objective requires an adjustment of the condenser to “match” the annular diaphragm and the phase absorption ring. PTS: 1

DIF:

Cognitive Level: 2

REF: p. 115


11. In a polarizing microscope, the polarizing filters are: a. placed on top of each other. b. placed at right angles to each other. c. red and blue in color. d. used to brighten the field being observed. ANS: B

A polarizing filter may be defined as a sieve that takes ordinary light waves, which vibrate in all directions, and allows only light waves of one orientation (north-south or east-west) to pass through the filter. One of the polarizers is rotated until the two are at right angles to each other. PTS: 1

DIF:

Cognitive Level: 1

REF: p. 118

12. A brightfield microscope may be converted to a darkfield microscope by: a. use of a special darkfield condenser in place of the usual condenser. b. replacement of the usual filters. c. use of a special ocular. d. changing the type of light source. ANS: A

Any brightfield microscope may be converted to a darkfield microscope by use of a special darkfield condenser in place of the usual condenser. PTS: 1

DIF:

Cognitive Level: 2

REF: p. 114

13. Fluorescent objects (naturaTl,EsS taT inB edA, N orKlS abEeL leL d)EcRa. nC beOsMeen with a fluorescent microscope

because they absorb the a. blue; green b. red; orange c. green; purple d. purple; red

light and emit light of yellow or

wavelength.

ANS: A

Certain objects have the ability to fluoresce, which means they absorb light of certain very short (ultraviolet) wavelengths and emit light of longer (visible) wavelengths. In fluorescence microscopy with transmitted light and a compound microscope, the darkfield condenser is preceded by a special exciter filter that allows only shorter wavelength blue light to pass and cross on the specimen plane. If the specimen contains an object that fluoresces (either naturally or because of staining or labeling with certain fluorescent dyes), it will absorb the blue light and emit light of a longer yellow or green wavelength. PTS: 1

DIF:

Cognitive Level: 1

14. An artificial neural network: a. simulates the action of a robot. b. simulates human brain processing of information. c. simulates automated instruments. d. follows strict algorithms. ANS: B

REF: p. 115


An artificial neural network (ANN) is an information-processing model that simulates the way the human brain processes information. PTS: 1

DIF:

Cognitive Level: 1

REF: p. 118

15. The most dramatic change in microscopy in the last 3 decades, has been: a. invention of the electron microscope. b. the ability to digitize images. c. the ability to transmit microscopic images electronically. d. both b and c. ANS: D

The most dramatic change in microscopy over the last 3 decades is the ability to digitize image specimens and transmit these images electronically for remote analysis. Recent progress in microscope design is opening new doors to multidimensional microscopy. Digitalization increases our ability to enhance features, extract information, or modify images. Digitized images can be shared with remote observers for workflow automation, image acquisition, storage, and retransmission. This capability is now called virtual microscopy. PTS: 1

DIF:

Cognitive Level: 2

REF: p. 118


Chapter 06: Systems of Measurement, Laboratory Equipment, and Reagents Turgeon: Linné & Ringsrud’s Clinical Laboratory Science, 7th Edition MULTIPLE CHOICE 1. What is the standard unit for the measurement of length in the metric system? a. Meter b. Gram c. Liter d. Feet ANS: A

The standard unit for the measurement of length or distance is the meter (m). One meter equals 39.37 inches (in), slightly more than a yard in the English system. PTS: 1

DIF:

Cognitive Level: 1

REF: p. 134

2. What is the standard unit of volume in the metric system? a. Meter b. Gram c. Liter d. Feet ANS: C

In the clinical laboratory the standard unit of volume is the liter (L). It was not included in the list of base units of the SI system because the liter is a derived unit. The standard unit of 3 ER.COM NK volume in the SI system isTthEeScT ubBicAm etS erE(L mL ). In 1964 the Conférence Générale des Poids et Mésures (CGPM) accepted the litre (liter) as a special name for the cubic decimeter. One liter is slightly more than 1 quart (qt) in the English system (1 L = 1.06 qt). PTS: 1

DIF:

Cognitive Level: 1

REF: p. 134

3. Convert 50° F to ° C (Celsius). a. 5 b. 10 c. 20 d. 40 ANS: B

°C= (°F – 32). Therefore, ° C = (50° F – 32). °C= (18°F) = 10° C. PTS: 1

DIF:

4. Convert 20° C to ° F. a. 32 b. 46 c. 68 d. 100

Cognitive Level: 3

REF: p. 135


ANS: C

°F= (° C + 32). °F= (20° C) + 32. ° F = 36° C + 32 = 68° F. PTS: 1

DIF:

Cognitive Level: 3

REF: p. 135

5. A pipette that has been calibrated to deliver a fixed volume of liquid by drainage is known as a. b. c. d.

pipette. a volumetric a serologic a graduated both b and c

ANS: A

For most general laboratory use, there are two main types of manual pipettes: the volumetric (or transfer) pipette and the graduated (or measuring) pipette. A pipette that has been calibrated to deliver a fixed volume of liquid by drainage is known as a volumetric, or transfer pipette. These pipettes consist of a cylindrical bulb joined at both ends to narrow glass tubing. Graduated pipettes are another way to deliver a particular amount of liquid, but this pipette delivers the amount of liquid contained between two calibration marks on a cylindrical tube, or pipette. Another pipette used in the laboratory is the serologic pipette. It is much like the graduated pipette in appearance. PTS: 1

DIF:

Cognitive Level: 2

REF: p. 139

6. The balance is the moTsE tS coTmBmAoNnK lySuE seLdLbE alR an.cC eO inMa clinical laboratory. a. mechanical analytical b. electronic analytical c. triple beam d. None of the above are correct. ANS: B

The electronic analytical balance is easy to use and has replaced the traditional mechanically operated analytical balance in most clinical laboratories. PTS: 1

DIF:

Cognitive Level: 2

REF: p. 143

7. The purest grade of reagent water is: a. deionized. b. type I. c. type II. d. type III. ANS: B

Type I reagent water is the most pure form of water and should be used for procedures that require maximum water purity. Type II reagent water is used for general laboratory tests that do not require type I water. Type III reagent water can be used for some qualitative laboratory tests, such as those done in general urinalysis. Type III water can be used as a water source for preparation of type I or type II water and for washing and rinsing laboratory glassware.


PTS: 1

DIF:

Cognitive Level: 2

8. Type I reagent water must be used a. immediately b. within 1 working day c. within 24 hours d. within a month

REF: p. 148

after its production.

ANS: A

It is important to store reagent water appropriately. Type I water must be used immediately after its production to prevent carbon dioxide from being absorbed into it. Types II and III water should be used as soon as possible to prevent contamination with airborne microbes. PTS: 1

DIF:

Cognitive Level: 2

REF: p. 148

9. Distilled water is defined as water from which: a. iron, magnesium, calcium, and other minerals have been removed by heating and

cooling the vapor. b. ions have been removed by charcoal filters. c. microbiological organisms have been removed. d. All of the above are correct. ANS: D

Distilled water is defined as water from which iron, magnesium, calcium, and other minerals have been removed by heating and cooling the vapor (distillation). The process of distillation also removes microbiological organisms, but volatile impurities such as carbon dioxide, chlorine, and ammonia areTnE otSrT em t hCaO sM been distilled meets the specifications BoAvNedK.SWEaLteLr EthRa. for types II and III water. PTS: 1

DIF:

Cognitive Level: 1

REF: p. 149

10. In the process of deionization, water is: a. heated and cooled to liquefy the condensation. b. passed through a resin column containing positively (+) and negatively (–) charged

particles. c. first heated and then passed through a resin column containing charged particles. d. both b and c. ANS: B

In the process of deionization, water is passed through a resin column containing positively (+) and negatively (–) charged particles. These particles combine with ions present in the water to remove them; this water is known as deionized water. PTS: 1

DIF:

Cognitive Level: 2

11. A reagent is defined as: a. any concentrated solution used for testing. b. any substance used to produce a chemical reaction. c. a solution contained in an instrument. d. a solution with a known concentration.

REF: p. 149


ANS: B

A reagent is defined as any substance used to produce a chemical reaction. PTS: 1

DIF:

Cognitive Level: 1

REF: p. 150

12. The purest grade of chemicals used in the preparation of reagents is a. analytic (AR) b. chemically pure (CP) c. United States Pharmacopeia (USP) and National Formulary (NF) d. technical or commercial

grade.

ANS: A

Analytic reagent (AR) grade chemicals are of a high degree of purity and are used often in the preparation of reagents in the clinical laboratory. Chemically pure (CP) grade are sufficiently pure to be used in many analyses in the clinical laboratory; they may not be acceptable for research and various clinical laboratory techniques unless they have been specifically analyzed for the desired procedure. USP and NF grade are generally less pure than CP grade. Technical or commercial grade chemicals are used only for industrial purposes and are generally not used in the preparation of reagents for the clinical laboratory. PTS: 1

DIF:

Cognitive Level: 1

REF: p. 150

13. The purpose of a material safety data sheet (MSDS) is to provide information about: a. possible hazards. b. safe handling instructions. c. storage and disposal of the specific chemical that it accompanies. d. all of the above. ANS: D

An MSDS accompanies the shipment of all hazardous chemicals and should be available in the laboratory for anyone to review. The MSDS contains information about possible hazards, safe handling, storage, and disposal of the particular chemical it accompanies. PTS: 1

DIF:

Cognitive Level: 2

REF: p. 150

14. Flammable solvents should be stored in: a. a refrigerator. b. a freezer. c. special safety cans. d. a draft-free location. ANS: C

Flammable solvents such as acetone and ether should always be stored in special safety cans or other appropriate storage devices and in approved storage units. PTS: 1

DIF:

Cognitive Level: 2

REF: p. 150

15. Solutions prepared from the most highly purified types of chemicals available are

solutions. a. reagent b. standard c. molar


d. acidic ANS: B

Standard solutions are prepared from the most highly purified types of chemicals available. This group includes primary, reference, and certified standards. PTS: 1

DIF:

Cognitive Level: 1

REF: p. 151

16. The labels on bottles should include: a. the name and concentration of the reagent. b. the date on which the reagent was prepared. c. the initials of the person who made the reagent. d. all of the above. ANS: D

All labels on bottles should include the name and concentration of the reagent, the date on which the reagent was prepared, and the initials of the person who made the reagent. PTS: 1

DIF:

Cognitive Level: 2

17. Each new lot or batch of reagent should be tested with a. controls b. standards c. calibrators d. all of the above

REF: p. 151

with the existing reagent.

ANS: D

Controls, standards, and calibrators are all means of testing a new lot or batch of reagent with the existing reagent. PTS: 1

DIF:

Cognitive Level: 2

REF: p. 151


Chapter 07: Laboratory Mathematics and Solution Preparation Turgeon: Linné & Ringsrud’s Clinical Laboratory Science, 7th Edition MULTIPLE CHOICE 1. In rounding off numbers: a. when the digit next to the last one to be retained is less than 5, the last digit should

be left unchanged. b. when the digit next to the last one to be retained is greater than 5, the last digit is

increased by 1. c. if the additional digit is 5, the last digit reported is changed to the nearest even number. d. follow all of the above. ANS: D

Test results sometimes produce insignificant digits. It is then necessary to round off the numbers to a chosen number of significant value so as not to imply an accuracy of precision greater than the test is capable of delivering. The following general rules may be used in rounding off decimal values to the proper place. When the digit next to the last one to be retained is less than 5, the last digit should be left unchanged. When the digit next to the last one to be retained is greater than 5, the last digit is increased by 1. If the additional digit is 5, the last digit reported is changed to the nearest even number. PTS: 1

DIF:

Cognitive Level: 1

REF: p. 164

2. Calculate the concentration expressed in percent, if 10 g of NaOH is diluted to 500 mL with

water. a. 1% b. 2% c. 5% d. 10% ANS: B

If 10 g of NaOH is diluted to 500 mL with water, a proportion can be set up to solve this problem: x = 2% solution (preferably expressed as 2 g/dL) Remember that the percent expression is based on how much solute is present in 100 mL (or 1 dL) of the solution. PTS: 1

DIF:

Cognitive Level: 3

REF: pp. 165-166

3. What is the molarity of a solution containing 50 g of NaCl per liter? a. 0.2M b. 0.5M c. 0.9M d. 1.5M


ANS: C

Molarity equals the number of moles per liter, and the number of moles equals the grams divided by the gram-molecular weight. Step 1: Find the gram-molecular weight of NaCl. It is 58.5 g (Na = 23 and Cl = 35.5). Step 2: Find the moles per liter.

Step 3: The number of moles per liter of solution equals the molarity; the solution in the example is therefore 0.855M, rounded off to 0.9M. PTS: 1

DIF:

Cognitive Level: 3

REF: p. 166

4. The term dilution refers to: a. the volume or number of parts of the substance to be diluted in the total volume. b. the volume or number of parts of the substance to be diluted in the parts of the

final solution. c. an expression of concentration that indicates the relative amount of substance in

solution. d. all of the above. ANS: D

A dilution refers to the volume or number of parts of the substance to be diluted in the total volume, or parts, of the final solution. A dilution is an expression of concentration, not an expression of volume; it indicates the relative amount of substance in solution. PTS: 1

DIF:

Cognitive Level: 1

REF: p. 168

5. If a serum specimen is diluted 1:2 with buffer beginning in tube 2 and serially rediluted 1:2 in

each of five subsequent test tubes, what is the concentration of serum in tube 5? a. 0.5 b. 0.25 c. 0.125 d. 0.06 ANS: D

If a serum specimen is diluted 1:2 with buffer. A series of five tubes are prepared, in which each succeeding tube is rediluted 1:2. This is accomplished by placing 1 mL of diluent into each of four tubes (tubes 2 to 5). Tube 1 contains 1 mL of undiluted serum. Tube 2 contains 1 mL of undiluted serum plus 1 mL of diluent, resulting in a 1:2 dilution of serum. A 1-mL portion of the 1:2 dilution of serum is placed in tube 3, resulting in a 1:4 dilution of serum (1/2  1/2 = 1/4). A 1-mL portion of the 1:4 dilution from tube 3 is placed in tube 4, resulting in a 1:8 dilution (1/4  1/2 = 1/8). Finally, 1 mL of the 1:8 dilution from tube 4 is added to tube 5, resulting in a 1:16 dilution (1/8  1/2 = 1/6). The concentration of serum in terms of milliliters in each tube is calculated by multiplying the previous concentration (mL) by the succeeding dilution. In this example, tube 1 contains 1 mL of serum, tube 2 contains 1 mL  1/2 = 0.5 mL of serum, and tubes 3 to 5 contain 0.25, 0.125, and 0.06 mL of serum, respectively.


PTS: 1

DIF:

Cognitive Level: 3

REF: pp. 169-170

TRUE/FALSE 1. Laboratory results should only contain the digits necessary for the precision of the

determination. ANS: T

Using more digits than are necessary to calculate and report the results of a laboratory determination has several disadvantages. It is important that the number used contain only the digits necessary for the precision of the determination. Using more digits than necessary is misleading in that it ascribes more accuracy to the determination than is actually the case. There is also the danger of overlooking a decimal point and making an error in judging the magnitude of the answer. Digits in a number that are needed to express the precision of the measurement from which the number is derived are known as significant figures. Use the known accuracy of the method to determine the number of digits that are significant in the answer and, as a general rule, retain one more figure than this. Take the accuracy of the least accurate measurement, or the measurement with the least number of significant figures, as the accuracy of the final result. PTS: 1

DIF:

Cognitive Level: 1

REF: p. 164

2. A significant figure should contain only the digits in a number needed to express the precision

of the measurement. ANS: T

Using more digits than areTnE ecSeT ssB arA yNtoKcSaE lcL ulL atE eR an.dCrO epMort the results of a laboratory determination has several disadvantages. It is important that the number used contain only the digits necessary for the precision of the determination. Using more digits than necessary is misleading in that it ascribes more accuracy to the determination than is actually the case. There is also the danger of overlooking a decimal point and making an error in judging the magnitude of the answer. Digits in a number that are needed to express the precision of the measurement from which the number is derived are known as significant figures. Use the known accuracy of the method to determine the number of digits that are significant in the answer and, as a general rule, retain one more figure than this. Take the accuracy of the least accurate measurement, or the measurement with the least number of significant figures, as the accuracy of the final result. PTS: 1

DIF:

Cognitive Level: 1

REF: p. 164

3. The more digits that are used in laboratory assay results, the more precise the answer is. ANS: F


Using more digits than are necessary to calculate and report the results of a laboratory determination has several disadvantages. It is important that the number used contain only the digits necessary for the precision of the determination. Using more digits than necessary is misleading in that it ascribes more accuracy to the determination than is actually the case. There is also the danger of overlooking a decimal point and making an error in judging the magnitude of the answer. Digits in a number that are needed to express the precision of the measurement from which the number is derived are known as significant figures. Use the known accuracy of the method to determine the number of digits that are significant in the answer and, as a general rule, retain one more figure than this. Take the accuracy of the least accurate measurement, or the measurement with the least number of significant figures, as the accuracy of the final result. PTS: 1

DIF:

Cognitive Level: 1

REF: p. 164

4. Report a laboratory result with the most number of significant figures as the accuracy of the

final result. ANS: F

Using more digits than are necessary to calculate and report the results of a laboratory determination has several disadvantages. It is important that the number used contain only the digits necessary for the precision of the determination. Using more digits than necessary is misleading in that it ascribes more accuracy to the determination than is actually the case. There is also the danger of overlooking a decimal point and making an error in judging the magnitude of the answer. Digits in a number that are needed to express the precision of the measurement from which the number is derived are known as significant figures. Use the known accuracy of the method to determine the number of digits that are significant in the answer and, as a general ruTleE, S reT taB inAoNnK eS mE oL reLfiEgR ur. eC thOanM this. Take the accuracy of the least accurate measurement, or the measurement with the least number of significant figures, as the accuracy of the final result. PTS: 1

DIF:

Cognitive Level: 1

REF: p. 164

MATCHING

Match the term with the appropriate definition. a. the amount of matter per unit volume of a substance b. relates the weight of 1 mL of a solution and the weight of 1 mL of pure water at 4°C (1 g) 1. density 2. specific gravity 1. ANS: A REF: p. 165 2. ANS: B REF: p. 165

PTS: 1

DIF: Cognitive Level: 1

PTS: 1

DIF: Cognitive Level: 1

Match the following terms with an appropriate definition. a. a solution prepared from a stock solution and usually stored in the refrigerator b. one that contains a known, exact amount of the substance being measured


c. one that contains reagents used in the procedure but not the substance to be

measured 3. standard solution 4. working standard solution 5. blank solution 3. ANS: B REF: pp. 170-171 4. ANS: A REF: pp. 170-171 5. ANS: C REF: pp. 170-171

PTS: 1

DIF:

Cognitive Level: 1

PTS: 1

DIF:

Cognitive Level: 1

PTS: 1

DIF:

Cognitive Level: 1


Chapter 08: Basic and New Techniques in the Clinical Laboratory Turgeon: Linné & Ringsrud’s Clinical Laboratory Science, 7th Edition MULTIPLE CHOICE 1. One of the most frequently used techniques in the clinical laboratory is: a. spectrophotometry. b. chemiluminescence. c. nephelometry. d. electrophoresis. ANS: A

One of the techniques used most frequently in the clinical laboratory is photometry, or, specifically, absorbance or reflectance spectrophotometry. PTS: 1

DIF:

Cognitive Level: 2

2. The principle of photometry uses a. color b. color variation c. electrical resistance d. both a and b

REF: p. 178

to determine the concentrations of various substances.

ANS: D

Photometry uses color and color variation to determine the concentrations of various substances. A photometric component is employed in many of the automated analyzers EhLeLuEseRo.f CcoOlM currently in use in the cliniT caEl SlaTbB orA atN orKyS .T orimetry as a means of quantitative measurement depends primarily on two factors, the color itself and the intensity of the color. Any substance to be measured by spectrophotometry must be colored to begin with or must be capable of being colored. PTS: 1

DIF:

Cognitive Level: 2

REF: p. 178

3. Spectrophotometry measures the concentration of an unknown substance by: a. counting the number of particles in a solution. b. comparing an unknown colored substance to a standard solution. c. measuring the electrical voltage generated in a chemical reaction. d. measuring the amount of heat generated in a chemical reaction. ANS: B

When spectrophotometry is used as a method for quantitative measurement, the unknown colored substance is compared with a similar substance of known strength (a standard solution). PTS: 1

DIF:

Cognitive Level: 2

REF: p. 179

4. The term light is used to describe: a. radiant energy with wavelengths visible to the human eye. b. wavelengths bordering on those visible to the human eye. c. all forms of radiant energy.


d. both a and b. ANS: D

The term light is used to describe radiant energy with wavelengths visible to the human eye or wavelengths bordering on those visible to the human eye. PTS: 1

DIF:

Cognitive Level: 2

REF: p. 179

5. Beer’s law states: a. the concentration of a substance is directly proportional to the amount of light

absorbed. b. inversely proportional to the logarithm of the transmitted light. c. inversely proportional to the amount of light absorbed. d. both a and b. ANS: D

Beer’s law states that the concentration of a substance is directly proportional to the amount of light absorbed or inversely proportional to the logarithm of the transmitted light. (Percent transmittance and absorbance are related photometric terms.) Beer’s law is the basis for the use of photometry in quantitative measurement. PTS: 1

DIF:

Cognitive Level: 1

REF: p. 180

6. When reflectance spectrophotometry is used as an analytic method: a. the light reflected from the surface of a colorimetric reaction is used to measure the

amount of unknown colored product generated in the reaction. b. a beam of light is directed at a flat surface. c. the amount of light reflT ecEteSdTiB sA mN eaKsuSrE edLiL nE a Rre.flC ecOtaMnce spectrophotometer. d. all of the above are correct. ANS: D

Reflectance spectrophotometry is another quantitative spectrophotometric technique; the light reflected from the surface of a colorimetric reaction is used to measure the amount of unknown colored product generated in the reaction. A beam of light is directed at a flat surface, and the amount of light reflected is measured in a reflectance spectrophotometer. This technology has been employed in automated instrumentation, including many of the handheld instruments for bedside testing and smaller instruments used in physicians’ offices and clinics. PTS: 1

DIF:

Cognitive Level: 2

REF: p. 186

7. A popular at-home application of reflectance spectrophotometry in point-of-care testing

(POCT) is: a. blood volume estimation. b. drug monitoring. c. glucose monitoring. d. pH determination. ANS: C


This technology is used in many of the bedside or POCT using smaller handheld instruments as well as for home testing done by patients themselves. Common bedside testing and self-testing instruments used for quantitation of blood glucose (employing a single-test methodology) for maintaining good diabetic control are one example. Chemistry and therapeutic drug monitoring analyzer systems also employ this technology. PTS: 1

DIF:

Cognitive Level: 2

REF: p. 186

8. An analytic method that depends on light-scattering properties of antigen-antibody

complexes is: a. absorption spectrophotometry. b. reflectance spectrophotometry. c. nephelometry. d. ion-selective electrodes. ANS: C

Nephelometry has become increasingly more popular in diagnostic laboratories and depends on the light-scattering properties of antigen-antibody complexes. The quantity of cloudiness or turbidity in a solution can be measured photometrically. PTS: 1

DIF:

Cognitive Level: 2

REF: p. 187

9. Fundamental prerequisite for a nephelometry measurement is: a. formation of a macromolecular complex. b. the reaction between the protein being assayed and a specific antisera. c. formation of an insoluble complex. d. all of the above. ANS: A

Formation of a macromolecular complex is a fundamental prerequisite for nephelometric protein quantitation. The procedure is based on the reaction between the protein being assayed and a specific antiserum. Protein in a patient specimen reacts with specific nephelometric antisera to human proteins and forms insoluble complexes. PTS: 1

DIF:

Cognitive Level: 1

REF: p. 187

10. The principle of flow cytometry is based on the fact that: a. cells have an electrical charge. b. proteins can be precipitated. c. cells are stained in suspension with an appropriate fluorochrome. d. fluorescent dyes are nonspecific. ANS: C

The principle of flow cytometry is based on the fact that cells are stained in suspension with an appropriate fluorochrome, which may be either an immunologic reagent, a dye that stains a specific component, or some other marker with specified reactivity. Fluorescent dyes used in flow cytometry must bind or react specifically with the cellular component of interest such as reticulocytes, peroxidase enzyme, or DNA content. PTS: 1

DIF:

Cognitive Level: 2

REF: p. 188


11. The protein molecule formed when a foreign material is introduced into an immunocompetent

individual is a(n): a. antigen. b. antibody. c. allergen. d. reagent. ANS: B

When foreign material (called antigens or immunogens) is introduced into the body, protein molecules called antibodies are formed in response. For example, certain bacteria, when introduced into the body, elicit the production of specific antibodies. PTS: 1

DIF:

Cognitive Level: 1

REF: p. 190

12. Immunoassays depend on: a. antigen solubility. b. antibody complexity. c. antigen-antibody reactions. d. antibody solubility. ANS: C

Immunoassays utilize antigen-antibody reactions. In the clinical chemistry laboratory, antibodies are used to detect and measure an antigen, such as a drug or medication, present in the serum of the patient. PTS: 1

DIF:

Cognitive Level: 1

REF: p. 190

nS coTmBpA etN itK ivS eE imLm 13. A reaction format used in nToE LuEnRoc.hCem OiMcal assays includes: a. sandwich assays. b. two-site assays. c. excess reagent assays. d. all of the above. ANS: D

In noncompetitive assay, polyclonal or monoclonal antibodies are used. One of two major types of reaction formats is noncompetitive-excess reagent, two-site, and sandwich assays. PTS: 1

DIF:

Cognitive Level: 1

REF: p. 191

14. A newer analytic technology that is becoming the technology of choice by most

immunodiagnostic manufacturers is: a. chemiluminescence. b. electrophoresis. c. spectrophotometry. d. enzyme immunoassay. ANS: A

Chemiluminescence is being pursued as the technology of choice by most immunodiagnostic manufacturers. Chemiluminescence has excellent sensitivity and dynamic range. It does not require sample radiation, and nonselective excitation and source instability are eliminated. Most chemiluminescent reagents and conjugates are stable and relatively nontoxic.


PTS: 1

DIF:

Cognitive Level: 2

REF: p. 193

15. The analytic technology that relies on direct measurement of electrical potential caused by the

activity of free ions is: a. pH meter. b. ion-selective electrodes (ISEs). c. chromatography. d. gel electrophoresis. ANS: B

ISEs are a potentiometric method of analysis that involve the direct measurement of electrical potential caused by the activity of free ions. ISEs are designed to be sensitive toward individual ions. An example of an ISE universally used in the clinical laboratory is the pH electrode. Specialized probes, ISEs, can measure concentrations of ionic species other than hydrogen ions [H+], including fluoride, chloride, ammonia, sodium, potassium, calcium, sulfide, and nitrate. PTS: 1

DIF:

Cognitive Level: 2

REF: p. 197

16. The piece of equipment that measures the H+ is: a. pH meter. b. photometer. c. spectrophotometer. d. flow cytometer. ANS: A

The pH meter measures H+ concentration by sensing differences in the electrical charges inside and outside of the prToE beS. T TB heAdNifKfeSreEnLceLiEnRe. leC ctO ricMal charge between the two surfaces creates an electrical potential, or voltage, and this causes an electrical current to flow through the wire at the other end of the probe. PTS: 1

DIF:

Cognitive Level: 1

REF: p. 198

17. Coulometry measures the: a. amount of current passing between two electrodes in an electrochemical cell. b. color generated in a chemical reaction. c. heat generated in a chemical reaction. d. relative speed of current passing between two electrodes in an electrochemical cell. ANS: A

Coulometry measures the amount of current passing between two electrodes in an electrochemical cell. Clinical applications of coulometry include the FreeStyle Connect blood glucose monitoring system (Abbott Labs) in the point-of-care setting (hospitals and medical clinics) and an older application in the measurement of chloride ions in serum, plasma, urine, and other body fluids. PTS: 1

DIF:

Cognitive Level: 2

18. Electrophoresis is a technique for: a. separation. b. separation and purification of ions.

REF: p. 198


c. the separation of proteins. d. all of the above. ANS: D

Electrophoresis is a technique for separation and purification of ions, proteins, and other molecules of biochemical interest. It is used frequently in the clinical chemistry laboratory to separate serum proteins. PTS: 1

DIF:

Cognitive Level: 1

REF: p. 199

19. In chromatography: a. mixtures of solutes dissolved in a common solvent are separated from one another. b. mixtures of solvents are separated from one another. c. solutions with various pH measurements are separated from one another. d. either b or c are true. ANS: A

Mixtures of solutes dissolved in a common solvent are separated from one another by a differential distribution of the solutes between two phases. The solvent, one phase, is mobile and carries the mixture of solutes through the second phase. PTS: 1

DIF:

Cognitive Level: 2

REF: pp. 200-201

TRUE/FALSE 1. One criterion for a good standard curve is that the line is straight. ANS: T

The criteria for a good standard curve are the: • line is straight. • line connects all points. • line goes through the origin, or intersect, of the two axes. • origin of the graph paper is the point on the vertical and horizontal axes where there is 100%T and zero concentration. PTS: 1

DIF:

Cognitive Level: 2

REF: p. 182

2. One criterion for a good standard curve is that the line must connect two points. ANS: F

The criteria for a good standard curve are the: • line is straight. • line connects all points. • line goes through the origin, or intersect, of the two axes. • origin of the graph paper is the point on the vertical and horizontal axes where there is 100%T and zero concentration. PTS: 1

DIF:

Cognitive Level: 2

REF: p. 182


3. One criterion for a good standard curve is that the line goes through the origin, or intersect, of

the two axes. ANS: T

The criteria for a good standard curve are the: • line is straight. • line connects all points. • line goes through the origin, or intersect, of the two axes. • origin of the graph paper is the point on the vertical and horizontal axes where there is 100%T and zero concentration. PTS: 1

DIF:

Cognitive Level: 2

REF: p. 182

4. Quality control tests for spectrophotometers consist of checking wavelength accuracy. ANS: T

Quality control testing for spectrophotometry consists of checking: • Wavelength accuracy • Stray light • Linearity Wavelength accuracy is ensured when the wavelength indicated on the control dial is the actual wavelength of light passed by the monochromator. This is checked with standard absorbing solution or filters with maximum absorbance of known wavelength. Stray light refers to any wavelengths outside the band transmitted by the monochromator. Linearity is demonstrated when a change in concentration results in a straight-line calibration curve, as discussed under Beer’s law. PTS: 1

DIF:

Cognitive Level: 2

REF: p. 185

5. Quality control tests for spectrophotometers consist of checking nonlinearity. ANS: F

Quality control testing for spectrophotometry consists of checking: • Wavelength accuracy • Stray light • Linearity Wavelength accuracy is ensured when the wavelength indicated on the control dial is the actual wavelength of light passed by the monochromator. This is checked with standard absorbing solution or filters with maximum absorbance of known wavelength. Stray light refers to any wavelengths outside the band transmitted by the monochromator. Linearity is demonstrated when a change in concentration results in a straight-line calibration curve, as discussed under Beer’s law. PTS: 1

DIF:

Cognitive Level: 2

REF: p. 185

6. Quality control tests for spectrophotometers consist of checking stray light. ANS: T

Quality control testing for spectrophotometry consists of checking: • Wavelength accuracy


• •

Stray light Linearity Wavelength accuracy is ensured when the wavelength indicated on the control dial is the actual wavelength of light passed by the monochromator. This is checked with standard absorbing solution or filters with maximum absorbance of known wavelength. Stray light refers to any wavelengths outside the band transmitted by the monochromator. Linearity is demonstrated when a change in concentration results in a straight-line calibration curve, as discussed under Beer’s law. PTS: 1

DIF:

Cognitive Level: 2

REF: p. 185

7. Various enzymes are used in enzyme immunoassay. In order to be used, an enzyme must

fulfill certain criteria, including a high amount of stability. ANS: T

Various enzymes are used in enzyme immunoassay (see Table 6-2). Commonly used enzyme labels are horseradish peroxidase, alkaline phosphatase, glucose-6-phosphate dehydrogenase, and beta-galactosidase. To be used in EIA, an enzyme must fulfill a number of criteria, including: • A high amount of stability • Extreme specificity • Absence from the antigen or antibody • No alteration by inhibitor with the system PTS: 1

DIF:

Cognitive Level: 2

REF: p. 191

KS 8. Various enzymes are used T inEeSnT zyBmAeNim mE unLoL asEsR ay.. C InOoMrder to be used, an enzyme must fulfill certain criteria, including multiple antigen specificity. ANS: F

Various enzymes are used in enzyme immunoassay (see Table 6-2). Commonly used enzyme labels are horseradish peroxidase, alkaline phosphatase, glucose-6-phosphate dehydrogenase, and beta-galactosidase. To be used in EIA, an enzyme must fulfill a number of criteria, including: • A high amount of stability • Extreme specificity • Absence from the antigen or antibody • No alteration by inhibitor with the system PTS: 1

DIF:

Cognitive Level: 2

REF: p. 191

9. Various enzymes are used in enzyme immunoassay. In order to be used, an enzyme must

fulfill certain criteria, including absence from the antigen or antibody. ANS: T

Various enzymes are used in enzyme immunoassay (see Table 6-2). Commonly used enzyme labels are horseradish peroxidase, alkaline phosphatase, glucose-6-phosphate dehydrogenase, and beta-galactosidase. To be used in EIA, an enzyme must fulfill a number of criteria, including: • A high amount of stability


• • •

Extreme specificity Absence from the antigen or antibody No alteration by inhibitor with the system

PTS: 1

DIF:

Cognitive Level: 2

REF: p. 191

10. Various enzymes are used in enzyme immunoassay. In order to be used, an enzyme must

fulfill certain criteria, including no alteration by inhibitor with the system. ANS: T

Various enzymes are used in enzyme immunoassay (see Table 6-2). Commonly used enzyme labels are horseradish peroxidase, alkaline phosphatase, glucose-6-phosphate dehydrogenase, and beta-galactosidase. To be used in EIA, an enzyme must fulfill a number of criteria, including: • A high amount of stability • Extreme specificity • Absence from the antigen or antibody • No alteration by inhibitor with the system PTS: 1

DIF:

Cognitive Level: 2

REF: p. 191

MATCHING

Match the approximate wavelength with the observed color. Use an answer only once. a. not visible (infrared light) b. red c. orange d. yellow e. green f. blue g. violet h. not visible (ultraviolet light) 1. <380 2. 380-440 3. 440-500 4. 500-580 5. 580-600 6. 600-620 7. 620-750 8. >750 1. ANS: H REF: p. 180 2. ANS: G REF: p. 180 3. ANS: F REF: p. 180 4. ANS: E

PTS: 1

DIF:

Cognitive Level: 1

PTS: 1

DIF:

Cognitive Level: 1

PTS: 1

DIF:

Cognitive Level: 1

PTS: 1

DIF:

Cognitive Level: 1


REF: 5. ANS: REF: 6. ANS: REF: 7. ANS: REF: 8. ANS: REF:

p. 180 D p. 180 C p. 180 B p. 180 A p. 180

PTS: 1

DIF:

Cognitive Level: 1

PTS: 1

DIF:

Cognitive Level: 1

PTS: 1

DIF:

Cognitive Level: 1

PTS: 1

DIF:

Cognitive Level: 1


Chapter 09: Delivery of Laboratory Testing From Point of Care to Total Automation Turgeon: Linné & Ringsrud’s Clinical Laboratory Science, 7th Edition MULTIPLE CHOICE 1. Important characteristics of point-of-care testing (POCT) instruments include: a. rapid turnaround time. b. easy portability with single-use disposable reagent cartridges or test strips. c. easy-to-perform protocol with one or two steps. d. all of the above. ANS: D

Important characteristics of POCT instruments include: 1. Rapid turnaround time 2. Easy portability with single-use disposable reagent cartridges or test strips 3. Easy-to-perform protocol with one or two steps 4. Accuracy and precision of results comparable with central laboratory analyzers 5. Minimal quality control tracking 6. Storage at ambient temperature for reagents 7. Use of bar code technology for test packs, controls, and specimens 8. Economical cost and maintenance free 9. Software for automatic calibration, system lockouts, and data management 10. Hard copy or electronic data output that interfaces with a laboratory information system (LIS) or other tracking software PTS: 1

DIF:

TCEogSnTitiBveALNeKveSl:E1LLER.COM

REF: p. 214

2. A laboratory test that is not performed in a traditional laboratory is called a a. provider-performed microscopy b. waived c. moderately complex d. highly complex

test.

ANS: B

Diagnostic testing that is not performed within a traditional laboratory is called waived testing by The Joint Commission (TJC). The Clinical Laboratory Improvement Acts of 1988 (CLIA ’88) subjects all clinical laboratory testing to federal regulation and inspection. According to CLIA, test procedures are grouped into several categories. Waived tests are simple procedures with little chance of negative outcomes if performed inaccurately. PTS: 1

DIF:

Cognitive Level: 2

REF: p. 215

3. Human chorionic gonadotropin (-hCG) found in a woman’s urine specimen is secreted by a

fertilized egg, a. 24 b. 24-48 c. 48-72 d. more than 72 ANS: C

hours after implantation.


For the first 6 to 8 weeks after conception, -hCG helps maintain the corpus luteum and stimulate the production of progesterone. In a normal pregnancy, detectable amounts of about 25 mIU/mL -hCG are secreted 2 to 3 days (48-72 hours) after implantation, or approximately 8 to 10 days after conception or fertilization. Peak levels are reached approximately 2 to 3 months after the last menstrual period. PTS: 1

DIF:

Cognitive Level: 1

REF: p. 216

4. The specimen of choice for -hCG testing is: a. serum. b. urine. c. cerebrospinal fluid. d. saliva. ANS: A

Most of the kits currently can be used for both serum and urine -hCG but show better sensitivity with serum, because the concentration of -hCG in serum is not subject to the wide variation found in urine -hCG as a result of changes in urine concentration. Urine for the hCG assay (pregnancy test) must be collected at a suitable time after fertilization to allow the concentration of the hormone to rise to a significant detectable level. PTS: 1

DIF:

Cognitive Level: 2

REF: p. 216

5. If urine is used for pregnancy testing, it should be: a. first morning. b. random. c. 24-hour. d. sterile. ANS: A

The first morning urine specimen is required because it contains the highest concentration of hormone. Collect the urine specimen in a clean glass or plastic container. PTS: 1

DIF:

Cognitive Level: 1

REF: p. 217

6. Fecal occult blood testing tests for: a. whole blood in the stool. b. hemoglobin in fecal specimens. c. presence of inflammation causing bleeding. d. presence of infection. ANS: A

Tests for hemoglobin in fecal specimens are often referred to as tests for occult blood. PTS: 1

DIF:

Cognitive Level: 1

REF: p. 218

7. The value of the fecal occult blood test is that it could determine the cause of or aid in

detecting: a. hypochromic anemia. b. ulcerative disease. c. neoplastic diseases.


d. all of the above. ANS: D

Detection of occult blood in the feces is important in determining the cause of hypochromic anemias resulting from chronic loss of blood and in detecting ulcerative or neoplastic diseases of the gastrointestinal system. Blood in the feces may result from bleeding anywhere along the gastrointestinal tract, from the mouth to the anus. PTS: 1

DIF:

Cognitive Level: 2

REF: p. 219

8. False-positive fecal occult blood results can be caused by ingestion of: a. red meat. b. horseradish. c. apples and oranges. d. both a and b. ANS: D

Several interfering substances may give false-positive results for occult blood. These include those of dietary origin with peroxidase activity, especially myoglobin and hemoglobin in red meat. Vegetable peroxidase, as found in horseradish, can also cause false-positive results. Several foods have been identified as causing erroneous reactions. These include turnips, broccoli, bananas, black grapes, pears, plums, and melons. Cooking generally destroys these peroxidases, and therefore patients are generally instructed to eat only cooked foods. PTS: 1

DIF:

Cognitive Level: 2

REF: p. 219

9. A false-negative fecal occult blood can be caused by: a. beets and carrots. b. chicken. c. multivitamins. d. vitamin C. ANS: D

The ingestion of more than 250 mg/day of vitamin C is to be avoided, because it may cause false-negative results. PTS: 1

DIF:

Cognitive Level: 2

REF: p. 219

10. The American Cancer Society recommends that

fecal specimen(s) from consecutive specimen(s) be collected for colorectal screening. a. 1; 1 b. 2; 3 c. 3; 3 d. 4; 4 ANS: B

The American Cancer Society recommends that two samples from three consecutive specimens be collected for colorectal screening. Test kits are usually supplied to patients in groups of three slides. PTS: 1

DIF:

Cognitive Level: 2

REF: p. 220


11. LIS can be applied to: a. specimen processing. b. inventory control. c. data entry on patients’ charts. d. all of the above. ANS: D

Computer technology is not limited to just testing; it is applied to many laboratory-related preanalytical and postanalytical functions (e.g., specimen processing, inventory control, quality control, online monitoring, data entry on patients’ charts, and data interpretation). PTS: 1

DIF:

Cognitive Level: 2

REF: p. 222

12. Computer hardware for an LIS system includes: a. central processing unit (CPU). b. random access memory (RAM). c. an operating system. d. both a and b. ANS: D

A computer system consists of both hardware and software components. Hardware is the physical or “hard” parts of the computer, and software programs contain the instructions that direct computer processes or functions. Computer hardware for every LIS server can be divided into: 1. central processing unit (CPU) 2. random access memory (RAM) 3. peripheral devices external to the CPU PTS: 1

DIF:

Cognitive Level: 2

REF: p. 223

13. Computer interfaces are important to the laboratory for all of the following reasons except: a. transmission of tests results is faster. b. accuracy of analytical testing is improved. c. transposition of numbers errors are eliminated. d. charting of patient results can be direct. ANS: B

Computer interfaces do not contribute to the accuracy of actual laboratory testing, but interfaces are important to the laboratory because they contribute to the overall effectiveness of the computer support of laboratory operations. It is critical to remember that interfaces pass patient information between computers without direct human intervention. Laboratory time has been saved by the interfacing of the laboratory computer with the analytic testing instrument so that the test result can be entered directly into the computer information system or LIS and then become part of a patient’s chart. PTS: 1

DIF:

Cognitive Level: 2

14. An advantage of a 2D bar code is that it is: a. omnidirectionally scannable. b. readable upside down. c. readable backward, forward, or diagonally.

REF: pp. 225-226


d. all of the above. ANS: D

2D symbols are omnidirectionally scannable—upside down, backward, forward, and even diagonally. PTS: 1

DIF:

Cognitive Level: 2

REF: p. 224

15. General functions of an LIS can include: a. bar code reading of specimens. b. selection of tests from a test menu. c. reporting of test results. d. all of the above. ANS: D

The functions of the LIS can be grouped into three categories: preanalytical, analytical, and postanalytical. Examples of LIS functions are as follows: • Preanalytical: Test ordering, specimen labels, specimen accessioning, specimen tracking • Analytical: Automated results entry, manual results entry, quality control, validation of results, network to laboratory automation systems • Postanalytical: Patient cumulative reports, workload recording, billing, network to other systems PTS: 1

DIF:

Cognitive Level: 1

REF: pp. 226-228

16. Handheld automated equipment has been developed for many applications, including blood

coagulation and blood gases. Other measurements include: a. electrolytes b. urea c. glucose d. all of the above ANS: D

Handheld automated equipment has been developed for many applications. Measurements can include electrolytes, prothrombin time, partial thromboplastin time, activated clotting time, hematocrit, arterial blood gases, urea, and glucose. PTS: 1

DIF:

Cognitive Level: 1

REF: p. 221

17. The major benefits of laboratory automation include all of the following except: a. reduction of medical errors. b. improved safety for laboratory staff. c. longer turnaround time of results. d. more reliable results. ANS: C

Patient safety benefits from automation include reduction of medical errors; workflow standardization for more precise, consistently reliable test results; and improved test turnaround time for faster diagnosis and better patient care. Automated specimen processing and testing also improve safety for laboratory technologists. PTS: 1

DIF:

Cognitive Level: 2

REF: p. 230


18. Which of the following is the sequence of major steps in automated analysis that mimic

manual techniques? a. Specimen collection and processing, specimen and reagent measurement and delivery, chemical reaction phase, measurement phase, and signal processing and data handling b. Specimen and reagent measurement and delivery, specimen collection and processing, chemical reaction phase, measurement phase, and signal processing and data handling c. Specimen collection and processing, specimen and reagent measurement and delivery, signal processing and data handling, chemical reaction phase, and measurement phase d. Signal processing and data handling, specimen collection and processing, specimen and reagent measurement and delivery, chemical reaction phase, and measurement phase ANS: A

The major steps designed by manufacturers to mimic manual techniques are: 1. Specimen collection and processing 2. Specimen and reagent measurement and delivery 3. Chemical reaction phase 4. Measurement phase 5. Signal processing and data handling PTS: 1

DIF:

Cognitive Level: 1

REF: p. 230

19. The three most highly autoT mEaS teT dB arA eaNsKoS fE thL eL clE inR ic. alClO abMoratory are: a. microbiology, urinalysis, and hematology. b. clinical chemistry, hematology, and urinalysis. c. clinical chemistry, microbiology, and blood banking. d. immunology, microbiology, and hematology. ANS: B

Automated instruments have been designed to perform the most frequently ordered tests because it is known that six tests make up 50% of the workload of the average chemistry laboratory. The three most highly automated or semiautomated clinical laboratory specialties are clinical chemistry, hematology, and more recently, urinalysis. Automated or semiautomated testing is less frequently used in blood banking and microbiology, although automated systems for testing and screening donated blood and identification of biochemical reactions of bacteria are becoming more common. PTS: 1 20.

DIF:

Cognitive Level: 1

Leukocytes can be differentiated by: a. automated three-part differential. b. automated five-cell differential. c. digital imaging. d. all of the above. ANS: D

REF: p. 232


Automated three-part differentials consist of a size-distributed histogram of white blood cells. In most of these instruments, three subpopulations of white cells are counted: lymphocytes, other mononuclear cells, and granulocytes. A computer calculates the number of particles in each area as a percentage of the total white blood cell count histogram. Many instruments provide five-cell differentials. New cell identification systems use digital imaging to identify cells. PTS: 1

DIF:

Cognitive Level: 1

REF: p. 235

21. Systems for automated or semiautomated urinalysis chemical testing systems use: a. visual reading. b. reflectance photometers. c. flow-cell analysis. d. any of the above. ANS: B

Reflectance photometers are used for reading their respective reagent strips in automated and semiautomated strips. In a fully automated system, a reflectance spectrophotometric is also used. PTS: 1

DIF:

Cognitive Level: 1

REF: p. 235

22. The amplification of low levels of specific DNA sequences to higher quantities suitable for

further analysis is accomplished by: a. polymerase chain reaction (PCR). b. Southern blot. c. Western blot. d. microarrays. ANS: A

Polymerase chain reaction (PCR) is an in vitro method that amplifies low levels of specific DNA sequences in a sample to higher quantities suitable for further analysis. PTS: 1

DIF:

Cognitive Level: 2

REF: p. 235

MATCHING

Match the following categories of POCT with the appropriate characteristic. a. slide examinations of freshly collected body fluids b. simple procedures with little chance of negative outcomes if performed inaccurately c. usually nonautomated, requiring considerable judgment d. more complex than waived, usually automated 1. 2. 3. 4.

waived moderately complex highly complex provider-performed microscopy (PPM)

1. ANS: B REF: p. 215

PTS: 1

DIF:

Cognitive Level: 2


2. ANS: D REF: p. 215 3. ANS: C REF: p. 215 4. ANS: A REF: p. 215

PTS: 1

DIF:

Cognitive Level: 2

PTS: 1

DIF:

Cognitive Level: 2

PTS: 1

DIF:

Cognitive Level: 2


Chapter 10: Introduction to Clinical Chemistry Turgeon: Linné & Ringsrud’s Clinical Laboratory Science, 7th Edition MULTIPLE CHOICE 1. Which of the following hormones lowers the blood glucose level by promoting glucose entry

into cells? a. Cortisol b. Glucagon c. Insulin d. Thyroxine ANS: C

Insulin, secreted by the pancreas following a meal, facilitates entry of the energy-containing sugar, glucose, into body cells. Glucagon, also secreted by the pancreas, opposes the action of insulin, to maintain the blood sugar in the normal range when fasting between meals. The remaining two hormones, cortisol and thyroxine, also serve to increase blood sugar levels by promoting gluconeogenesis, the formation of glucose from nonglucose sources. PTS: 1

DIF:

Cognitive Level: 1

REF: p. 247

2. Contributory factors to which form of diabetes include a genetic predisposition and

autoimmune destruction of pancreatic  cells? a. Type 1 b. Type 2 c. Gestational diabetes mT elE litSuT s BANKSELLER.COM d. Stress-induced hyperglycemia ANS: A

Type 1 diabetes is typically diagnosed in children of families with a genetic predisposition to the disorder. This form of diabetes is characterized by an absolute lack of insulin, a consequence of autoimmune mechanisms that destroys the  cells of the pancreas. In type 2 diabetes, genetic factors also play a role. Individuals with type 2 diabetes characteristically develop insulin resistance later in life, which is associated with an unhealthy diet and sedentary lifestyle. Gestational diabetes is routinely screened for during pregnancy. If diagnosed, dietary changes can be made to reduce complications experienced with childbirth. Stress-induced hyperglycemia occurs in both diabetic and nondiabetic individuals. Treatment of critically ill patients with this condition involves maintaining the blood sugar at normal or near normal levels, reducing morbidity and mortality rates for this group of individuals. PTS: 1

DIF:

Cognitive Level: 2

3. Hypoglycemia is not a consequence of: a. insulinoma. b. islet cell hyperplasia. c. insulin resistance. d. glycogen storage disease. ANS: C

REF: p. 248


The term insulin resistance refers to the ineffective utilization of insulin by insulin-dependent cells of the body. This is the situation encountered in type 2 diabetes. As a result, glucose is less effectively transported into the cell and the patient is hyperglycemic rather than hypoglycemic. Both insulinoma and islet cell hyperplasia are forms of inappropriate hyperactivity of pancreatic cells, leading to excessive insulin production, thereby lowering the blood sugar level. Lowering of blood sugar also occurs in glycogen storage disease. In the latter case, glycogen, a reserve form of glucose, cannot be broken down and released into the bloodstream when needed. PTS: 1

DIF:

Cognitive Level: 2

REF: p. 251

4. The anticoagulant of choice for glucose testing when analysis is not performed promptly is: a. EDTA. b. sodium fluoride. c. lithium heparin. d. sodium citrate. ANS: B

Sodium fluoride prevents glycolysis, or the breakdown of glucose, in the blood sample. Without this anticoagulant, if not tested promptly, the blood glucose will decrease with time, resulting in a falsely low glucose level. The other anticoagulants listed do not prevent glycolysis and are unacceptable. Glycolysis will also continue to occur in samples collected with no anticoagulant, if the serum is not separated from the cells following phlebotomy. PTS: 1

DIF:

Cognitive Level: 1

REF: pp. 251-252

5. A major benefit of point-of-care testing (POCT) for blood glucose is that it: TtE a. allows for immediate in erS veTnB tiA onN. KSELLER.COM b. does not require oversight by the laboratory. c. can only be performed by laboratory personnel. d. eliminates the need for glucose testing in the laboratory. ANS: A

POCT allows for immediate action to be taken to normalize the blood sugar level in patients with diabetes. POCT, performed routinely, improves prognosis. POCT performed in a health care setting is under the jurisdiction of the laboratory. Other members of the health care team (e.g., nurses) must be trained before they can perform POCT. Routine screening of individuals for diabetes is most easily performed as part of a battery of tests, including glucose, in the laboratory. PTS: 1

DIF:

Cognitive Level: 2

REF: p. 252

6. Which of the following methods is not used to quantitate glucose levels? a. Glucose oxidase b. Glucose dehydrogenase c. Clinitest d. Hexokinase ANS: C


Clinitest is a manual method used to detect reducing substances, including glucose, in the urine. It is important to note that this test may be positive in some instances, because of the presence of compounds other than glucose. Clinitest is a semiquantitative test, which estimates the relative quantity of reducing substance present. The remaining answer choices are all enzymes, employed in methods to quantitate the glucose level in whole blood, as performed on POCT instruments, or in the laboratory on automated analyzers. PTS: 1

DIF:

Cognitive Level: 1

7. The adult reference range for fasting serum glucose is a. 40-75 b. 55-105 c. 60-95 d. 70-110

REF: pp. 253-254

mg/dL.

ANS: D

Following a strict fast of 8 to 12 hours, the fasting plasma glucose should be 70-110 mg/dL. A result on the high side of the reference range should be monitored and additionally testing such as hemoglobin A1C should be tested. The glucose range in choice C (60-95 mg/dL) is the corresponding level in a whole blood sample. The lower range accounts for the dilution factor resulting from the presence of cellular elements, primarily red blood cells. A blood glucose level below 60 mg/dL is termed hypoglycemia. PTS: 1

DIF:

Cognitive Level: 1

REF: p. 254

8. Estimation of the blood glucose level over the previous 4 to 6 weeks is determined with which

test? a. HbA1c b. Ketones c. Clinitest d. Microalbumin ANS: A

HbA1c (glycosylated hemoglobin) is hemoglobin bound to glucose, present in red blood cells. Because red blood cells have a 120-day life span in the circulation, determination of HbA1c serves as a measure of the average blood glucose concentration over an extended time period. Ketones are present in the blood (ketonemia) during diabetic ketoacidosis and during periods of starvation when the body’s fat stores are utilized for energy. Clinitest is a screening test once used to detect glucose overflow into the urine when the renal threshold for glucose was exceeded. Today, direct testing of blood glucose is the preferred method. Microalbumin is a test performed on urine, invaluable in monitoring renal function in people with diabetes. Microalbuminuria is an indication of the onset of nephropathy, a common complication of uncontrolled diabetes. PTS: 1

DIF:

Cognitive Level: 1

REF: p. 251

9. Osmolality measures: a. the number of sodium ions in plasma. b. the number of sodium ions in urine or plasma. c. total concentration of all ions and molecules in serum or urine. d. total concentration of all ions and molecules in plasma.


ANS: D

Osmolality is based on the number of dissolved particles in a solution. Osmolality measures the total concentration of all of the ions and molecules present in serum or urine. Sodium, glucose, and urea are major contributors to the total osmolality of serum. PTS: 1

DIF:

Cognitive Level: 2

REF: p. 255

10. The term for the condition associated with elevated serum potassium is: a. hyponatremia. b. hypernatremia. c. hypokalemia. d. hyperkalemia. ANS: D

The prefix hyper refers to an increased amount of a particular substance. “Kal” signifies potassium, or K+, and the suffix -emia means condition. Conversely, hypokalemia is lower than normal serum potassium (the prefix hypo means lack or deficiency). The first two choices describe the conditions of too little and increased sodium, or Na+, in the blood, respectively. PTS: 1

DIF:

Cognitive Level: 1

REF: p. 256

11. The term chloride shift refers to the: a. exchange of chloride with sodium to maintain electrical neutrality. b. exchange of chloride with bicarbonate between red blood cells and plasma. c. contribution of chloride to serum osmolality. d. kidney’s excretion of cT hlEoS ridTeBbAyNthKeSreEnLalLtE ubRu. leC s.OM ANS: B

Chloride and bicarbonate are exchanged at the red blood cell membrane to maintain both electrical neutrality and acid-base balance. Both chloride and bicarbonate are negatively charged. Electrical neutrality would become increasingly out of balance if chloride were to be exchanged with sodium. It is true that chloride is a major contributor to serum osmolality and that the kidneys may either excrete or retain chloride depending on the body’s needs; however, these processes are not what is referred to as the chloride shift. PTS: 1

DIF:

Cognitive Level: 2

12. An increased anion gap may be due to: a. lactic acidosis. b. hypercalcemia. c. hypermagnesemia. d. hypoproteinemia. ANS: A

REF: p. 256


The anion gap is increased when negatively charged anions, other than chloride and bicarbonate, are increased in the plasma. These anions include ketones, present in diabetic ketoacidosis; lactic acid, an increase indicating inadequate tissue oxygenation; and alcohols, including ethanol and methanol. An increased anion gap is therefore suggestive of metabolic disorders or alcohol intoxication. Calcium and magnesium are cations. Elevated levels of calcium and magnesium—hypercalcemia and hypermagnesemia, respectively—will decrease the anion gap. The anion gap will also be decreased in hypoproteinemia because less protein is available to bind plasma cations. PTS: 1

DIF:

Cognitive Level: 2

REF: p. 257

13. Determination of plasma sodium with an ion-selective electrode (ISE) will be altered if the

specimen is lipemic (increased plasma lipids) in which manner? a. Increased with a direct ISE method b. Increased with an indirect ISE method c. Decreased with a direct ISE method d. Decreased with an indirect ISE method ANS: D

Direct ISE measurements are those made without sample dilution. Increased levels of lipids do not alter the result with this type of measurement. With indirect ISE measurements the sample is prediluted. Because lipids occupy plasma volume, dilution of the specimen will falsely decrease the concentration of sodium detected by the electrode. PTS: 1

DIF:

Cognitive Level: 1

REF: p. 258

14. The primary form of nonprotein nitrogen in the blood is: a. b. c. d.

urea. creatinine. cystatin C. creatine.

ANS: A

Urea or blood urea nitrogen (BUN) is the major nonprotein, nitrogen-containing compound that is present in the blood. Urea is the form in which the kidneys excrete excess body nitrogen, preventing the buildup in the blood plasma of ammonia, the primary metabolite of protein breakdown. The remaining compounds also contain nitrogen. Both creatine and creatinine are integral to metabolic reactions in muscle tissue, while cystatin C, a byproduct of cellular metabolism, may be used to estimate the glomerular filtration rate (GFR). PTS: 1

DIF:

Cognitive Level: 1

REF: p. 260

15. An adult male has a serum BUN of 56 mg/dL and a creatinine of 0.8 mg/dL. These results are

consistent with: a. decreased GFR. b. prerenal azotemia. c. renal azotemia. d. postrenal azotemia. ANS: B


This patient has an elevated BUN and normal creatinine. The BUN-to-creatinine ratio is greater than 20. Because the creatinine is normal, this is a case of prerenal azotemia. Prerenal azotemia is due to poor perfusion of the kidneys or an increased rate of protein breakdown. Postrenal azotemia is also indicated by an elevated BUN to creatinine ratio; however, creatinine is increased in this case. In renal disease, elevations in both BUN and creatinine characteristically result in a ratio between 12 and 20. A ratio of less than 12 is present in acute tubular necrosis. The GFR is used to assess renal function and is most frequently predicted using the serum creatinine. A decreased GFR is an indicator of renal insufficiency. PTS: 1

DIF:

Cognitive Level: 3

REF: p. 261

16. Which of the following conditions is least likely to contribute to an increased serum uric acid

level? a. Chemotherapy b. Renal disease c. Liver disease d. Gout ANS: C

Uric acid is a nitrogen-containing product of purine catabolism. Purines are nucleic acids, components of DNA, RNA, cofactors, and energy-containing molecules. Plasma increases in uric acid are therefore seen during chemotherapy when there is increased cell destruction, in a disorder of purine metabolism referred to as gout, and in renal disease when the clearance of uric acid is diminished. Another nitrogen-containing compound, ammonia, is a byproduct of protein catabolism, which occurs primarily in the liver. Liver disease is therefore associated with an increased plasma ammonia level. PTS: 1

DIF:

TCEogSnTitiBveALNeKveSl:E2LLER.COM

REF: p. 264

17. The first step in the laboratory determination of serum cholesterol is to ensure that all

cholesterol present is in the free form. This is accomplished with the use of: a. cholesterol esterase. b. glycerol. c. cholesterol oxidase. d. lipase. ANS: A

The enzyme cholesterol esterase converts cholesterol esters present in the sample to free cholesterol. Cholesterol oxidase is used in the second step of the determination of serum cholesterol. The oxidase catalyzes the formation of hydrogen peroxide. Hydrogen peroxide then reacts with a chromogen, allowing for spectrophotometric detection. Lipase is an enzyme employed in the first step of triglyceride quantitation. Glycerol is not an enzyme but a product of triglyceride hydrolysis. PTS: 1

DIF:

Cognitive Level: 1

REF: p. 265

18. Fasting lipid profile results for a 48-year-old male with diabetes are as follows: total

cholesterol: 298 mg/dL, HDL cholesterol: 26 mg/dL, and triglycerides: 320 mg/dL. The patient has no evidence of coronary artery disease (CAD). What is the recommended course of action? a. The LDL is normal, and no action is indicated.


b. The LDL is elevated, and the therapeutic goal is to decrease it to less than 100

mg/dL. c. The LDL is elevated, and the therapeutic goal is to decrease it to less than 130

mg/dL. d. The LDL is elevated, and the therapeutic goal is to decrease it to less than 160 mg/dL. ANS: C

For patients without clinical evidence of coronary or other atherosclerotic vascular disease, the National Cholesterol Education Program (NCEP) recommends health screening, including measurement of total cholesterol (TC) and HDL cholesterol, at least once every 5 years. Further evaluation is performed for those patients with a high TC, low HDL cholesterol (<35 mg/dL), or borderline TC who have at least two CAD risk factors (age >45 for men, age >55 for women or postmenopausal state without estrogen replacement, high blood pressure, smoking, diabetes, HDL <35 mg/dL, or family history of CAD before age 55 in male first-degree relative or before age 65 in female first-degree relative). The NCEP recommends that treatment decisions be based on the calculated level of LDL. For patients with an elevated LDL (160 mg/dL) who have fewer than two risk factors in addition to elevated LDL and who do not have clinical evidence of atherosclerotic disease, the goal of treatment is an LDL level less than 160 mg/dL. For those who have at least two other risk factors, the goal of treatment is an LDL level less than 130 mg/dL. When LDL levels remain higher than 160 mg/dL despite dietary measures, and the patient has two or more risk factors (in addition to high LDL), or when LDL levels remain higher than 190 mg/dL even without added risk factors, the addition of drug treatment should be considered. For those with CAD, peripheral vascular disease, or cerebrovascular disease, the goal of treatment is an LDL less than 100 mg/dL. All patients with clinicalTeE viS deTnBceAoNfKcS orEoL naLryEoRr.oC thOerMatherosclerotic disease should be evaluated with a fasting blood sample for measurement of TC, triglyceride, and HDL. LDL is calculated. PTS: 1

DIF:

Cognitive Level: 3

REF: p. 266

19. Laboratory diagnosis of an acute myocardial infarction (MI) is most accurately determined

with which of the following two cardiac markers? a. CK-MB and homocysteine b. CRP (C-reactive protein) and cardiac troponin c. Myoglobin and lactate dehydrogenase (LDH) d. Myoglobin and cardiac troponin ANS: D

The combination of myoglobin and cardiac troponins is the best choice of markers for acute MI. With these two markers, elevations (supporting diagnosis of an MI), are seen over a 2- to 10-day window of time. Homocysteine and CRP are both compounds determined to be risk factors, predicting increased likelihood of acute MI. Both CK-MB and LDH are enzymes, once utilized as markers of an acute coronary event. Today, the increased specificity of troponins, combined with early detection of the muscle protein myoglobin, is the preferred combination of cardiac markers. PTS: 1

DIF:

Cognitive Level: 1

REF: p. 268


20. Which of the following laboratory tests assist in the diagnosis of heart failure? a. BNP b. Fibrinogen c. CRP d. D-dimer ANS: A

BNP, or B-type natriuretic peptide, is a powerful regulator of fluid volume released primarily by cardiac cells in response to ventricular volume increase and pressure overload. Increased levels of BNP in the blood are diagnostic of congestive heart failure. Fibrinogen and CRP are acute-phase proteins, which serve as markers of inflammation and therefore cardiovascular risk. D-dimer, associated with thrombus formation and dissolution, is a nonspecific marker of active plaque formation in atherosclerotic heart disease. PTS: 1

DIF:

Cognitive Level: 1

21. In bilirubin assays, conjugated bilirubin is referred to as a. unconjugated b. total c. indirect d. direct

REF: p. 269

reacting bilirubin.

ANS: D

Conjugated bilirubin is the water-soluble product of heme degradation formed in the liver. This form of bilirubin is also referred to as direct or direct-reacting bilirubin because it reacts with diazo reagent in the bilirubin procedure. Indirect bilirubin, or bilirubin in its unconjugated form, is detected by a similar procedure; however, a compound such as methanol is added which aT ccEeS leT raB teA sN thK eS reEacLtL ioE nR of.iC ndOirMect bilirubin to result in a measure of total (both direct and indirect) bilirubin. Consequently, the terms direct and conjugated are synonymous, as are the terms indirect and unconjugated when referring to the two forms of bilirubin. PTS: 1

DIF:

Cognitive Level: 2

REF: p. 271

22. Which of the following enzymes is normally found in highest levels in hepatocytes, or liver

cells? a. ALT b. AST c. GGT d. Alkaline phosphatase ANS: A

ALT (alanine aminotransferase) is found in the greatest concentration in liver cells. In various forms of liver disease, including hepatitis and acute liver necrosis, the serum level of ALT is significantly increased because of liver damage. Although AST (aspartate aminotransferase) and alkamine phosphatase may be elevated in liver disease, they are not specific to the liver. For instance, AST is elevated in instances of heart, kidney, and muscle damage, whereas alkaline phosphatase is also utilized for diagnosis of bone, intestinal, and placental disorders. Serum elevation of GGT (gamma glutamyltransferase) is an early indicator of liver disease, a marker of chronic alcoholism, and may also be an indicator of renal damage.


PTS: 1

DIF:

Cognitive Level: 2

REF: p. 272

23. A reversal of the normal serum albumin/globulin (A/G) ratio occurs in a. heart and lung b. kidney and bone c. heart and liver d. kidney and liver

_ disease.

ANS: D

Kidney and liver disease are the two major causes of decreased serum albumin ratios. The liver is the site of albumin synthesis such that damage to this organ will decrease serum albumin, whereas kidney damage results in urinary loss of albumin. In both instances, when disease has progressed significantly, the ratio becomes less than the normal 2:1. The globulin component may be increased or decreased in various conditions, including chronic inflammation and monoclonal immunoglobulin disease. These conditions will also alter the A/G ratio. PTS: 1

DIF:

Cognitive Level: 2

REF: p. 272

24. The two enzymes used in the diagnosis of acute pancreatitis are: a. AST and ALT. b. amylase and lipase. c. alkaline phosphatase and amylase. d. GGT and lipase. ANS: B

Amylase and lipase determinations are central to the diagnosis of acute pancreatitis. Serum Mever, a concurrent increase in lipase amylase may also originateTfEroSmTB thA eN saKliS vaEryLgLlE anRd. s;ChO ow in conjunction with abdominal symptoms indicates a diagnosis of pancreatitis. AST and ALT are most commonly used to diagnose and follow the course of liver disease. Elevation of these two enzymes is a contraindication for the use of many drugs, such as statins, which may affect liver function. An elevation of GGT may be an indicator of early renal disease, chronic alcoholism, or renal damage. Alkaline phosphatase is an enzyme associated with disorders of the liver, bone, intestine, and placenta. PTS: 1

DIF:

Cognitive Level: 1

and thyroid stimulating hormone is 25. Free T4 is a. increased; decreased b. decreased; increased

REF: p. 274

in hypothyroidism.

ANS: B

Hypothyroidism: Thyroid Stimulating Hormone: Increased Free T4 (FT4): Decreased Hyperthyroidism: Thyroid Stimulating Hormone: Decreased Free T4 (FT4): Increased PTS: 1

DIF:

Cognitive Level: 2

REF: p. 275


26. Which of the following is the least utilized role of a tumor marker? a. Cancer screening b. Cancer diagnosis c. Monitoring of patients for a recurrence of cancer d. Determination of the extent of tumor burden ANS: B

Diagnosis of cancer is generally not made with the laboratory finding of an elevated tumor marker concentration. Tumor markers commonly tested for are associated with a relatively high rate of false-positive results compared with other diagnostic tests. Diagnosis is made in conjunction with other testing, such as a biopsy. The value of tumor markers is their use in screening for cancer, in specific populations, such as PSA (prostate specific antigen) in men older than age 50 years. Tumor markers are also frequently employed to monitor patients during therapy to determine its effectiveness, to detect recurrence following treatment, and to approximate the extent of tumor burden. PTS: 1

DIF:

Cognitive Level: 1

REF: p. 275

TRUE/FALSE 1. It is important to record the exact time and date in relation to the last dose of the drug when

drawing a blood specimen for therapeutic drug monitoring. ANS: T

It is important to record the exact time and date of the specimen in relation to the last dose of the drug, with a note of allToE thS erTdBruAgNs K prSeE scL riL beEdR . .COM PTS: 1

DIF:

Cognitive Level: 1

REF: p. 278

MATCHING

Match the following terms with the correct description. Use an answer only once. a. decreased plasma bicarbonate b. excess elimination of acid c. acid-rich fluids are lost d. decreased elimination of CO2 1. 2. 3. 4.

metabolic acidosis metabolic alkalosis respiratory acidosis respiratory alkalosis

1. ANS: A REF: p. 260 2. ANS: C REF: p. 260 3. ANS: D REF: p. 260 4. ANS: B

PTS: 1

DIF:

Cognitive Level: 2

PTS: 1

DIF:

Cognitive Level: 2

PTS: 1

DIF:

Cognitive Level: 2

PTS: 1

DIF:

Cognitive Level: 2


REF: p. 260

Match the name of the tumor specific marker assay with an appropriate application. a. suggests early recurrence of breast cancer b. secreted in endodermal sinus tumors c. detected in choriocarcinoma d. found in 20% of smokers 5. alpha fetoprotein 6. -hCG 7. CA 27.29 8. CEA 5. ANS: B REF: p. 276 6. ANS: D REF: p. 276 7. ANS: A REF: p. 276 8. ANS: D REF: p. 276

PTS: 1

DIF:

Cognitive Level: 2

PTS: 1

DIF:

Cognitive Level: 2

PTS: 1

DIF:

Cognitive Level: 2

PTS: 1

DIF:

Cognitive Level: 2


Chapter 11: Principles and Practice of Clinical Hematology Turgeon: Linné & Ringsrud’s Clinical Laboratory Science, 7th Edition MULTIPLE CHOICE 1. Less than a. 0.5 b. 1 c. 10 d. 50

% of the bone marrow consists of stem cells.

ANS: B

Blood, or the circulatory system, has several functions. It carries nutrients to the tissues. Oxygen, the most important nutrient, is carried by the red cells. Waste products, the end products of metabolism, are carried by the blood to the organs of excretion (e.g., kidneys). PTS: 1

DIF:

Cognitive Level: 1

2. The total volume of blood in an average adult is about a. 2 b. 4 c. 6 d. 8

REF: p. 293

liters.

ANS: C

The total volume of blood in an average adult is about 6 L, or 7% to 8% of the body weight. About 45% of this amountTisEcSoT mB poAsN edKoSfEreLdLbEloRo. dC ceOllMs (RBCs, erythrocytes), white blood cells (WBCs, leukocytes), and platelets (thrombocytes); the remaining 55% is the liquid fraction, plasma. PTS: 1

DIF:

Cognitive Level: 1

3. Blood cells are initially formed in the a. yolk sac b. liver c. spleen d. bone marrow

REF: p. 293

of the embryo.

ANS: A

In the embryo, self-renewing hematopoietic stem cells develop initially in the primitive yolk sac and then migrate to the fetal liver. Beginning in the fetal liver and later at about the fourth month of fetal life, the bone marrow begins functioning as a blood cell producer. PTS: 1

DIF:

Cognitive Level: 1

REF: p. 293

4. An immature RBC that has just lost its nucleus is referred to as a: a. prorubricyte. b. rubricyte. c. metarubricyte. d. reticulocyte.


ANS: D

The young RBC that has just extruded its nucleus is referred to as a reticulocyte. It is about the same size as or slightly larger than a mature RBC. Reticulocytes differ from mature RBCs morphologically because they contain a fine basophilic reticulum or network of RNA, a cytoplasmic remnant that decreases as the cell matures. When stained with Wright stain, reticulocytes appear pink-gray or pale purple—they have a slight bluish tinge. This polychromasia (many colors) represents the presence of RNA within the cell. With a special stain, such as brilliant cresyl blue or new methylene blue, the basophilic reticulum of RNA appears blue. Under normal conditions, reticulocytes remain and mature further in the bone marrow for a day or two before they are released into the peripheral blood. Normally about 1% of the circulating RBCs are reticulocytes. PTS: 1

DIF:

Cognitive Level: 2

REF: p. 294 | p. 296

5. Neutrophils, eosinophils, and monocytes act as: a. antibody producers. b. inactivators of foreign antigens. c. phagocytic scavengers. d. both a and b. ANS: C

Neutrophils, eosinophils, and monocytes act as phagocytic scavengers—they engulf and destroy invading microorganisms and clear the body of unwanted particulate material such as dead or injured tissue cells. The lymphocytes and plasma cells act as immunocytes, inactivating foreign antigens by antibody production and by delayed hypersensitivity reactions. PTS: 1

DIF:

TCEogSnTitiBveALNeKveSl:E2LLER.COM

REF: pp. 306-307

6. The most popular anticoagulant used to prevent clotting in order to perform a complete blood

count is: a. EDTA. b. heparin. c. sodium fluoride. d. sodium citrate. ANS: A

Three types of anticoagulants are commonly used in the hematology laboratory: EDTA, heparin, and sodium citrate. EDTA is the most commonly used anticoagulant in hematology for the complete blood cell count (CBC) or any of its component tests (hemoglobin, packed cell volume [microhematocrit], total leukocyte count and leukocyte differential count, and platelet count). PTS: 1

DIF:

Cognitive Level: 1

REF: p. 310

7. The most popular anticoagulant used to prevent clotting in order to perform a coagulation test

(e.g., prothrombin time) is: a. EDTA. b. heparin. c. sodium fluoride. d. sodium citrate.


ANS: D

Sodium citrate in the concentration of a 3.2% solution has been adopted as the appropriate concentration by the International Committee for Standardization in Hematology and the International Society for Thrombosis and Haemostasis. PTS: 1

DIF:

Cognitive Level: 1

REF: p. 310

8. The maximum allowable time for determination of a WBC count, microhematocrit, or platelet

count using blood that has been collected in EDTA is a. 4 b. 8 c. 24 d. 48

hours, if it is refrigerated at 4° C.

ANS: C

Laboratory tests should be done on fresh specimens whenever possible. WBC counts, microhematocrits, and platelet counts can be determined up to 24 hours after blood is collected in EDTA, if it is refrigerated at 4° C. PTS: 1

DIF:

Cognitive Level: 1

REF: p. 310

9. Blood samples that are unsuitable for hematology testing include a. hemolyzed b. clotted c. icteric d. both a and b

specimens.

ANS: D

In addition to specimens that have not been properly stored or promptly analyzed, two types of blood samples are unsuitable for hematology tests: clotted samples and samples that are hemolyzed during collection or handling. Clotted specimens are not suitable for cell counts because the blood cells are trapped in the clot and are therefore not counted. A cell count on a clotted sample will be falsely low. In hemolyzed specimens, the RBCs are no longer intact, and RBC counts on hemolyzed samples also give falsely low results. Although hemolyzed specimens are generally considered unacceptable for testing, it must be remembered that there are rare cases of intravascular hemolysis in which hemolysis is a clinically significant finding and not cause for rejection of the specimen for testing. PTS: 1

DIF:

Cognitive Level: 2

REF: p. 311

10. When the concentration of a salt solution is equal inside and outside of an RBC, it is: a. isotonic. b. hypotonic. c. hypertonic. d. equaltonic. ANS: A

When the concentration of a salt solution is the same in the diluent solution as it is inside the RBC, the diluent is called an isotonic solution. If the diluent is less concentrated than the inside of the RBC, the solution is called hypotonic. If the solution outside the RBC is more concentrated than that inside it, the outside solution is called hypertonic.


PTS: 1

DIF:

Cognitive Level: 1

REF: p. 311

11. Hemoglobin consists of: a. heme. b. iron-containing portion of the molecule. c. globin (protein-containing portion of the molecule). d. all of the above. ANS: D

Hemoglobin consists of the heme (iron-containing) portion of the molecule combined with the globin (protein-containing) portion. These two portions combine to form an activated form of hemoglobin that is ready to transport oxygen. Each hemoglobin molecule consists of four heme groups and a globin moiety, which is composed of four polypeptide chains. PTS: 1

DIF:

12. Oxyhemoglobin contains a. 1 b. 2 c. 4 d. 8

Cognitive Level: 1

REF: p. 297

molecule(s) per hemoglobin molecule.

ANS: C

The heme group is an iron complex containing one iron atom. Iron is essential for the primary function of the hemoglobin molecule—that is, carrying oxygen to the tissues. When reduced hemoglobin is exposed to oxygen at increased pressure, oxygen is taken up at the iron atom until each molecule of hemToE glSoT biB nA haNsKbS ouEnLdLfoEuRr . oxCyO geMn molecules, one molecule at each iron atom. The molecule fully saturated with oxygen (four oxygen molecules per hemoglobin) is called oxyhemoglobin. Oxyhemoglobin carries oxygen from the lungs to the body tissues. Hemoglobin returning to the lungs with carbon dioxide from the tissues is known as reduced hemoglobin. PTS: 1 13.

DIF:

Cognitive Level: 2

REF: p. 297

The principal adult hemoglobin contains: a. two alpha and two beta globin chains. b. three alpha and one beta globin chain. c. two alpha and two gamma chains. d. either a or c. ANS: A

The principal adult hemoglobin, HbA, contains two alpha () and two beta () globin chains. In another form of adult hemoglobin (HbA2), the alpha chains are paired with two delta () polypeptide chains. The delta chains are related to beta chains. PTS: 1

DIF:

14. Hemoglobin F makes up a. 5 b. 10

Cognitive Level: 1

% of hemoglobin in an adult.

REF: p. 298


c. 25 d. 75 ANS: A

The combination of HbA and HbA2 should normally make up 95% of the hemoglobin in an adult, with HbF making up 5% or less. Hemoglobin F is the major form found during intrauterine life and at birth. PTS: 1

DIF:

Cognitive Level: 1

REF: p. 298

15. An example of a clinically important abnormal hemoglobin is hemoglobin: a. S. b. C. c. E. d. All of the above are correct. ANS: D

The four clinically important abnormal hemoglobins are HbS, HbC, HbD, and HbE. These are all hereditary, and the disorders affect the protein portion of the hemoglobin molecule, altering the structure of the polypeptide chain. These abnormal hemoglobins, as well as the normal ones, can be distinguished from one another by electrophoresis. PTS: 1

DIF:

Cognitive Level: 1

REF: p. 300

16. When insufficient amounts of oxygen are present in the circulating blood, a patient’s skin will

appear: a. bright cherry red. b. pale pink. c. blue. d. white. ANS: C

When sufficient quantities of these hemoglobin derivatives are present in the circulating blood, hypoxia (lack of oxygen), or cyanosis (bluish discoloration of the skin and mucous membranes) will be seen clinically. PTS: 1

DIF:

Cognitive Level: 1

REF: p. 300

17. Carboxyhemoglobin: a. cannot bind to oxygen. b. cannot carry oxygen to tissues. c. will result in carbon monoxide poisoning. d. All of the above are correct.. ANS: D

The hemoglobin molecule has a much greater affinity for carbon monoxide (CO) than for oxygen and will readily combine with CO if it is present even in low concentration. Carboxyhemoglobin (HbCO) cannot bind to and carry oxygen and will result in carbon monoxide poisoning, even at relatively low concentrations of carbon monoxide. PTS: 1

DIF:

Cognitive Level: 2

REF: p. 301


18. The reference range for hemoglobin in normal adult females is a. 15 to 20 b. 12 to 16 c. 9 to 14 d. 13 to 18

g/dL.

ANS: B

At birth, the hemoglobin concentration is normally 15 to 20 g/dL. It decreases to 9 to 14 g/dL by about 2 months of age. By 10 years of age, the normal hemoglobin will be 12 to 15 g/dL. Normal adult values range from 12 to 16 g/dL in women and 13 to 18 g/dL in men. There may be a slight decrease in the hemoglobin level after age 50. PTS: 1

DIF:

Cognitive Level: 1

REF: p. 297

19. Another term used for the RBC measurement, hematocrit, is: a. RBC count. b. packed cell volume. c. packed cell count. d. erythrocyte indices. ANS: B

The hematocrit (Hct) is a macroscopic observation by which the percentage volume of the packed RBCs is measured. Hematocrit is therefore also known as the packed cell volume, or PCV. More correctly, the test name is the PCV and the equipment is the hematocrit. PTS: 1

DIF:

Cognitive Level: 1

REF: p. 312

icSinTdB ivAidNuK alS s,EaLqL uiEckR. quCaO litMy control check on hemoglobin 20. In normochromic, normocy TtE results can be done by: a. remeasuring the hemoglobin. b. checking the hematocrit. c. comparing the hemoglobin and hematocrit results. d. none of the above. ANS: C

A quick quality control check on hemoglobin results (in g/dL) is done by comparing them with the hematocrit results (in % units), using the following formula: Hb  3 ± Hct 3 units This rule applies only when RBCs are of normal size (normocytic) and color (normochromic). PTS: 1

DIF:

Cognitive Level: 2

REF: p. 313

21. The reference range of the microhematocrit in adult males is: a. 32% to 36%. b. 36% to 45%. c. 41.5% to 50.4%. d. 45% to 55.5%. ANS: C

The average reference range for the microhematocrit of an adult male is 41.5% to 50.4%. This range may differ slightly depending on the instrument being used for measurement.


PTS: 1

DIF:

Cognitive Level: 2

REF: p. 297

22. A hematocrit determined by automated instrumentation is consistently

when done by the

spun microhematocrit method. a. lower b. higher c. the same as d. not comparable ANS: A

When hematocrits are determined by use of automated hematology analyzers, the hematocrit is determined indirectly by determining the average size of the RBC population (the mean corpuscular volume, or MCV) and multiplying it by the total RBC count. The hematocrit value as determined by automation is therefore consistently lower than that done by the spun microhematocrit method. PTS: 1

DIF:

Cognitive Level: 2

REF: p. 313

23. If an excess of anticoagulant, for example, EDTA is present in a specimen for

microhematocrit determinations, this error will cause a. a falsely lower b. no observable change in the c. a falsely higher d. no difference in the

result.

ANS: A

Unique to the microhematocrit is the error caused by excess EDTA (inadequate blood for the fixed amount of EDTA in T thE eS blT ooBdAcNoK lleScE tioLnLtE ubRe. ).CTO heMmicrohematocrit will be falsely low because of cell shrinkage. Thus, heparinized capillary tubes must not be used with anticoagulated blood samples. PTS: 1

DIF:

Cognitive Level: 2

REF: p. 310

24. A hematocrit and PCV are the same measurement. However, the hematocrit is expressed in

percentage (%) but the PCV is expressed as: a. g/dL. b. L/L. c. 1012/L. d. 109/L. ANS: B

In current laboratory practice, hematocrit values are generally expressed as a percentage. With the SI system, however, PCV is preferably expressed as a decimal; the units are undesignated, but units of liter per liter (L/L) are implied, so a 45% result is reported as 0.45. PTS: 1

DIF:

Cognitive Level: 1

25. The reference range for the MCV is: a. 65 to 85 pg. b. 65 to 86 fL. c. 80 to 96 g/dL.

REF: p. 320


d. 80 to 96 fL. ANS: D

The MCV in normal adults is between 80 and 96 fL. PTS: 1

DIF:

Cognitive Level: 1

REF: p. 320

26. The RBC distribution width is: a. measurement of the degree of anisocytosis present. b. measurement of the degree of RBC size variability. c. abbreviated as RDW. d. all of the above. ANS: D

Red cell distribution width (RDW) is a measurement of the degree of anisocytosis present, or the degree of RBC size variability in a blood sample. PTS: 1

DIF:

Cognitive Level: 1

REF: p. 321

27. The anticoagulant of choice for cell counts is: a. lithium. b. sodium citrate. c. EDTA. d. heparin. ANS: C

Free-flowing capillary blood obtained from a skin puncture or venous blood preserved with an anticoagulant may be used. The anticoagulant of choice is EDTA.

TESTBANKSELLER.COM

PTS: 1

DIF:

Cognitive Level: 1

REF: p. 312

28. The most important requirement for the diluting fluid used to determine a WBC count is the

ability to: a. lyse cells. b. allow cells of interest to fluoresce. c. maintain cellular integrity. d. do either a or b. ANS: A

In the methods for WBC (leukocyte) counts, the diluting fluid must meet a very different requirement than for RBCs. It must be able to destroy the more numerous RBCs so that the WBCs may be counted more readily. PTS: 1

DIF:

Cognitive Level: 1

29. Leukopenia is a(n): a. increased total white blood count. b. decreased total while blood count. c. increased number of segmented neutrophils. d. decreased number of lymphocytes. ANS: B

REF: p. 312


The normal white cell count varies from 4.4 to 11.3  109/L in adults. An increase in the white cell (leukocyte) count above the normal upper limit is termed leukocytosis. A decrease below the normal lower limit is termed leukopenia. Leukopenia may occur with certain viral infections, with typhoid fever and malaria, after radiation therapy, after the administration of certain drugs, and in pernicious anemia. PTS: 1

DIF:

Cognitive Level: 1

REF: p. 314

30. A total platelet count of 250  109/L is: a. below normal. b. within the reference range. c. above normal. d. normal for children but not for adults. ANS: B

For platelet counts (109/L): (range 150 to 450). PTS: 1

DIF:

Cognitive Level: 1

REF: p. 315

31. The criteria for a satisfactory blood smear include: a. no interruption of the smear with ridges, waves, or holes; progressively going from

a thick smear to a thinner smear. b. covering one quarter to half of the length of the slide; moving smoothly from a thin

to a thick smear. c. making the smear with a smooth action and having a good feathered edge with a

defined border on the end. d. both a and c. ANS: A

When a satisfactory blood smear is observed macroscopically, the body of the film should be smooth and not interrupted by ridges, waves, or holes. It should be thickest at the origin and gradually thin out, rather than having alternating thick and thin areas. A good blood film should cover half to three-fourths of the length of the slide. All of the initial drop of blood should be incorporated into the film, not just part of it. The thin end of the smear should have a good feather edge; that is, the film should fade away without a defined border on the end. In some institutions a fairly straight feather edge is sought, while others prefer a more tongue-like edge. A defined border at the end of a blood film indicates that most of the WBCs have piled up at the end. When this occurs, the heavier neutrophils accumulate at the end to a greater extent than the other types of WBCs, giving an incorrect distribution of the types of WBCs in the body of the smear. Platelets also tend to accumulate at the end of a smear, decreasing the number in the body of the smear. This will also result in inaccurate percentages for the cell types within the body of the film, as the relatively stickier neutrophils and platelets tend to concentrate in such tails. PTS: 1

DIF:

Cognitive Level: 2

REF: p. 347

32. A properly stained Wright-Giemsa stained blood smear has a _

with the naked eye. a. blue b. red c. pink

appearance when observed


d. yellow ANS: C

If the blood film has been stained properly, it will appear pink when observed with the naked eye. PTS: 1

DIF:

Cognitive Level: 1

REF: p. 303

33. Examination of a blood smear should begin using 10 objective where the RBCs are: a. spread far apart. b. just touching each other or barely overlapping. c. clumped together. d. it doesn’t matter. ANS: B

A blood smear is examined with the low-power (10) objective. With the low-power objective, an area of the film is found where the red cells are just touching and are not overlapping or piled on top of one another. PTS: 1

DIF:

Cognitive Level: 1

REF: p. 321

34. An initial evaluation of a blood smear should include): a. an evaluation of the overall quality of the blood film. b. an estimate of the red and white (leukocyte) blood cell counts. c. a scan of the blood film for abnormal cells and clumps of platelets. d. all of the above. ANS: D

Low-power examination (10 objective) should include a(n): • evaluation of the overall quality of the blood film • estimate of the red cell and white (leukocyte) blood cell counts • scan of the blood film for abnormal cells and clumps of platelets PTS: 1

DIF:

Cognitive Level: 1

REF: p. 322

35. When RBCs are examined morphologically, what characteristics should be observed? a. Variations in color b. Variations in size (anisocytosis) c. Variations in shape (poikilocytosis) d. All of the above ANS: D

When RBCs are examined morphologically, the following characteristics must be observed and noted: • Variations in color • Variations in size (anisocytosis) • Variations in shape (poikilocytosis) • Variations in structure and inclusions • Presence of artifacts and abnormal distribution patterns • Presence of nucleated red cells


PTS: 1

DIF:

Cognitive Level: 1

REF: p. 323

36. The identification of leukocytes is guided by: a. overall size of the cell. b. shape and size of the cellular nucleus. c. color of the cytoplasm. d. all of the above. ANS: D

The size of the cell, the shape and size of the nucleus, and the staining reactions of the nucleus and the cytoplasm aid in the identification of leukocytes. PTS: 1

DIF:

Cognitive Level: 2

REF: p. 329

37. The blood cell that produces immunoglobulins (antibodies) is the: a. lymphocyte. b. plasma cell. c. monocyte. d. neutrophil. ANS: B

In addition to the five types of white cells that normally appear in the peripheral blood, the plasma cell (plasmacyte) can occur in certain blood specimens. The plasma cell is thought to be a derivative of the B lymphocyte. Plasma cells function in the synthesis of immunoglobulins. PTS: 1

DIF:

Cognitive Level: 1

REF: p. 309

38. If a patient has a total leukocyte count of 20.0  109/L with 70% neutrophils and 30%

lymphocytes on the leukocyte differential smear, what is the absolute lymphocyte count? a. 3.6  109/L b. 6.0  109/L c. 6.6  109/L d. 14.0  109/L ANS: B

An absolute cell count by cell type is obtained by multiplying the relative number of WBCs (in decimal units) by the total white cell count per liter. For example, if a patient’s WBC count is 7.5  109/L and 68% neutrophils are identified in the leukocyte differential, then the relative neutrophil count is 68%. The absolute neutrophil count is: 0.68  7.5  109/L = 5.1  109 neutrophils/L. The absolute count should be reported to the nearest 0.1  109/L cells. Many laboratories report both the relative and absolute values for leukocyte differentials. PTS: 1

DIF:

Cognitive Level: 3

REF: p. 309

39. The stage of erythrocyte (RBC) development during which remnants of RNA can be seen is: a. polychromatophilic normoblast (rubricyte). b. orthochromic normoblast (metarubricyte). c. reticulocyte (diffusely basophilic erythrocyte). d. mature erythrocyte.


ANS: C

The stages of erythrocyte maturation from the youngest to the mature cell are: pronormoblast (rubriblast), basophilic normoblast (prorubricyte), polychromatophilic normoblast (rubricyte), orthochromic normoblast (metarubricyte), reticulocyte (diffusely basophilic erythrocyte), and mature erythrocyte. RBCs that have just extruded their nucleus and have small numbers of ribosomes and or cytoplasmic RNA are in the reticulocyte stage of development. PTS: 1

DIF:

Cognitive Level: 2

REF: p. 296

40. One of the most frequent conditions in which hypochromic, microcytic RBCs can be observed

is a. b. c. d.

anemia. iron deficiency pernicious sickle cell either iron deficiency or sickle cell

ANS: A

Iron deficiency is one of the most frequent conditions in which hypochromic, microcytic RBCs are seen. PTS: 1

DIF:

Cognitive Level: 2

REF: p. 323

41. Mature RBCs that are larger than normal are called: a. macrocytic. b. microcytic. c. ovalocytic. d. normocytic. ANS: A

Asynchronous maturation or dyssynchronous development are terms to describe when development of the cellular nucleus and cytoplasm does not progress at the same rate. For example, in pernicious anemia, cells of the megaloblastic sequence have an immature chromatin pattern in the nucleus. Ultimately, the out-of-sync development results in mature macrocytic RBCs. PTS: 1

DIF:

Cognitive Level: 2

REF: p. 323

42. RBCs that have a central area of hemoglobin are called: a. spherocytes. b. target cells. c. microcytes. d. discocytes. ANS: B

Target cells (codocytes) look like targets, showing a peripheral ring of hemoglobin, an area of pallor or clearing, and then a central area of hemoglobin. This cell circulates as a bell-shaped cell but takes on the target shape when dried on a slide for morphologic examination. Target cells represent another membrane defect; they have excessive cell membrane in relation to the amount of hemoglobin. PTS: 1

DIF: Cognitive Level: 1

REF: p. 326


43. Howell-Jolly bodies represent: a. residual DNA. b. residual RNA. c. toxic granules. d. either a or b. ANS: A

Howell-Jolly bodies are round, densely staining purple granules that stain like dense nuclear chromatin. Howell-Jolly bodies are remnants of the red cell nucleus and thus are DNA. Under normal conditions they are derived from nuclear fragmentation (karyorrhexis) or incomplete expulsion of the nucleus in the later stages of red cell maturation and are thought to be aberrant chromosomes in certain abnormal conditions. PTS: 1

DIF:

Cognitive Level: 2

REF: p. 327

44. The term shift to the left refers to a(n): a. increase in mature neutrophils. b. increase in immature neutrophils. c. increase in immature lymphocytes. d. decrease in mature neutrophils. ANS: B

The term shift to the left refers to an increase in immature neutrophils in the circulating blood. Neutrophils may leave the bone marrow prematurely in cases of acute infection or acute leukemias. PTS: 1

DIF:

Cognitive Level: 2

REF: p. 302

45. Observation of many smudge cells on a blood smear can be associated with a diagnosis of: a. chronic myelogenous leukemia. b. chronic lymphocytic leukemia. c. acute bacterial infection. d. acute viral infection. ANS: B

Smudge cells represent old or degenerating lymphocytes. If many smudge cells are observed, a diagnosis of chronic lymphocytic leukemia can be suggested. PTS: 1

DIF:

Cognitive Level: 2

REF: p. 334

46. Reticulocytes represent RBCs that: a. have lost their nuclei but not their cytoplasmic RNA. b. are immature, nucleated RBCs. c. are increased in untreated pernicious anemia. d. have abnormal staining characteristics. ANS: A

Reticulocytes are young RBCs that have matured enough to have lost their nuclei but not their cytoplasmic RNA. They do not have the full amount of hemoglobin. PTS: 1

DIF:

Cognitive Level: 2

REF: p. 296


47. The reference range for adult (male) reticulocytes is a. 0 to 0.2 b. 0.3 to 0.5 c. 0.5 to 1.5 d. 0.5 to 2.5

% (uncorrected).

ANS: D

In the circulating blood, 0.5% to 1.5% of the RBCs are usually reticulocytes. This is based on a normal RBC life span of 120 days and a normal RBC count, necessitating replacement of approximately 1% of the adult circulating RBCs each day. Female ranges are 0.9% to 1.9%; male ranges are 1.1% to 2.1%. PTS: 1

DIF:

Cognitive Level: 2

REF: p. 319

48. The erythrocyte sedimentation rate (ESR) is used: a. to track tissue healing after a myocardial infarction (heart attack). b. as a nonspecific index of inflammatory activity. c. to diagnose anemias. d. Both a and b are correct. ANS: D

The ESR is a nonspecific screening test for inflammatory activity. It is a measure of the presence and severity of pathologic processes. In addition, as patients recover from a myocardial infarction, the sedimentation rate slowly returns to normal. It may still be increased long after other clinical manifestations have disappeared, showing that the defense mechanisms of the body continue to be more active than normal. PTS: 1

DIF:

TCEogSnTitiBveALNeKveSl:E2LLER.COM

REF: p. 319

TRUE/FALSE 1. Mature platelets are brightly colored oval bodies on a peripheral blood smear. ANS: F

Mature platelets are small, colorless bodies, generally round or ovoid in shape. PTS: 1

DIF:

Cognitive Level: 2

REF: p. 309

2. Platelets maintain the structure or integrity of the endothelium by plugging any gaps in the

lining. ANS: T

Platelets act to maintain the structure of integrity of the endothelial cells lining the vascular system by plugging any gaps in the lining. PTS: 1

DIF:

Cognitive Level: 2

3. Platelets act as plugs around the opening of a wound. ANS: T

REF: p. 309


Platelets act as plugs around the opening of a wound and release certain factors necessary for the formation of a blood clot. PTS: 1

DIF:

Cognitive Level: 2

REF: pp. 309-310

4. Platelets release certain factors that are required for the formation of a blood clot. ANS: T

Platelets act as plugs around the opening of a wound and release certain factors necessary for the formation of a blood clot. PTS: 1

DIF:

Cognitive Level: 2

REF: p. 310

5. Heme iron is normally recycled for reuse. ANS: T

Heme is itself a complex iron-containing molecule. Normally heme is excreted from the body as bilirubin, which is eventually converted to the various bile salts and pigments. The iron is normally removed and retained by the mononuclear phagocytic system, stored, and reused in the production of new hemoglobin. PTS: 1

DIF:

Cognitive Level: 1

REF: p. 297

MATCHING

Match the RBC indices term with the appropriate equation. a. b. c. 1. MCV 2. mean corpuscular hemoglobin 3. mean corpuscular hemoglobin concentration 1. ANS: A REF: p. 320 2. ANS: B REF: p. 320 3. ANS: C REF: p. 320

PTS: 1

DIF:

Cognitive Level: 1

PTS: 1

DIF:

Cognitive Level: 1

PTS: 1

DIF:

Cognitive Level: 1

Match the errors in staining with the appropriate description. a. if the pH of the staining system is too alkaline, the parts of the cells taking up basic dye will be overstained, giving an overall blue effect, with very dark blue to black nuclear chromatin and bluish RBCs b. parts of the cell taking up acidic dye will be overstained and will appear too red, while the parts of the cells taking up basic dye will appear pale c. an excessively blue appearance on gross examination


d. improper washing (not washing enough to remove the metallic scum) 4. 5. 6. 7.

pH of the staining system is too acidic pH of the staining system is too alkaline overstaining, inadequate washing, or using too thick a film large amounts of precipitated stain

4. ANS: B REF: p. 352 5. ANS: A REF: p. 352 6. ANS: C REF: p. 352 7. ANS: D REF: p. 352

PTS: 1

DIF:

Cognitive Level: 2

PTS: 1

DIF:

Cognitive Level: 2

PTS: 1

DIF:

Cognitive Level: 2

PTS: 1

DIF:

Cognitive Level: 2

Match the following categories of microscopic observations of RBCs with the appropriate example. a. Howell-Jolly body b. schistocyte c. hypochromia d. macrocytes 8. 9. 10. 11.

variation in RBC color alteration in RBC size alteration in RBC shape an RBC inclusion

8. ANS: C REF: p. 323 9. ANS: D REF: p. 323 10. ANS: B REF: p. 325 11. ANS: A REF: p. 326

PTS: 1

DIF:

Cognitive Level: 1

PTS: 1

DIF:

Cognitive Level: 1

PTS: 1

DIF:

Cognitive Level: 1

PTS: 1

DIF:

Cognitive Level: 1

Match the cellular characteristic with the appropriate cell type. a. usually the most numerous of the mature leukocytes (WBCs) b. dark-purple or black granules in the cytoplasm c. orange granules in the cytoplasm d. cytoplasm usually stains a light blue e. the largest mature WBC 12. 13. 14. 15. 16.

neutrophil lymphocyte monocyte eosinophil basophil

12. ANS: A

PTS: 1

DIF:

Cognitive Level: 1


REF: p. 306 13. ANS: D REF: p. 308 14. ANS: E REF: p. 307 15. ANS: C REF: p. 303 16. ANS: B REF: p. 303

PTS: 1

DIF:

Cognitive Level: 1

PTS: 1

DIF:

Cognitive Level: 1

PTS: 1

DIF:

Cognitive Level: 1

PTS: 1

DIF:

Cognitive Level: 1

Match the adult mean reference value (%) with each of the leukocyte types. a. 0.5 b. 59 c. 3 d. 4-6 e. 34 17. 18. 19. 20. 21.

neutrophils lymphocytes monocytes eosinophils basophils

17. ANS: B REF: p. 309 18. ANS: E REF: p. 309 19. ANS: D REF: p. 309 20. ANS: C REF: p. 309 21. ANS: A REF: p. 309

PTS: 1

DIF: Cognitive Level: 1

PTS: 1

DIF: Cognitive Level: 1

TESTBANKSELLER.COM

PTS: 1

DIF: Cognitive Level: 1

PTS: 1

DIF: Cognitive Level: 1

PTS: 1

DIF: Cognitive Level: 1

Match the following terms with an appropriate definition. a. variation in cell size b. increased central pallor in RBCs c. faint blue-colored RBCs 22. hypochromia 23. polychromasia 24. anisocytosis 22. ANS: B REF: p. 323 23. ANS: C REF: p. 323 24. ANS: A REF: p. 323

PTS: 1

DIF:

Cognitive Level: 1

PTS: 1

DIF:

Cognitive Level: 1

PTS: 1

DIF:

Cognitive Level: 1

Match the following granulocyte alterations with an appropriate description.


a. composed of lysosomal materials and fused primary granules b. associated with acute bacterial infections c. pale blue inclusions associated with burns 25. toxic granulation 26. Döhle bodies 27. Auer rods 25. ANS: B REF: p. 332 26. ANS: C REF: p. 332 27. ANS: A REF: p. 333

PTS: 1

DIF:

Cognitive Level: 2

PTS: 1

DIF:

Cognitive Level: 2

PTS: 1

DIF:

Cognitive Level: 2


Chapter 12: Introduction to Hemostasis Turgeon: Linné & Ringsrud’s Clinical Laboratory Science, 7th Edition MULTIPLE CHOICE 1. The final result of primary hemostasis is: a. platelet plug. b. constriction of a blood vessel. c. a blood clot. d. promotion of bleeding to cleanse a wound. ANS: A

Primary hemostasis results in the formation of a platelet plug. The most immediate response of the body to bleeding is vasoconstriction. In this process, the damaged blood vessel constricts, decreasing the blood flow through the injured area. Platelet adhesion is essential to the formation of a platelet plug. Platelets must be available in adequate numbers and must be functioning normally for this to occur. PTS: 1

DIF:

Cognitive Level: 2

REF: p. 358

2. The final result of secondary hemostasis is: a. platelet plug. b. constriction of a blood vessel. c. a blood clot. d. promotion of bleeding to cleanse a wound. ANS: C

Secondary hemostasis results in the formation of a blood clot because the coagulation factors present in the blood interact, forming a fibrin network and a thrombus, to stop the bleeding completely. Slow lysis of the thrombus begins, and final repair to the site of the injury takes place. PTS: 1

DIF:

Cognitive Level: 2

REF: p. 358

3. An unbalanced hemostatic mechanism can produce: a. thrombosis. b. bleeding. c. inadequate number of platelets. d. both a and b. ANS: D

An unbalanced hemostatic system or mechanism produces bleeding or thrombosis. These conditions can result from a defect in the vascular system itself, inadequate numbers of platelets, an inadequate fibrin clotting mechanism, or faulty fibroblastic repair. PTS: 1

DIF:

Cognitive Level: 2

REF: p. 358

4. The protein “glue” that optimally binds platelets and collagen after vessel injury is: a. fibronectin. b. von Willebrand’s Factor (VIII:vWF).


c. factor II. d. factor I. ANS: B

Fibronectin is secreted by endothelial cells and platelets. It has been shown to assist in bonding platelets to the collagen substrate. But the additional protein factor, von Willebrand’s factor (VIII:vWF) acts as the glue necessary for optimal platelet-collagen binding to occur. Factors I and II are related to the coagulation cascade. PTS: 1

DIF:

Cognitive Level: 2

REF: p. 360

5. The formation of a blood clot relies on three major steps in which order? a. Formation of thromboplastin, formation of thrombin, and formation of fibrin b. Formation of thrombin, formation of thromboplastin, and formation of fibrin c. Formation of fibrin, formation of thrombin, and formation of thromboplastin d. Formation of fibrin, formation of thromboplastin, and formation of thrombin ANS: A

The mechanism of coagulation takes place in three major steps, with the formation of a fibrin thrombus being the major goal. The correct order is the formation of: 1. thromboplastin. 2. thrombin. 3. fibrin. PTS: 1

DIF:

Cognitive Level: 2

REF: p. 361

6. The final end product of fibrinolysis is: a. fibrinogen. b. fibrin. c. fibrin-split products. d. plasmin. ANS: C

As soon as the clotting process has begun, fibrinolysis is initiated to break down the fibrin clot that is formed. The active enzyme that is responsible for digesting fibrin or fibrinogen is plasmin. The decomposition products of fibrin and fibrinogen, called fibrin degradation products (FDPs) or fibrin split products (FSPs), are formed during fibrinolysis and are removed from the blood by the mononuclear phagocytic system. PTS: 1

DIF:

Cognitive Level: 2

REF: p. 367

7. Natural anticoagulants include: a. antithrombin III (AT-III). b. protein C. c. protein S. d. all of the above. ANS: D

To maintain a balance the body has a number of proteins that inhibit coagulation and help prevent unwanted thrombus formation. Natural in vivo anticoagulant systems are antithrombin III (AT-III, heparin cofactor II (HC-II) and protein C and its cofactor, protein S factor.


PTS: 1

DIF:

Cognitive Level: 1

REF: p. 367

8. The appropriate anticoagulant for prothrombin time assay is: a. EDTA. b. 3.2% sodium citrate. c. 3.8% sodium citrate. d. heparin. ANS: B

Screening coagulation assays are performed on plasma that has been processed from blood anticoagulated with a 3.2% sodium citrate solution. By a long-established convention for coagulation tests, nine volumes of carefully drawn blood are mixed with one volume of citrate anticoagulant (1:10 ratio). The standard ratio of anticoagulant to sample is 1:10 for persons with hematocrits between 20% and 60%. PTS: 1

DIF:

Cognitive Level: 1

9. A prothrombin time assay should be conducted within a. 2 b. 4 c. 8 d. 24

REF: p. 371

hours after specimen collection.

ANS: D

Prothrombin time assays should be done within 24 hours of blood collection, if the specimen remains capped. After that time, plasma must be frozen. Freezing should occur at –20°C for up to 2 weeks and –70°C for up to 6 months. PTS: 1

DIF:

Cognitive Level: 1

REF: p. 371

10. Most patients on warfarin therapy should maintain an international normalized ratio (INR) of

approximately a. 0.5-1.5 b. 1-2 c. 2-3 d. 3-4

times of a normal value.

ANS: C

The INR reference range is 0.9 to 1.13 in persons 1 year or older, and increasing INR values correspond to increased anticoagulation. Most patients on warfarin are required to maintain INR values between 2 and 3, or 2.5 and 3.5 for oral anticoagulant therapy because of a mechanical value. PTS: 1

DIF:

Cognitive Level: 1

REF: p. 369

11. In the activated partial thromboplastin time (APTT), calcium chloride is mixed with: a. kaolin. b. phospholipid component of thromboplastin. c. platelets. d. phospholipids component of thrombin. ANS: B


The APTT, like the prothrombin time assay, is automated using the same instrumentation as the PT. But in the APTT assay an activator, calcium chloride, is mixed with a phospholipid component of thromboplastin (a platelet substitute) before addition to the plasma being tested. PTS: 1

DIF:

Cognitive Level: 1

REF: p. 373

12. D-dimer assay is important in detecting: a. factor VIII deficiency. b. disseminated intravascular coagulation (DIC). c. fibrinogen deficiency. d. defective platelet aggregation. ANS: B

Particle agglutination is used for detecting D-dimers associated with DIC. PTS: 1

DIF:

Cognitive Level: 1

REF: p. 374

TRUE/FALSE 1. A function or activity of platelets is to form an aggregate plug or platelet mass that can

physically slow down or stop blood loss. ANS: T

Platelets have a variety of important functions to: • react to injury of vessels by forming an aggregate plug or platelet mass that can physically slow down or stop blood loss. • help activate and be a pTaE rtS icT ipB anAt N inKpSlaEsL mL aE coRa. guClaOtiMon to more effectively serve as a barrier to extensive blood loss. • maintain the endothelial lining of the blood vessels. In addition, in the formation of a platelet plug, fibronectin is secreted by endothelial cells and platelets. Platelet Factor 3 (PF3) is a phospholipoprotein (phospholipid) that is contained on or within the plasma membrane of the platelets and is needed in activating certain of the coagulation factors. PTS: 1

DIF:

Cognitive Level: 2

REF: p. 360

2. A function or activity of platelets is to help activate and be a participant in plasma

coagulation. ANS: T

Platelets have a variety of important functions to: • react to injury of vessels by forming an aggregate plug or platelet mass that can physically slow down or stop blood loss. • help activate and be a participant in plasma coagulation to more effectively serve as a barrier to extensive blood loss. • maintain the endothelial lining of the blood vessels. In addition, in the formation of a platelet plug, fibronectin is secreted by endothelial cells and platelets. Platelet Factor 3 (PF3) is a phospholipoprotein (phospholipid) that is contained on or within the plasma membrane of the platelets and is needed in activating certain of the coagulation factors.


PTS: 1

DIF:

Cognitive Level: 2

REF: p. 360

3. A function or activity of platelets is to maintain the endothelial lining of the blood vessels. ANS: T

Platelets have a variety of important functions to: • react to injury of vessels by forming an aggregate plug or platelet mass that can physically slow down or stop blood loss. • help activate and be a participant in plasma coagulation to more effectively serve as a barrier to extensive blood loss. • maintain the endothelial lining of the blood vessels. In addition, in the formation of a platelet plug, fibronectin is secreted by endothelial cells and platelets. Platelet Factor 3 (PF3) is a phospholipoprotein (phospholipid) that is contained on or within the plasma membrane of the platelets and is needed in activating certain of the coagulation factors. PTS: 1

DIF:

Cognitive Level: 2

REF: p. 360

4. A function or activity of platelets is to secrete fibronectin. ANS: T

Platelets have a variety of important functions to: • react to injury of vessels by forming an aggregate plug or platelet mass that can physically slow down or stop blood loss. • help activate and be a participant in plasma coagulation to more effectively serve as a barrier to extensive bloT oE dS loT ssB. ANKSELLER.COM • maintain the endothelial lining of the blood vessels. In addition, in the formation of a platelet plug, fibronectin is secreted by endothelial cells and platelets. Platelet Factor 3 (PF3) is a phospholipoprotein (phospholipid) that is contained on or within the plasma membrane of the platelets and is needed in activating certain of the coagulation factors. PTS: 1

DIF:

Cognitive Level: 2

REF: p. 360

5. A function or activity of platelets is to contain PF3 on or within the plasma membrane of the

platelets. ANS: T

Platelets have a variety of important functions to: • react to injury of vessels by forming an aggregate plug or platelet mass that can physically slow down or stop blood loss. • help activate and be a participant in plasma coagulation to more effectively serve as a barrier to extensive blood loss. •

maintain the endothelial lining of the blood vessels. In addition, in the formation of a platelet plug, fibronectin is secreted by endothelial cells and platelets. Platelet Factor 3 (PF3) is a phospholipoprotein (phospholipid) that is contained on or within the plasma membrane of the platelets and is needed in activating certain of the coagulation factors.


PTS: 1

DIF:

Cognitive Level: 2

REF: p. 360

MATCHING

Match the following coagulation factors with the appropriate description (use an answer only once). a. prothrombin b. hemophilia B c. von Willebrand’s factor d. fibrinogen 1. 2. 3. 4.

factor I factor II factor VIII factor IX

1. ANS: D REF: p. 362 2. ANS: A REF: p. 362 3. ANS: C REF: p. 362 4. ANS: B REF: p. 362

PTS: 1

DIF:

Cognitive Level: 2

PTS: 1

DIF:

Cognitive Level: 2

PTS: 1

DIF:

Cognitive Level: 2

PTS: 1

DIF:

Cognitive Level: 2

Match the three groups of coagulation factors based on their properties. a. factors II, VII, IX, and T XESTBANKSELLER.COM b. factors XI and XII, prekallikrein (Fletcher factor), and HMWK (Fitzgerald factor) c. factors I, V, VIII, and XIII 5. fibrinogen group 6. prothrombin group 7. contact group 5. ANS: C REF: p. 364 6. ANS: A REF: p. 364 7. ANS: B REF: p. 364

PTS: 1

DIF:

Cognitive Level: 1

PTS: 1

DIF:

Cognitive Level: 1

PTS: 1

DIF:

Cognitive Level: 1

Match the pathway with the appropriate description. a. It is thought that tissue injury, following exposure to foreign substances such as collagen, activates the pathway. b. The pathway taken when tissue thromboplastin, a substance not found in the blood, enters the vascular system and, in the presence of calcium and factor VII, activates factor X. 8. intrinsic pathway 9. extrinsic pathway


8. ANS: A REF: p. 365 9. ANS: B REF: p. 366

PTS: 1

DIF:

Cognitive Level: 2

PTS: 1

DIF:

Cognitive Level: 2


Chapter 13: Renal Physiology and Urinalysis Turgeon: Linné & Ringsrud’s Clinical Laboratory Science, 7th Edition MULTIPLE CHOICE 1. A routine urinalysis can provide information regarding: a. patient’s general health. b. a clinical picture of the patient. c. a potential disease. d. all of the above. ANS: D

In general, a urinalysis will provide information concerning reflects a patient’s general health, as well as a clinical picture of the patient and potential disease/disorder such as metabolic or systemic (nonrenal) disorders. Tests for the presence of protein, blood, nitrite, and leukocyte esterase together with the physical properties and the finding of casts, cells, and certain crystals are most helpful in assessing and treating renal and urinary tract disease. On the other hand, tests for glucose, ketone bodies, bilirubin, and urobilinogen are useful indicators in metabolic and systemic disorders such as diabetes and jaundice. PTS: 1

DIF:

Cognitive Level: 2

REF: p. 385

2. Quality assessment of control specimens should include: a. testing of all opened bottles of reagent strips or tablets each morning. b. testing of each new bottle on opening. c. recording data on the record sheet daily. TESTBANKSELLER.COM d. all of the above. ANS: D

Certain factors must be kept in mind and understood when the quality assessment of a urine control is conducted. These include: 1. test all opened bottles of reagent strips or tablets each morning. 2. test each new bottle on opening. 3. record data on the record sheet daily. PTS: 1

DIF:

Cognitive Level: 2

3. The functional (working) unit of the kidney is the: a. nephron. b. glomerulus. c. cortex. d. medulla. ANS: A

REF: p. 386


The urinary system consists of two kidneys, two ureters, the bladder, and the urethra. The working unit of the kidney is the nephron, where urine is formed. The formed urine flows from the kidney into the ureter and is passed to the bladder for temporary storage. It is eliminated from the body through the urethra. The nephron consists of the glomerulus and the renal tubules. The kidney itself has two anatomic portions, the outer cortex, which is made up of the glomerular portions of the nephron and the proximal convoluted tubules, and the central medulla, consisting of the loop of Henle, the distal convoluted tubules, and the collecting tubes. PTS: 1

DIF:

Cognitive Level: 1

REF: p. 387

4. Functions of the kidney include all of the following except: a. removal of waste products. b. processing of nutrients. c. retention of nutrients. d. acid-base, water, and electrolyte balance. ANS: B

The functions of the kidney are carried out by means of filtration, reabsorption, and secretion. The main functions include: 1. removal of waste products, primarily nitrogenous wastes from protein metabolism, and acids. 2. retention of nutrients, such as electrolytes, protein, water, and glucose. 3. acid-base balance. 4. water and electrolyte balance. 5. hormone synthesis, such as erythropoietin, renin, and vitamin D. PTS: 1

DIF:

TCEogSnTitiBveALNeKveSl:E1LLER.COM

REF: p. 387

5. The function of the distal convoluted tubule of the kidney is to: a. reduce the volume of the urine. b. maintain water and electrolyte balance. c. remove acid from the body. d. do both b and c. ANS: D

There are two main functions of the nephron at the distal tubule. The final reabsorption of sodium occurs at this point (maintaining water and electrolyte balance), and excess acid is removed from the body (acid-base balance). The descending and ascending loops of Henle function to reduce the volume of the urine while reabsorbing or recovering sodium and chloride. PTS: 1

DIF:

Cognitive Level: 2

REF: p. 389

6. Important factors in the collection and handling of urine include all of the following except

the: a. container used. b. collection procedure. c. conditions of storage and preservation from the time of collection until the

specimen is tested. d. use of a sterile container.


ANS: D

Important considerations in the proper collection and handling of urine for routine examination include the container used, the collection procedure, and the conditions of storage and preservation from the time of collection until the specimen is tested. A good working relationship and good avenues of communication are necessary between all members of the medical team to ensure that a suitable specimen is collected and delivered to the laboratory for examination. In general, sterile containers are not used. Sterile specimens are needed for microbiology cultures. PTS: 1

DIF:

Cognitive Level: 2

REF: p. 391

7. The most desirable type of routine urinalysis specimen is: a. sterile. b. first, early morning. c. random, mid-day. d. 24-hour. ANS: B

Although any random urine specimen voided during the day may be used for routine urinalysis, a fairly concentrated specimen is preferable to a dilute one. The first specimen voided in the morning is usually the most concentrated one, as fluids are not taken while the patient is asleep but dissolved substances are still excreted by the kidneys. A concentrated specimen is especially desirable for examining for protein and the contents of the urinary sediment. For testing for the presence of glucose, the best specimen to use is one voided 2 or 3 hours after a meal (postprandial). This is the major exception to the recommended use of the first morning specimen for routine tests. PTS: 1

DIF:

Cognitive Level: 1

REF: p. 391

8. Urine should be examined collection. a. immediately after b. within 30 minutes, if not refrigerated or suitably preserved at c. within 2 hours if kept at room temperature after d. within 24 hours if refrigerated after ANS: B

The most important consideration is that the urine specimen be fresh or suitably preserved, usually by refrigeration. Ideally, the urine should be examined within 30 minutes of collection, because decomposition begins within this time and urine is an excellent culture medium for bacterial growth. If examination within 30 minutes of collection is impossible, then test the specimen within 1 or 2 hours, but it should be refrigerated at 4° C as soon as possible after voiding. It can generally be preserved in this way for 6 to 8 hours, with no gross alterations (except possibly for bilirubin and urobilinogen, which are also susceptible to exposure to light). Refrigeration can precipitate amorphous urates or phosphates, which may obscure more important components in the microscopic examination. Specimens left for more than 2 hours at room temperature are not acceptable. Chemical preservatives are usually reserved for 24-hour urine collections, because they may interfere with parts of the routine urinalysis. PTS: 1

DIF:

Cognitive Level: 1

REF: p. 391


9. The major contributing factor to the decomposition of urine is: a. the growth of bacteria. b. the formation of crystals. c. fading of color. d. change in pH. ANS: A

Decomposition of urine primarily involves the growth of bacteria. At room temperature, bacteria reproduce rapidly. This bacterial growth results in a cloudy-looking specimen. Changes in pH also occur as a result of bacterial growth. These changes interfere markedly with other tests. Other substances, namely phosphates and urates, may precipitate out of solution, adding to the turbidity of the urine specimen. PTS: 1

DIF:

Cognitive Level: 1

REF: p. 393

10. The minimum acceptable volume of urine for routine urinalysis is a. 5 b. 12 c. 25 d. 50

mL.

ANS: B

The minimum volume for routine urinalysis is usually 12 mL, but 50 mL is preferable. Twelve milliliters is the minimum amount necessary for the usual processing procedure, in which 12 mL of urine is placed in a disposable centrifuge tube, centrifuged, and concentrated 12:1, so that 1 mL of sediment is retained for microscopic analysis of the sediment. This volume also allows for a coTnEvS enTieBnA t, NstK anSdEarLdL izE edRv.oCluOmMe of urine for assessment of physical properties, such as color and transparency, which are often observed in the centrifuge tube. Smaller volumes may be accepted for chemical analysis from oliguric patients or from infants. If only 3 mL of urine is collected, a 12:1 concentration of sediment may still be made. A drop of unconcentrated urine placed on the desired portion of a reagent strip or observed microscopically may be all that is possible in some situations. PTS: 1

DIF:

Cognitive Level: 1

11. The first part of the routine urinalysis is the a. physical b. chemical c. microscopic d. either b or c

REF: p. 392

examination.

ANS: A

The first part of a routine urinalysis usually involves an assessment of physical properties, such as volume, color, transparency, odor, and foam. These simple observations are extremely useful both for the eventual diagnosis of the patient and for the laboratory personnel who perform the complete urinalysis. Such tests often give clues leading to findings in subsequent portions of the urinalysis. For example, if a urine specimen is cloudy and red, the presence of red blood cells (RBCs) will probably be revealed by microscopic analysis of the urinary sediment.


PTS: 1

DIF:

Cognitive Level: 1

REF: p. 394

12. The normal pH of freshly voided urine is: a. 4 or 5. b. 5 or 6. c. 6 or 7. d. 7 or 8. ANS: B

Because the kidney is generally working to eliminate excess acid, the pH of urine is normally between 5 and 7, with a mean of 6. The kidney is capable of producing urine ranging in pH from 4.6 to 8.0. Freshly voided urine usually has a pH of 5 or 6. PTS: 1

DIF:

Cognitive Level: 2

REF: p. 389

13. In the detection and diagnosis of renal disease, the most significant single finding probably is: a. glucose. b. protein. c. pH d. ketones. ANS: B

In the detection and diagnosis of renal disease, probably the most significant single finding is that of urinary protein. The presence of protein when correlated with certain chemical tests, especially tests for blood, nitrite, and leukocyte esterase, and findings in the microscopic analysis of the urinary sediment, is part of the eventual diagnosis. PTS: 1

DIF:

TCEogSnTitiBveALNeKveSl:E2LLER.COM

REF: p. 400

14. In testing for glucose in the urine, all of the following are true except: a. if the blood glucose concentration becomes too high (usually greater than 180 to

200 mg/dL), the excess glucose will not be reabsorbed into the blood and will be eliminated from the body in the urine. b. the renal threshold is the lowest blood glucose concentration that will result in glycosuria. c. the most common condition in which the renal threshold for glucose is exceeded is diabetes mellitus. d. the occurrence of glycosuria is diagnostic of diabetes mellitus. ANS: D

The occurrence of measurable glucose in the urine is not normal. The blood glucose concentration normally varies between 60 and 110 mg/dL, depending on the method of analysis. After a meal it may increase to 120 to 160 mg/dL. Normally all the glucose in the blood is filtered by the glomerulus and reabsorbed into the blood. However, if the blood glucose concentration becomes too high (usually greater than 180 to 200 mg/dL), the excess glucose will not be reabsorbed into the blood and will be eliminated from the body in the urine. The lowest blood glucose concentration that will result in glycosuria is termed the renal threshold, and it varies somewhat from person to person. The most common condition in which the renal threshold for glucose is exceeded is diabetes mellitus. Although diabetes mellitus is suspected in cases of glycosuria, the occurrence of glycosuria is not diagnostic of the condition, because glycosuria has many other causes.


PTS: 1

DIF:

Cognitive Level: 2

REF: p. 406

15. Reagent strips for glucose detection use): a. an enzyme, glucose oxidase. b. a chemical, hydrogen peroxide. c. a chemical, copper sulfate. d. both a and b. ANS: D

The reagent strip tests for urinary sugar are specific for glucose because they are based on the use of the enzyme glucose oxidase. An enzyme may be described as a biological catalyst, a substance that must be present before a chemical reaction will occur. Glucose oxidase, like most enzymes, is absolutely specific. It will react only in the presence of glucose, and it will not react with any other substance. Hydrogen peroxide catalyzes the reaction. PTS: 1

DIF:

Cognitive Level: 1

REF: p. 407

16. A false-positive urinary glucose result can be observed if: a. contamination by bleach or other strong oxidizing agents occurs. b. reagent strips are exposed to air by improper storage. c. large urinary concentrations of ascorbic acid are present. d. both a and b occur. ANS: D

Because reagent strip tests are all specific for glucose, most interferences lead to reduced or false-negative results. False-positive results • Contamination by bleach or other strong oxidizing agents may oxidize the reduced form of the dye present on the reagent strip, causing a color change in the absence of glucose. This shows the importance of using contamination-free urine containers and work surfaces. • Trace values may be seen in very dilute urine specimens because of increased sensitivity at low-specific gravity. • Reagent strips exposed to air by improper storage have been shown to give false-positive results. False-negative or delayed results • Large urinary concentrations of ascorbic acid, from therapeutic doses of vitamin C or from drugs, such as tetracyclines, in which ascorbic acid is used as a reducing agent. Ascorbic acid blocks or delays the reaction by acting as a reducing agent reacting with the released hydrogen peroxide (rather than the chromogen in the reagent strip). Multistix products are inhibited by ascorbic acid concentrations of 50 mg/dL (500 mg/L) or greater in specimens containing small amounts (75 to 125 mg/dL) of glucose. Chemstrip is unaffected by ascorbic acid concentration less than 100 mg/dL (1000 mg/L). • With questionable ascorbic acid interference, repeat tests on urine voided at least 10 hours after the last administration of vitamin C, or test for ascorbic acid. • Ketone bodies at moderate levels (=40 mg/dL) in specimens containing small amounts (75 to 125 mg/dL) of glucose. This combination of high ketone and low glucose is unlikely in a patient with diabetes. • Sodium fluoride is an enzyme inhibitor. Do not use as a preservative.


Refrigerated specimens, because of decreased enzyme activity. Urine must be at room temperature when tested.

PTS: 1

DIF:

Cognitive Level: 2

REF: p. 407

17. Ketone bodies can be detected in urine in: a. lack of adequate insulin control. b. starvation. c. extreme dieting. d. all of the above. ANS: D

The presence of ketone bodies in urine is an early indication of lack of adequate insulin control. Ketone bodies can also be found in the urine in conditions of starvation or extreme dieting. PTS: 1

DIF:

Cognitive Level: 2

REF: p. 408

18. The presence of bilirubin in the urine is an early sign of: a. liver disease. b. pancreatic cancer. c. intestinal inflammation. d. various infectious diseases. ANS: A

The presence of bilirubin in the urine is an early sign of liver cell disease (hepatocellular disease) and of obstruction to the bile flow from the liver. It is especially useful in the early detection and monitoring oTf E heSpT atB itA isN , aKhSigEhL lyLiE nfRec.tiCoO usMdisease of particular importance to laboratory workers. PTS: 1

DIF:

Cognitive Level: 1

REF: p. 410

19. Specimens for bilirubin testing must be: a. absolutely fresh. b. less than 2 hours old. c. less than 8 hours old. d. less than 24 hours old. ANS: A

Urine specimens must be tested when absolutely fresh or bilirubin will oxidize to biliverdin. The test is specific for bilirubin; it will not react with biliverdin. PTS: 1

DIF:

Cognitive Level: 1

REF: p. 412

20. Select the microscopic elements that could be found in normal or abnormal urinary sediment:

0-5/hpf (high-power field) RBCs. a. Normal b. Abnormal ANS: A

A few RBCs are present in the urine of healthy persons. The number varies, but generally 5 or fewer RBCs per hpf in the concentrated sediment is considered normal.


The presence of a few white blood cells (WBCs) or leukocytes in the concentrated urine sediment is normal. Reference values vary, but more than a few (as many as five WBCs/hpf) is considered abnormal. A few squamous epithelial cells are seen in most urine specimens. Renal epithelial cells are the single layer of cells that line the nephron from the proximal to the distal convoluted tubules, plus the cells lining the collecting ducts to the pelvis of the kidney. Their occurrence in urine is important, for it implies a serious pathological condition and destruction of renal tubules, as does the presence of epithelial casts. Most urine specimens contain at least a few bacteria because of contamination when the urine is voided. Bacteria multiply rapidly when urine stands at room temperature. In specimens that are obtained in a manner suitable for urine culture and kept under sterile conditions, the presence of bacteria may indicate a urinary tract infection. Simple hyaline casts are the least important clinically, and a few (<2 per low-power field [lpf]) may be seen in urine from healthy persons. Simple hyaline casts may be seen in increased numbers after strenuous exercise; however, the sediment returns to normal in 24 to 48 hours. They may be seen in large numbers (20 or 30/lpf) in moderate or severe renal disease. PTS: 1

DIF:

Cognitive Level: 2

REF: p. 416

21. Select the microscopic elements that could be found in normal or abnormal urinary sediment:

>10 WBCs/hpf. a. Normal b. Abnormal ANS: B

A few RBCs are present inTtE heSuTriBnA eN ofKhSeaEltLhL yE peRrs.oC nsO. M The number varies, but generally 5 or fewer RBCs per hpf in the concentrated sediment is considered normal. The presence of a few white blood cells (WBCs) or leukocytes in the concentrated urine sediment is normal. Reference values vary, but more than a few (as many as five WBCs/hpf) is considered abnormal. A few squamous epithelial cells are seen in most urine specimens. Renal epithelial cells are the single layer of cells that line the nephron from the proximal to the distal convoluted tubules, plus the cells lining the collecting ducts to the pelvis of the kidney. Their occurrence in urine is important, for it implies a serious pathological condition and destruction of renal tubules, as does the presence of epithelial casts. Most urine specimens contain at least a few bacteria because of contamination when the urine is voided. Bacteria multiply rapidly when urine stands at room temperature. In specimens that are obtained in a manner suitable for urine culture and kept under sterile conditions, the presence of bacteria may indicate a urinary tract infection. Simple hyaline casts are the least important clinically, and a few (<2 per low-power field [lpf]) may be seen in urine from healthy persons. Simple hyaline casts may be seen in increased numbers after strenuous exercise; however, the sediment returns to normal in 24 to 48 hours. They may be seen in large numbers (20 or 30/lpf) in moderate or severe renal disease. PTS: 1

DIF:

Cognitive Level: 2

REF: p. 418

22. Select the microscopic elements that could be found in normal or abnormal urinary sediment:

Moderate number of squamous epithelial cells.


a. Normal b. Abnormal ANS: A

A few RBCs are present in the urine of healthy persons. The number varies, but generally 5 or fewer RBCs per hpf in the concentrated sediment is considered normal. The presence of a few white blood cells (WBCs) or leukocytes in the concentrated urine sediment is normal. Reference values vary, but more than a few (as many as five WBCs/hpf) is considered abnormal. A few squamous epithelial cells are seen in most urine specimens. Renal epithelial cells are the single layer of cells that line the nephron from the proximal to the distal convoluted tubules, plus the cells lining the collecting ducts to the pelvis of the kidney. Their occurrence in urine is important, for it implies a serious pathological condition and destruction of renal tubules, as does the presence of epithelial casts. Most urine specimens contain at least a few bacteria because of contamination when the urine is voided. Bacteria multiply rapidly when urine stands at room temperature. In specimens that are obtained in a manner suitable for urine culture and kept under sterile conditions, the presence of bacteria may indicate a urinary tract infection. Simple hyaline casts are the least important clinically, and a few (<2 per low-power field [lpf]) may be seen in urine from healthy persons. Simple hyaline casts may be seen in increased numbers after strenuous exercise; however, the sediment returns to normal in 24 to 48 hours. They may be seen in large numbers (20 or 30/lpf) in moderate or severe renal disease. PTS: 1

DIF:

Cognitive Level: 2

REF: p. 420

23. Select the microscopic elem haAt N coKuS ldEbLeLfoEuRnd.C inOnM ormal or abnormal urinary sediment: TeEnStsTtB

Moderate number of renal epithelial cells. a. Normal b. Abnormal ANS: B

A few RBCs are present in the urine of healthy persons. The number varies, but generally 5 or fewer RBCs per hpf in the concentrated sediment is considered normal. The presence of a few white blood cells (WBCs) or leukocytes in the concentrated urine sediment is normal. Reference values vary, but more than a few (as many as five WBCs/hpf) is considered abnormal. A few squamous epithelial cells are seen in most urine specimens. Renal epithelial cells are the single layer of cells that line the nephron from the proximal to the distal convoluted tubules, plus the cells lining the collecting ducts to the pelvis of the kidney. Their occurrence in urine is important, for it implies a serious pathological condition and destruction of renal tubules, as does the presence of epithelial casts. Most urine specimens contain at least a few bacteria because of contamination when the urine is voided. Bacteria multiply rapidly when urine stands at room temperature. In specimens that are obtained in a manner suitable for urine culture and kept under sterile conditions, the presence of bacteria may indicate a urinary tract infection.


Simple hyaline casts are the least important clinically, and a few (<2 per low-power field [lpf]) may be seen in urine from healthy persons. Simple hyaline casts may be seen in increased numbers after strenuous exercise; however, the sediment returns to normal in 24 to 48 hours. They may be seen in large numbers (20 or 30/lpf) in moderate or severe renal disease. PTS: 1

DIF:

Cognitive Level: 2

REF: p. 421

24. Select the microscopic elements that could be found in normal or abnormal urinary sediment:

Few bacteria in a fresh, nonsterile specimen. a. Normal b. Abnormal ANS: A

A few RBCs are present in the urine of healthy persons. The number varies, but generally 5 or fewer RBCs per hpf in the concentrated sediment is considered normal. The presence of a few white blood cells (WBCs) or leukocytes in the concentrated urine sediment is normal. Reference values vary, but more than a few (as many as five WBCs/hpf) is considered abnormal. A few squamous epithelial cells are seen in most urine specimens. Renal epithelial cells are the single layer of cells that line the nephron from the proximal to the distal convoluted tubules, plus the cells lining the collecting ducts to the pelvis of the kidney. Their occurrence in urine is important, for it implies a serious pathological condition and destruction of renal tubules, as does the presence of epithelial casts. Most urine specimens contain at least a few bacteria because of contamination when the urine is voided. Bacteria multiply rapidly when urine stands at room temperature. In specimens that are obtaineT dE inSaTmBaAnN neKr S suEitL abLlE eR fo. rC urO inMe culture and kept under sterile conditions, the presence of bacteria may indicate a urinary tract infection. Simple hyaline casts are the least important clinically, and a few (<2 per low-power field [lpf]) may be seen in urine from healthy persons. Simple hyaline casts may be seen in increased numbers after strenuous exercise; however, the sediment returns to normal in 24 to 48 hours. They may be seen in large numbers (20 or 30/lpf) in moderate or severe renal disease. PTS: 1

DIF:

Cognitive Level: 2

REF: p. 434

25. Select the microscopic elements that could be found in normal or abnormal urinary sediment:

Hyaline cases (20 to 30/lpf). a. Normal b. Abnormal ANS: B

A few RBCs are present in the urine of healthy persons. The number varies, but generally 5 or fewer RBCs per hpf in the concentrated sediment is considered normal. The presence of a few white blood cells (WBCs) or leukocytes in the concentrated urine sediment is normal. Reference values vary, but more than a few (as many as five WBCs/hpf) is considered abnormal.


A few squamous epithelial cells are seen in most urine specimens. Renal epithelial cells are the single layer of cells that line the nephron from the proximal to the distal convoluted tubules, plus the cells lining the collecting ducts to the pelvis of the kidney. Their occurrence in urine is important, for it implies a serious pathological condition and destruction of renal tubules, as does the presence of epithelial casts. Most urine specimens contain at least a few bacteria because of contamination when the urine is voided. Bacteria multiply rapidly when urine stands at room temperature. In specimens that are obtained in a manner suitable for urine culture and kept under sterile conditions, the presence of bacteria may indicate a urinary tract infection. Simple hyaline casts are the least important clinically, and a few (<2 per low-power field [lpf]) may be seen in urine from healthy persons. Simple hyaline casts may be seen in increased numbers after strenuous exercise; however, the sediment returns to normal in 24 to 48 hours. They may be seen in large numbers (20 or 30/lpf) in moderate or severe renal disease. PTS: 1

DIF:

Cognitive Level: 2

REF: p. 424

26. Structures that can be confused microscopically with RBCs include: a. leukocytes (WBCs). b. yeast. c. calcium oxalate crystals. d. both b and c. ANS: D

Yeast and calcium oxalate crystals can be confused with RBCs when viewed microscopically. Yeast cells are often mistaken for RBCs. They are generally smaller than RBCs and show considerable size variationT , eEvS enTw YO eaMst cells have a typically ovoid shape, BiAthNinKaSsEpLecLim EeRn.. C lack color, and have a smooth and refractive appearance. The most distinguishing characteristic is the presence of little buds, or projections, because of their manner of reproduction. PTS: 1

DIF:

Cognitive Level: 2

REF: pp. 417-418

27. The least pathological type of cast is: a. hyaline. b. finely granular. c. waxy. d. fatty. ANS: A

Hyaline casts can be seen in a variety of conditions. Waxy and fatty casts are more indicative of renal destruction. PTS: 1

DIF:

Cognitive Level: 2

REF: p. 423

TRUE/FALSE 1. Urine specimens must be fresh and centrifuged for reagent strip testing. ANS: F


A urine specimen to be tested must be fresh, or adequately preserved, well mixed, not centrifuged, and at room temperature. The strip must be adequately moistened so that all test areas of the strip are brought into contact with the sample. Care must be taken not to leave the strip in contact with the sample too long, or chemicals will be leached out of the strip and be unavailable for the chemical reaction to occur. The strip is inserted into the specimen only briefly, for 1 second or less. Reagent strips must be kept in tightly capped containers. They will deteriorate rapidly when exposed to moisture, direct sunlight, heat, or volatile substances. Once the container is opened, use strips within 6 months. Chemstrip products are generally read at an end point or stable phase, allowing all results to be read at 1 minute, with results stable for 2 minutes. PTS: 1

DIF:

Cognitive Level: 2

REF: p. 398

2. The longer a reagent strip is in contact with a urine specimen, the better the results that will be

observed. ANS: F

A urine specimen to be tested must be fresh, or adequately preserved, well mixed, not centrifuged, and at room temperature. The strip must be adequately moistened so that all test areas of the strip are brought into contact with the sample. Care must be taken not to leave the strip in contact with the sample too long, or chemicals will be leached out of the strip and be unavailable for the chemical reaction to occur. The strip is inserted into the specimen only briefly, for 1 second or less. Reagent strips must be kept in tightly capped containers. They will deteriorate rapidly when exposed to moisture, direct sunlight, heat, or volatile substances. Once the container is opened, use strips within 6 months. Chemstrip products are generally read at an end point or stable phase, allowing all results to be read at 1 minute, with results stable for 2 minutes. PTS: 1

DIF:

Cognitive Level: 2

REF: p. 398

3. Reagent strips will deteriorate if the cap is kept open for a prolonged period. ANS: T

A urine specimen to be tested must be fresh, or adequately preserved, well mixed, not centrifuged, and at room temperature. The strip must be adequately moistened so that all test areas of the strip are brought into contact with the sample. Care must be taken not to leave the strip in contact with the sample too long, or chemicals will be leached out of the strip and be unavailable for the chemical reaction to occur. The strip is inserted into the specimen only briefly, for 1 second or less. Reagent strips must be kept in tightly capped containers. They will deteriorate rapidly when exposed to moisture, direct sunlight, heat, or volatile substances. Once the container is opened, use strips within 6 months. Chemstrip products are generally read at an end point or stable phase, allowing all results to be read at 1 minute, with results stable for 2 minutes. PTS: 1

DIF:

Cognitive Level: 2

4. Reagent strips will deteriorate if exposed to moisture or heat. ANS: T

REF: p. 398


A urine specimen to be tested must be fresh, or adequately preserved, well mixed, not centrifuged, and at room temperature. The strip must be adequately moistened so that all test areas of the strip are brought into contact with the sample. Care must be taken not to leave the strip in contact with the sample too long, or chemicals will be leached out of the strip and be unavailable for the chemical reaction to occur. The strip is inserted into the specimen only briefly, for 1 second or less. Reagent strips must be kept in tightly capped containers. They will deteriorate rapidly when exposed to moisture, direct sunlight, heat, or volatile substances. Once the container is opened, use strips within 6 months. Chemstrip products are generally read at an end point or stable phase, allowing all results to be read at 1 minute, with results stable for 2 minutes. PTS: 1

DIF:

Cognitive Level: 2

REF: p. 398

5. Reagent strips must be used within 1 year of opening the container. ANS: F

A urine specimen to be tested must be fresh, or adequately preserved, well mixed, not centrifuged, and at room temperature. The strip must be adequately moistened so that all test areas of the strip are brought into contact with the sample. Care must be taken not to leave the strip in contact with the sample too long, or chemicals will be leached out of the strip and be unavailable for the chemical reaction to occur. The strip is inserted into the specimen only briefly, for 1 second or less. Reagent strips must be kept in tightly capped containers. They will deteriorate rapidly when exposed to moisture, direct sunlight, heat, or volatile substances. Once the container is opened, use strips within 6 months. Chemstrip products are generally read at an end point or stable phase, allowing all results to be read at 1 minute, with results stable for 2 minutes. PTS: 1

DIF:

Cognitive Level: 2

REF: p. 398

6. Reagent strips (e.g., Chemstrips) have a stable reaction color for up to 5 minutes. ANS: F

A urine specimen to be tested must be fresh, or adequately preserved, well mixed, not centrifuged, and at room temperature. The strip must be adequately moistened so that all test areas of the strip are brought into contact with the sample. Care must be taken not to leave the strip in contact with the sample too long, or chemicals will be leached out of the strip and be unavailable for the chemical reaction to occur. The strip is inserted into the specimen only briefly, for 1 second or less. Reagent strips must be kept in tightly capped containers. They will deteriorate rapidly when exposed to moisture, direct sunlight, heat, or volatile substances. Once the container is opened, use strips within 6 months. Chemstrip products are generally read at an end point or stable phase, allowing all results to be read at 1 minute, with results stable for 2 minutes. PTS: 1

DIF:

Cognitive Level: 2

7. Upon standing, RBCs either swell or become crenated. ANS: T

REF: p. 398


Upon standing, RBCs either swell or become crenated. WBCs rapidly deteriorate in hypotonic solutions. Casts disintegrate as urine becomes alkaline. The growth of bacteria in urine produces a more alkaline solution. PTS: 1

DIF:

Cognitive Level: 2

REF: p. 437

8. WBCs rapidly deteriorate in hypotonic solutions. ANS: T

Upon standing, RBCs either swell or become crenated. WBCs rapidly deteriorate in hypotonic solutions. Casts disintegrate as urine becomes alkaline. The growth of bacteria in urine produces a more alkaline solution. PTS: 1

DIF:

Cognitive Level: 2

REF: p. 437

9. Casts disintegrate as urine becomes alkaline. ANS: T

Upon standing, RBCs either swell or become crenated. WBCs rapidly deteriorate in hypotonic solutions. Casts disintegrate as urine becomes alkaline. The growth of bacteria in urine produces a more alkaline solution. PTS: 1

DIF:

Cognitive Level: 2

REF: p. 437

10. The growth of bacteria in urine produces a more acidic solution. ANS: F

ST MBCs rapidly deteriorate in hypotonic Upon standing, RBCs eitheTrEsw elB l oArNbK ecSoE mL eL crE enRa. teC d.OW solutions. Casts disintegrate as urine becomes alkaline. The growth of bacteria in urine produces a more alkaline solution. PTS: 1

DIF:

Cognitive Level: 2

REF: p. 437

MATCHING

Match the terms with the appropriate situations that result in abnormal urine volumes. a. any increase in urine volume, even if the increase is only temporary b. complete absence of urine formation c. abnormally small amount of urine, less than 500 mL/24 hours d. excretion of more 400 mL urine at night e. consistent elimination of an abnormally large volume of urine (more than 2000 mL/24 hours) 1. 2. 3. 4. 5.

polyuria diuresis oliguria anuria nocturia

1. ANS: E

PTS: 1

DIF:

Cognitive Level: 1


REF: 2. ANS: REF: 3. ANS: REF: 4. ANS: REF: 5. ANS: REF:

pp. 394-395 A pp. 394-395 C pp. 394-395 B pp. 394-395 D pp. 394-395

PTS: 1

DIF:

Cognitive Level: 1

PTS: 1

DIF:

Cognitive Level: 1

PTS: 1

DIF:

Cognitive Level: 1

PTS: 1

DIF:

Cognitive Level: 1

Match the following potential causes of false-positive or false-negative results with chemical reagent strips. a. false positive b. false negative 6. proteins are present without the presence of albumin 7. a blue color forms for the protein reaction, if the urine is exposed to the reagent strip for too 8. 9. 10. 11. 12.

long the reaction for protein may be produced, if a urine specimen is exceptionally alkaline or highly buffered in the reaction to detect blood in the urine, microbial peroxidase activity associated with urinary tract infection can produce in the detection of blood in the urine, ascorbic acid in urine specimens containing more than 25 mg/dL medications (e.g., azo-containing compounds or dyes that color urine red or that turn red in an acidic medium when testing for nitrite for the detection of bacteria in urine) urinary preservatives in uriTnE eS teT stB inA gN foKr S leE uL koLcE ytR e. esCteOraMse

6. ANS: B REF: p. 402 7. ANS: A REF: p. 402 8. ANS: A REF: p. 402 9. ANS: A REF: p. 404 10. ANS: B REF: p. 404 11. ANS: A REF: p. 405 12. ANS: A REF: p. 406

PTS: 1

DIF:

Cognitive Level: 2

PTS: 1

DIF:

Cognitive Level: 2

PTS: 1

DIF:

Cognitive Level: 2

PTS: 1

DIF:

Cognitive Level: 2

PTS: 1

DIF:

Cognitive Level: 2

PTS: 1

DIF:

Cognitive Level: 2

PTS: 1

DIF:

Cognitive Level: 2

Match the respective type of casts with an associated condition (use an answer only once). a. exposure to nephrotoxic substances b. urinary tract infection c. acute pyelonephritis d. glomerular damage 13. WBC casts 14. epithelial casts


15. RBC casts and RBCs 16. bacterial casts 13. ANS: B REF: p. 424 14. ANS: A REF: p. 425 15. ANS: D REF: p. 425 16. ANS: C REF: p. 426

PTS: 1

DIF:

Cognitive Level: 2

PTS: 1

DIF:

Cognitive Level: 2

PTS: 1

DIF:

Cognitive Level: 2

PTS: 1

DIF:

Cognitive Level: 2

Match the name of each type of crystal with the appropriate description. a. found in acid urine b. envelope-shaped, glistening and colorless c. coffin lids d. colorless slender needles that are strongly birefringent with polarized light e. yellow or reddish-brown shapes (e.g., lemon- or barrel-shaped forms) 17. 18. 19. 20. 21.

amorphous urates uric acid calcium oxalate triple phosphate acyclovir

17. ANS: A REF: p. 429 18. ANS: E REF: p. 429 19. ANS: B REF: p. 432 20. ANS: C REF: p. 432 21. ANS: D REF: p. 434

PTS: 1

DIF:

Cognitive Level: 2

PTS: 1

DIF:

Cognitive Level: 2

PTS: 1

DIF:

Cognitive Level: 2

PTS: 1

DIF:

Cognitive Level: 2

PTS: 1

DIF:

Cognitive Level: 2


Chapter 14: Examination of Body Fluids and Miscellaneous Specimens Turgeon: Linné & Ringsrud’s Clinical Laboratory Science, 7th Edition MULTIPLE CHOICE 1. The type of specimen that is most critical to deliver immediately to the laboratory for

examination is a. cerebrospinal b. synovial c. pericardial d. peritoneal

fluid.

ANS: A

CSF specimens must be immediately delivered to the laboratory for examination. The four or five collection tubes must be handled using standard precautions. Tubes are designated for routine testing in hematology, microbiology, clinical chemistry, and immunology/serology. PTS: 1

DIF:

Cognitive Level: 1

REF: p. 452

2. A macroscopically turbid cerebrospinal fluid (CSF) may indicate: a. blood clots. b. increased white blood cell count. c. presence of hemorrhage. d. viral infection. ANS: B

Ra.teCr)O. M Normal spinal fluid is crysT taE l cSleTaB rA (itNlK ooSkE sL likLeEw Slight haziness in the specimen or turbidity may indicate an increased white blood cell count. Turbidity in spinal fluid may result from the presence of large numbers of leukocytes, bacteria, increased protein, or lipid. In addition, subarachnoid bleeding may be indicated by the presence of xanthochromia. PTS: 1

DIF:

Cognitive Level: 1

REF: p. 453

3. If CSF has a xanthochromic appearance, it can indicate bleeding: a. within 30 minutes to 1 hour. b. within 1 to 2 hours. c. between 12 hours and 2 to 4 weeks. d. within the past month. ANS: C

When the hemorrhage is old, the supernatant fluid will show yellow xanthochromia. The yellow color is caused by bilirubin, formed from hemoglobin from lysed red blood cells. This appears about 12 hours after a bleeding episode, peaks in 2 to 4 days, and gradually disappears in 2 to 4 weeks. PTS: 1

DIF:

Cognitive Level: 2

REF: p. 454

4. When a leukocyte differential smear made from a CSF specimen with an elevated leukocyte

count reveals an increase in polymorphonuclear cells, it may be indicative of a. viral

infection.


b. bacterial c. fungal d. parasitic ANS: B

A predominance of polynuclear cells usually indicates a bacterial infection, while the presence of many mononuclear cells indicates a viral infection. PTS: 1

DIF:

Cognitive Level: 2

REF: p. 455

5. CSF specimens should be delivered to the laboratory: a. immediately. b. within an hour. c. within 12 hours. d. within 24 hours. ANS: A

CSF specimens need to be delivered and assayed immediately. Chemical levels (e.g., glucose) can be altered if the specimen testing is delayed. PTS: 1

DIF:

Cognitive Level: 2

REF: p. 452

6. Serous fluids: a. have a composition that is unlike serum. b. are contained within the closed cavities of the body. c. are ultrafiltrates of plasma. d. Both a and b are correct. ANS: D

Serous fluids, a fluid that has a composition similar to serum, are the fluids contained within the closed cavities of the body. These cavities are lined by a contiguous membrane that forms a double layer of mesothelial cells, called the serous membrane. The fluids are ultrafiltrates of plasma, which are continuously formed and reabsorbed, leaving only a very small volume within the cavities. PTS: 1

DIF:

Cognitive Level: 2

REF: p. 457

7. Serous fluid for laboratory examination should be collected in: a. a sterile container. b. an EDTA tube for cell counts. c. an aseptic environment. d. all of the above. ANS: D

Serous fluid must be collected aseptically into a sterile container. One container should be an EDTA tube to enable the performance of cell counts. PTS: 1

DIF:

Cognitive Level: 2

8. Synovial fluids should be collected: a. into an anticoagulated tube. b. into a non-anticoagulated tube.

REF: p. 458


c. using a sterile needle and syringe. d. in all of the above ways. ANS: D

Synovial fluid is collected by needle aspiration, which is called arthrocentesis. The fluid is collected with a disposable needle and plastic syringe, to avoid contamination with confusing birefringent material. The fluid should be collected both anticoagulated and unanticoagulated. PTS: 1

DIF:

Cognitive Level: 2

9. Ideally, a semen specimen should be analyzed within a. 30 minutes b. 1 hour c. 2 hours d. 12 hours

REF: p. 460

of collection.

ANS: A

Ideally, seminal fluid should be analyzed within 30 minutes of collection. It is mandatory that the specimen be kept at 37° C and examined within 1 to 2 hours of collection. PTS: 1

DIF:

Cognitive Level: 2

REF: p. 464

10. Seminal fluid may be examined for: a. infertility studies. b. postvasectomy assessment. c. evaluation of probable sexual assault. d. all of the above. ANS: D

Seminal fluid can be analyzed for a variety of reasons, including infertility studies, artificial insemination protocols, postvasectomy assessment, and evaluation of probable sexual assault. PTS: 1 11. Normally, a. 15 b. 25 c. 50 d. 100

DIF:

Cognitive Level: 1

REF: p. 464

% or more of sperm are motile.

ANS: C

Normally, 50% or more sperm are motile. PTS: 1

DIF:

Cognitive Level: 1

REF: p. 465

12. An assay for fetal fibronectin (fFN) is performed, if the: a. mother is having symptoms of premature labor. b. fetus is suffering from hemolytic anemia. c. parents want to know the gender of the fetus. d. obstetrician wants to know if the fetus has a genetic disorder. ANS: A


fFN is performed if a woman is 26 to 34 weeks’ pregnant and having symptoms of premature labor. The goal then is to intervene to prevent the potentially serious health complications of a preterm baby. PTS: 1

DIF: Cognitive Level: 1

REF: p. 465


Chapter 15: Introduction to Microbiology Turgeon: Linné & Ringsrud’s Clinical Laboratory Science, 7th Edition MULTIPLE CHOICE 1. Mycology is the study of: a. viruses. b. parasites. c. yeasts and molds. d. bacteria. ANS: C

The study of fungi (yeast and molds) is called mycology. Virology is the study of viruses; parasitology is the study of parasites; and bacteriology is the study of bacteria. PTS: 1

DIF:

Cognitive Level: 1

REF: p. 516

2. What is the name of the type of infection that is acquired in a hospital? a. Opportunistic infection b. Normal flora c. Community acquired infection d. Nosocomial infection ANS: D

Infections that a patient acquires while they are in the hospital are called nosocomial infections. The organism involved was neither present nor incubating in the patient before TnES admission to the hospital. A opTpB orAtuNnK isS ticEiLnL feE ctR io. nCreOfeMrs to disease in a person whose immune system is compromised. These organisms usually do not cause disease in healthy individuals. A community-acquired infection is acquired outside the hospital setting. Normal flora is the population of organisms that reside in an individual and are beneficial to the individual. PTS: 1

DIF:

Cognitive Level: 1

REF: p. 478

3. All of the following are structures found in prokaryotic organisms except: a. mitochondria. b. capsules. c. ribosomes. d. cell wall. ANS: A

Prokaryotic organisms do not have membrane bound organelles such as mitochondria. They do have ribosomes, cell walls, and capsules. PTS: 1

DIF:

Cognitive Level: 1

4. Which of the following systematic designations is correct? a. Staphylococcus aureus b. Staphylococcus Aureus c. Staphylococcus aureus

REF: p. 478


d. Staphylococcus Aureus ANS: C

The correct convention to write the names of organisms is in italics. The genus is capitalized (Staphylococcus) and the species designation is lowercase (aureus). PTS: 1

DIF:

Cognitive Level: 1

REF: p. 480

5. What is the biosafety level of most clinical microbiology laboratories? a. Biosafety Level 1 b. Biosafety Level 2 c. Biosafety Level 3 d. Biosafety Level 4 ANS: B

Most clinical microbiology laboratories perform necessary standard precautions at Biosafety Level 2. This level is appropriate for the type of pathogens encountered in the clinical setting. These precautions include: specific training in handling pathogens, limited access to the laboratory, precautions for contaminated sharps, and use of a biological safety cabinet. Biosafety levels 3 and 4 require more stringent controls because the pathogens that are being handled are potentially lethal agents. Biosafety Level 1 agents are of a minimal hazardous nature and good standard work practices are used at this level. PTS: 1

DIF:

Cognitive Level: 1

REF: p. 480

6. Which of the following is a false statement when considering proper disinfection processes? a. The disinfectant should be dried immediately for best results. b. Phenol and diluted bleaTcE hS arTeBtw ctL ivL eE diR si. nfCeO ctM ants. AoNeKffSeE c. The work area should be disinfected prior to use and again when work is

completed. d. Disinfection is an ongoing process. ANS: A

The longer the surface is allowed to remain wet the more effective the disinfection process will be. Options b, c, and d are all true statements. Diluted bleach is an effective disinfectant particularly for eliminating viral contamination. In all microbiology laboratories, disinfection is an ongoing process whereby personnel are vigilant to make sure that the working environment is safe. PTS: 1

DIF:

Cognitive Level: 1

REF: p. 481

7. The technique of killing or destroying all microorganisms including bacterial spores

(endospores) is called: a. antisepsis. b. sterilization. c. physical disinfection. d. chemical disinfection. ANS: B


Endospores are difficult to kill or destroy, and sterilization methods are those techniques that effectively achieve this goal. Disinfection techniques, physical or chemical, will destroy many organisms but not necessarily all organisms. Also, disinfection will not destroy bacterial spores. Antisepsis is the process used to reduce the number of microorganisms on the surface of the skin. PTS: 1

DIF:

Cognitive Level: 1

REF: p. 482

8. Which of the following is a sterilization procedure that uses moist heat and how is this method

effective in killing bacterial spores? a. Filtration; filters all microbes larger than the filter pore size b. Boiling; kills all microorganisms and spores at 100° C (boiling) c. Hot-air chamber; kills all microorganisms and bacterial spores at 171° C for at least 1 hour d. Autoclaving; kills all microorganisms and bacterial spores at 121° C for 15 minutes at 15 lb above atmospheric pressure ANS: D

Autoclaving at 121° C for 15 minutes at 15 lb of pressure is an effective sterilization technique. All microorganisms (vegetative state) and bacterial spores are killed. Boiling is also a moist heat method but does not kill bacterial spores. The temperature is 100° C when boiling and the temperature is 121° C when autoclaving. The higher temperature in the autoclave is achieved by the use of pressure. Filtration is not a moist heat method but is an effective sterilization technique. Hot-air chambers use dry heat as a method for sterilizing items such as glassware. PTS: 1

DIF:

TCEogSnTitiBveALNeKveSl:E2LLER.COM

REF: p. 482

9. Sending appropriately collected specimens to the laboratory before antibiotic therapy is

initiated is important because: a. pathogenic bacteria could be hard to recover because of the effects of the antibiotic. b. collection of specimens from around the wound area can result in contamination of the culture. c. collection of a sputum specimen contaminated with saliva could yield a culture result that reflects normal mouth flora. d. All of the above are correct. ANS: D

All are correct. Antibiotics will begin to inhibit or kill the bacteria quickly and recovery of the pathogen may not be possible. Physicians act upon the results of the laboratory. Collection of specimens is also very important because, if it is a poor specimen, the culture results will be less than optimal. Saliva contaminated sputum could yield results that reflect the mouth flora and not the lower respiratory area where the pathogens are causing infection. Contaminated wound specimens make it difficult and more costly to isolate the pathogen causing the infection. PTS: 1

DIF:

Cognitive Level: 2

10. Which of the following specimens can be refrigerated? a. Urine for routine culture

REF: p. 483


b. Cerebrospinal fluid c. Genital culture d. Culture for meningitis ANS: A

Cerebrospinal fluid is not refrigerated. These specimens could contain organisms such as Neisseria meningitidis that are sensitive to reduced temperatures. Genital cultures could also contain Neisseria gonorrhoeae that is sensitive to reduced temperatures. PTS: 1

DIF:

Cognitive Level: 1

REF: p. 484

11. Which of the following methods is appropriate to collect blood for culture? a. Cleaning the venipuncture site with 70% alcohol b. Cleaning the venipuncture site with 70% alcohol, followed by povidone-iodine, at

the intended venipuncture site c. Cleaning the venipuncture site with 70% alcohol, followed by povidone-iodine,

moving in a circular motion beginning at the periphery of the area to be cleaned inward toward the venipuncture site d. All of the above ANS: B

The proper collection technique for blood cultures is to begin at the venipuncture site and move outward in a circular motion. This will eliminate potential contaminating skin flora at the site of the venipuncture than using the reverse option of working at the periphery inward. 70% alcohol solutions are adequate for drawing blood for other types of testing in the clinical laboratory but alcohol alone is insufficient for blood cultures. PTS: 1

DIF:

TCEogSnTitiBveALNeKveSl:E1LLER.COM

REF: p. 484

12. Which of the following specimen types for culture can be collected in a clean, nonsterile

container? a. Urine b. Stool c. Cerebrospinal fluid d. Blood ANS: B

Urine, cerebrospinal fluid, and blood are all sterile body sites and must be collected in sterile containers. Stool specimens for culture (and parasite detection) can be collected in clean, dry waterproof containers. PTS: 1

DIF:

Cognitive Level: 1

REF: p. 520

13. A growth of bacteria that began as one parent bacterium and divided multiple times to become

a visible aggregate is called a: a. culture. b. pure culture. c. slant culture. d. colony. ANS: D


A colony is a group of many bacteria that began as one parent bacterium that multiplied many times to produce a visible aggregate. Bacteria are microscopic entities that cannot be seen by the eye individually. However, when they multiply they become visible as colonies, in which numerous bacteria are present together. A pure culture refers to the whole agar plate (many colonies) that represents one species of bacteria. A slant culture is agar in a test tube that can be used for purposes such as biochemical testing or storage of bacteria over periods of time. PTS: 1

DIF:

Cognitive Level: 1

REF: p. 486

14. Routine microbiology laboratories have incubators that are set at all of the following

temperatures except: a. 30° C. b. 35° C. c. 42° C. d. 50° C. ANS: D

Routine cultures are incubated at 35° C ± 2 degrees reflecting the temperature at which most pathogens grow optimally (body temperature). Fungi are cultivated at 30° C, and Campylobacter spp. are best recovered by incubation at 42° C. PTS: 1

DIF:

Cognitive Level: 1

REF: p. 488

15. Which of the following is a differential stain used routinely in microbiology laboratories? a. Methylene blue b. Flagella stain c. Gram stain d. Capsule stain ANS: C

Gram staining is the most used differential stain in microbiology because it is able to differentiate between two groups of bacteria based on their reaction, gram positive or gram negative. Methylene blue is a simple stain that yields limited information about bacteria such as size and shape. Capsule and flagella stains are specialized stains to detect those particular structures in certain bacteria. PTS: 1

DIF:

Cognitive Level: 1

REF: p. 492

16. The prefix strepto- refers to bacteria in which of the following groupings? a. Clusters b. Pairs c. Chains d. Singular ANS: C

The prefix strepto- refers to bacteria in chains. Diplo- refers to bacteria in pairs, and staphyloto bacteria in clusters. Bacteria that are present singularly are observed as separate from one another. PTS: 1

DIF:

Cognitive Level: 1

REF: p. 492


17. Why do gram-positive bacterial cells retain the crystal violet–iodine complex when the

decolorizer is added? a. Gram-positive cells have an outer membrane that traps the complex. b. Gram-positive bacterial cells have a thinner peptidoglycan layer with teichoic acid cross-linkages. c. Gram-positive bacterial cells have a thicker peptidoglycan layer with teichoic acid cross-linkages. d. Gram-positive bacterial cells have a thick capsule layer that traps the complex. ANS: C

Gram-positive bacterial cells have a thick peptidoglycan layer with many teichoic acid cross-linkages. This is referred to as the cell wall. The thickness of the peptidoglycan layer with the cross-linkages aids in retention of the crystal violet–iodine complex. Gram-negative bacterial cells have a thinner cell wall (peptidoglycan) with an outer membrane similar to the cellular membrane. Although there is a special stain for capsules, the mechanism for retention of this complex is not related to the capsule. PTS: 1

DIF:

Cognitive Level: 1

REF: p. 493

18. What is the color of gram-negative bacterial cells after the decolorization step? a. Purple/blue b. Red/pink c. Iodine yellow d. Colorless ANS: D

Gram-negative bacterial cells do not have a thick peptidoglycan layer with teichoic acid ThEeS cross-linkages; therefore, w nT thB eA deNcK olSoE rizLinLgEaRg. enCt O (aMcetone, alcohol, or a combination of both) is added the crystal violet–iodine complex is washed out. The bacteria will be colorless at this point and when counterstained with safranin will be red/pink in color. PTS: 1

DIF:

Cognitive Level: 1

REF: p. 494

19. Which of the following is the “cold” method for staining acid-fast bacteria? a. Ziehl-Neelsen b. Kinyoun c. Fluorochrome d. All of the above ANS: B

The Kinyoun method uses phenol to replace heat; therefore, it is referred to as the “cold” method. Phenol and heat are used to facilitate penetration of the stain into acid-fast bacteria. They have a thick mycolic acid cell wall that makes it difficult for dyes to penetrate. PTS: 1

DIF:

Cognitive Level: 1

REF: p. 494

20. Which of the following bacteria are typically stained using the acid-fast technique? a. Mycobacteria b. Staphylococci c. Streptococci d. Escherichia coli


ANS: A

Mycobacteria have cell walls that contain a thick layer of mycolic acid. This trait is responsible for the ability to resist decolorization with acid-alcohol. Other bacteria will easily lose the primary dye (carbol fuchsin or in the fluorochrome method, auramine and rhodamine) and appear colorless at this step, and when counterstained they will all appear the same color. If methylene blue is used as the counterstain in the Kinyoun method, they will appear blue. The acid-fast bacteria will appear reddish-pink, the color of the primary dye carbol fuchsin. PTS: 1

DIF:

Cognitive Level: 1

REF: p. 494

21. Which type of microscope is used to observe acid-fast bacteria when they are stained with

auramine and rhodamine? a. Brightfield b. Electron c. Fluorescent d. All of the above ANS: C

Auramine and rhodamine are fluorochromes (dyes that absorb ultraviolet light and emit light of higher wavelengths), and the fluorescent microscope is used to detect the emitted light. This method is more rapid than the Kinyoun or Ziehl-Neelsen because the slides can be screened using lower magnifications (10 and 40). The acid-fast bacteria will fluoresce and stand out as a bright yellow/orange color. In the other two methods a brightfield microscope is used and the oil immersion objective (100) is needed to observe the bacteria. PTS: 1

DIF:

Cognitive Level: 1

REF: p. 494

TESTBANKSELLER.COM

22. Growth of several colonies originating from a single colony, and thus a single cell, is known

as a: a. subculture. b. primary culture. c. mixed culture. d. pure culture. ANS: D

This is the definition of a pure culture. In microbiology, pure cultures are necessary for accurate identification and susceptibility testing. Mixed cultures have more than one type (species) of bacteria present. Primary culture refers to the plates that are inoculated with the specimen. When these plates are incubated overnight bacterial populations can be observed. Depending on the infection, one or more species can be present. If more than one species is present, usually a subculture is performed to isolate each species on different agar plates in pure culture. PTS: 1

DIF:

Cognitive Level: 1

23. Which of the following refers to colony morphology? a. Cocci in chains b. Shiny, wet, or mucoid c. Yellow, dry, rough d. Both b and c

REF: p. 494


ANS: D

Options b and c are descriptions used for a colony or macroscopic morphology. The terms describe the appearance of the colonies as they grow on agar. These colony characteristics are helpful in beginning the identification process. Option a describes microscopic morphology. Both are important for identification but experienced microbiologists can quickly work to identify bacteria from typical colony morphology. PTS: 1

DIF:

Cognitive Level: 1

REF: p. 495

24. Which of the following is an organism that can grow in the presence or absence of oxygen? a. Aerobe b. Anaerobe c. Facultative anaerobe d. Microaerophile ANS: C

Facultative anaerobic organisms can grow in the presence or absence of oxygen. Aerobes grow in the presence of oxygen, and anaerobes cannot grow in the presence of oxygen. Microaerophiles grow best in a reduced oxygen environment. PTS: 1

DIF:

Cognitive Level: 1

REF: p. 496

25. Where do strict aerobes grow in thioglycolate broth? a. At the top b. In the middle c. At the bottom d. Throughout the broth ANS: A

Strict aerobes grow at the top where oxygen is available. The thioglycolate and a small amount (0.075%) of agar serve to create an anaerobic environment in the bottom of the tube; therefore, anaerobic organisms are found there. PTS: 1

DIF:

Cognitive Level: 1

REF: p. 497

26. Media that have dyes, antibiotics, or other chemical compounds added to inhibit certain

bacteria while allowing others to grow is called a. enrichment b. supportive c. selective d. differential

media.

ANS: C

Selective media are composed of certain ingredients such as dyes or antibiotics to “select” for one group of organisms while inhibiting other types. Enrichment media allow one bacterial pathogen to grow using nutrients for growth of that pathogen. Supportive media contain nutrients that allow nonfastidious organisms to grow at a normal rate. Differential media contain factors that aid in differentiating bacteria based on recognizable characteristics. This type of media may contain a carbohydrate such as lactose with an indicator to achieve this purpose. PTS: 1

DIF:

Cognitive Level: 1

REF: p. 497


27. Media that contain factors such as lactose with an indicator present to give distinctive,

recognizable characteristics is called a. enrichment b. supportive c. selective d. differential

media.

ANS: D

Differential media contain factors that aid in differentiating bacteria based on recognizable characteristics. This type of media may contain a carbohydrate such as lactose with an indicator to achieve this purpose. Selective media are composed of certain ingredients such as dyes or antibiotics to “select” for one group of organisms while inhibiting other types. Enrichment media allow one bacterial pathogen to grow using nutrients for growth of that pathogen. Supportive media contain nutrients that allow nonfastidious organisms to grow at a normal rate. PTS: 1

DIF:

Cognitive Level: 1

REF: p. 498

28. Chocolate agar contains all of the following except: a. antibiotics. b. X factor. c. V factor. d. lysed blood. ANS: A

Chocolate agar is an enrichment media that promotes the growth of fastidious bacteria. To ES achieve this purpose X andTV faT ctB orAsNaK reSaE ddLeL dE toRg.eC ntO lyMlyse red blood cells. This gives a richer medium than blood agar because more factors are available for growth. No antibiotics are added to this media. PTS: 1

DIF:

Cognitive Level: 1

REF: p. 498

29. Which of the following is a selective and differential medium for the recovery of

gram-positive organisms? a. Columbia colistin-nalidixic acid agar with blood (CNA) b. Eosin-methylene blue agar (EMB) c. MacConkey agar (MAC) d. Xylose-lysine desoxycholate agar (XLD) ANS: A

CNA agar is selective because it contains colistin and nalidixic agar (antibiotics) to inhibit gram-negative organisms. It also contains 5% sheep blood for differentiation of hemolytic organisms. EMB and MAC are selective and differential, but they inhibit gram-positive organisms. XLD is selective and differential also, but for the recovery of Salmonella and Shigella spp. PTS: 1

DIF:

Cognitive Level: 1

REF: p. 498

30. If a microbiologist wants to add a selective and enriched medium for the recovery of Neisseria

gonorrhoeae from genital specimens, which of the following would be appropriate?


a. b. c. d.

Chocolate agar (CHOC) Sheep blood agar (SBA or BA) Modified Thayer-Martin agar (MTM) Phenylethyl alcohol agar (PEA)

ANS: C

MTM contains hemoglobin, a supplement with X and V factors, and vitamins to grow pathogenic Neisseria spp. It is a modification of chocolate agar and contains several antimicrobial agents to inhibit the growth of normal flora and fungi from the genital tract. SBA and CHOC do not contain antimicrobials to inhibit genital flora. PEA is selective for gram-positive organisms by inhibiting gram-negative organisms. PTS: 1

DIF:

Cognitive Level: 2

REF: p. 500

31. A microbiologist observes colonies on Hektoen enteric agar (HE) that appear green. Which of

the following organisms should the microbiologist suspect? a. Salmonella spp. b. Escherichia coli c. Proteus spp. d. Shigella spp. ANS: D

HE contains bile salts, two indicators, bromthymol blue, acid fuchsin, and lactose. Gram-positive organisms are inhibited because of the addition of bile salts and dyes. Most enteric nonpathogens will ferment lactose and appear yellow. Therefore this agar is selective and differential for the growth of enteric pathogens, Salmonella and Shigella spp., because they do not ferment lactose. There are additional components that aid in the detection of TBthAioNsKulSfaEteLaLnE Salmonella spp. They are sToE diS um dR fe. rrC icOaM mmonium citrate; when an organism that produces hydrogen sulfide is present, the organism will form a black precipitate. Therefore, colonies of Salmonella (hydrogen sulfide producer) will appear green or transparent with black centers. Shigella spp. do not produce hydrogen sulfide and will appear green (the color of the media) or transparent. PTS: 1

DIF:

Cognitive Level: 2

REF: p. 498

32. Which of the following tests identifies Staphylococcus aureus? a. PYR and catalase b. Catalase and coagulase c. Bile solubility and optochin d. Indole and oxidase ANS: B

Staphylococcus aureus is catalase positive and coagulase positive. The catalase test separates the staphylococci (positive) from the streptococci (negative), and the coagulase test identifies the pathogen Staphylococcus aureus (positive) from the other staphylococci (most are negative). The PYR test can identify group A beta-hemolytic streptococci and Enterococcus spp. because they both have positive reactions in this test. Bile solubility and optochin tests are used to identify Streptococcus pneumoniae because this organism has positive reactions in both. Indole and oxidase tests are mostly used for gram-negative organisms. PTS: 1

DIF:

Cognitive Level: 1

REF: p. 503


33. Which of the following is the appearance of a positive catalase test? a. Fibrin clot b. Bubbles of oxygen c. Visible clumping d. Blue-green color ANS: B

The enzyme catalase breaks down hydrogen peroxide to water and oxygen. When this reaction occurs the oxygen can be observed as bubbles emanating from the area on the slide where the organism was placed. Therefore, a positive test is the presence of bubbles of oxygen. A fibrin clot is a positive reaction for the unbound coagulase test, and visible clumping is observed in the bound coagulase test for clumping factor. A blue-green color development is observed when 1% paradimethylaminocinnamaldehyde is the indicator used in the indole test. PTS: 1

DIF:

Cognitive Level: 1

REF: p. 503

34. The Gram stain is used to differentiate bacteria that have similar morphologic features. The

Gram stain method uses different reagents, including: a. Gram iodine. b. decolorization. c. a counterstain. d. all of the above. ANS: D

The Gram stain method uses four different reagents: 1) the primary stain crystal violet; 2) Gram iodine, which serves as a mordant; 3) decolorization with an alcohol-acetone solution; and 4) a counterstain, safraTnE inS . TBANKSELLER.COM PTS: 1

DIF:

Cognitive Level: 1

REF: p. 493

35. Which of the following is a rapid method to screen sputum specimens for the yeast

Cryptococcus neoformans? a. Coagulase b. Urease c. Indole d. Oxidase ANS: B

Cryptococcus neoformans possesses urease, an enzyme that hydrolyzes urea, releasing ammonia as an end product. The production of ammonia changes the pH to alkaline and is detected in the medium by a pH indicator (phenol red) change from yellow to magenta. The coagulase test separates the pathogen Staphylococcus aureus from the other staphylococci. Indole and oxidase tests are mostly used for gram-negative organisms. PTS: 1

DIF:

Cognitive Level: 1

REF: p. 504

36. The microbiologist uses TSI and LIA agars to rule out stool pathogens, Salmonella and

Shigella spp. The reactions observed are TSI: K/A, H2S+ and LIA: R/A. What should the technologist do next? a. Report Salmonella spp. recovered


b. Report Shigella spp. recovered c. Report normal stool flora d. Repeat all testing ANS: C

Although the hydrogen sulfide is usually suspicious of Salmonella the technologist would quickly look at the LIA: R/A and realize that this organism is lysine deaminase positive effectively eliminating both stool pathogens (both are lysine deaminase negative). The organism could possibly be a Proteus spp. (normal stool flora) and, as long as there are no other suspicious colonies, normal stool flora can be reported. PTS: 1

DIF:

Cognitive Level: 2

REF: pp. 503-504

37. The technologist uses TSI and LIA agars to rule out stool pathogens, Salmonella and Shigella

spp. The reactions observed are TSI: K/A H2S+ and LIA: K/K. What should the technologist do next? a. Report Salmonella spp. recovered b. Report Shigella spp. recovered c. Do further biochemical and serological testing to confirm Salmonella spp. d. Repeat all testing ANS: C

The reactions are suspicious of Salmonella spp., and further biochemical and serological testing should be done to confirm the TSI and LIA screening. The TSI: K/A indicates a glucose fermenter, lactose and sucrose nonfermenter, and an H2S producer. The LIA: K/K indicates a negative lysine deaminase and a positive lysine decarboxylase. Both are suspicious of Salmonella. PTS: 1

DIF:

Cognitive Level: 2

REF: pp. 503-504

38. Which of the following media contain sodium thiosulfate and ferric ammonium citrate for the

detection of hydrogen sulfide production? a. Triple sugar iron agar (TSI) b. Lysine iron agar (LIA) c. Xylose-lysine-desoxycholate agar (XLD) d. All of the above ANS: D

All of the above media contain sodium thiosulfate and ferric ammonium citrate for the detection of H2S. Salmonella is a pathogenic enteric bacterium that causes gastroenteritis therefore many different types of media are made specifically for the detection of this organism. These media also detect Shigella spp., another stool pathogen. PTS: 1

DIF:

Cognitive Level: 1

REF: p. 499 | p. 501

39. A clean-catch midstream urine specimen is sent to the laboratory for culture. The results of

the culture yield three organisms present, with no predominating organism. What type of workup should the technologist do? a. Complete workup of all organisms present b. No workup because of possible contamination c. Workup one of the organisms


d. Workup two of the organisms ANS: B

Clean-catch midstream urine specimens can easily be contaminated with normal flora from the area around the urethra. The contamination can be skin flora, vaginal flora, or stool flora. It is important to collect specimens in the manner described by the health care staff so that contamination is limited. In many cases, contaminated cultures need to be repeated, making diagnosis possibly lengthier. PTS: 1

DIF:

Cognitive Level: 1

REF: p. 505

40. A positive leukocyte esterase and nitrite test (urine rapid-screening test strips) indicates: a. most likely the patient has a UTI. b. the patient definitely does not have a UTI. c. a new specimen should be collected because these tests indicate contamination. d. the test strips are not reacting properly, and the quality control should be

rechecked. ANS: A

The leukocyte esterase and the nitrite tests are two indicators of bacteria present in urine specimens. This is a rapid method and is valuable in detecting UTIs. Leukocyte esterase is an enzyme present in the granules of neutrophils. Generally, neutrophils are increased in number in UTIs to fight the infection; therefore, it is a good indicator. Common organisms that cause UTIs such as Escherichia coli, Proteus, Klebsiella, Enterobacter, and Pseudomonas reduce nitrate to nitrite. The nitrite test then is a good indicator of infection also. If these tests are negative, it does not rule out an infection because certain organisms do not have the enzyme nitrate reductase to reduce nitrate to nitrite, and not all patients respond with the same neutrophil response. PTS: 1

DIF:

Cognitive Level: 1

REF: p. 506

41. If a urine culture was planted using a 0.001-mL loop and 75 colonies were observed, what is

the colony-forming units per millimeter of urine (CFU/mL)? a. 75 CFU/mL b. 750 CFU/mL c. 7500 CFU/mL d. 75,000 CFU/mL ANS: D

A 0.001-mL calibrated loop has a dilution factor of 1000. Therefore, the number of colonies observed is multiplied by 1000 to give the final CFU/mL. In this case, 75  1000 = 75,000 CFU/mL. PTS: 1

DIF:

Cognitive Level: 2

REF: p. 506

42. Rapid screening of throat specimens (nonculture method) is done to detect which of the

following pathogens? a. Streptococcus pneumoniae b. Escherichia coli c. Group A -hemolytic Streptococcus d. Staphylococcus aureus


ANS: C

Group A -hemolytic Streptococcus is the etiologic agent of “strep” throat. It is a pathogen that must be treated promptly, because of possible severe complications of infection such as scarlet fever or rheumatic fever. The genus and species designation is Streptococcus pyogenes. The Lancefield grouping is done by testing the organism for specific cell wall carbohydrate antigens. In the case of “strep” throat it is the group A antigen. PTS: 1

DIF:

Cognitive Level: 1

REF: p. 507

43. Which of the following describes alpha hemolysis on sheep blood agar? a. Complete clearing around the colonies of the red blood cells in the medium b. Partial hemolysis of the red blood cells in the medium appearing as a green

discoloration of the medium c. Partial hemolysis of the red blood cells in the medium appearing as a slight

clearing around the colonies d. No hemolysis of the red blood cells in the medium; therefore no change in the

color of the medium ANS: B

Alpha hemolysis refers to partial hemolysis of the red blood cells in the medium that appears as a green discoloration of the medium. Complete hemolysis is a distinct clearing of the red blood cells around the colonies and is termed beta hemolysis. Gamma hemolysis is no hemolysis; therefore, no change in the color of the medium is observed. PTS: 1

DIF:

Cognitive Level: 1

REF: p. 507

44. Which of the following tests can be used to identify group A -hemolytic Streptococcus? TESTBANKSELLER.COM a. PYR test b. Bacitracin disk c. Optochin d. Both A and B ANS: D

Group A -hemolytic streptococci are the only -hemolytic streptococci that are PYR positive. Other organisms are PYR positive but, in the -hemolytic streptococci, only group A is positive. Also, group A -hemolytic streptococci are susceptible to bacitracin (0.04-unit) and any zone of inhibition is positive for group A -hemolytic streptococci. PTS: 1

DIF:

Cognitive Level: 1

REF: p. 508

45. Which of the following is done to identify Streptococcus pneumoniae? a. Ethylhydrocupreine hydrochloride b. Optochin c. Bile solubility d. Desoxycholate ANS: D


Optochin is ethylhydrocupreine hydrochloride and is used to identify Streptococcus pneumoniae by placement of a disk impregnated with optochin on a pure culture of the suspected organism. A zone of inhibition greater than 14 mm indicates S. pneumoniae. The bile solubility test is performed with desoxycholate and is also used to identify S. pneumoniae. When the reagent is applied to colonies that are S. pneumoniae, after a period of 30 minutes the colonies will dissolve. Other -hemolytic streptococci will not dissolve in the bile salt. PTS: 1

DIF:

Cognitive Level: 1

REF: p. 508

46. Which of the following is one cause of bacterial vaginosis and is best diagnosed by the

presence of microscopic “clue cells”? a. Gardnerella vaginalis b. Neisseria gonorrhoeae c. Chlamydia trachomatis d. Trichomonas vaginalis ANS: A

“Clue cells” are exfoliated vaginal epithelial cells that are covered with tiny gram-variable bacilli and coccobacilli indicative of bacterial vaginosis (BV). Gardnerella vaginalis is one of the organisms that cause BV. BV is a polymicrobial condition resulting from the disruption of the normal flora (Lactobacillus spp.) in the vagina. Present besides G. vaginalis (gram-variable bacilli) can be anaerobic gram-negative bacteria and Mobiluncus spp. (anaerobic gram-positive curved bacilli). PTS: 1

DIF:

Cognitive Level: 1

REF: p. 508

47. Which of the following is an obligate intracellular pathogen that is a prevalent sexually

transmitted disease? a. Gardnerella vaginalis b. Neisseria gonorrhoeae c. Chlamydia trachomatis d. Trichomonas vaginalis ANS: C

Chlamydia trachomatis has become a prevalent sexually transmitted disease. This organism must (obligate) reside inside (intracellular) epithelial cells to survive. Specimens for analysis must contain these cells and not purulent exudate. A two-swab system can be used wherein a larger swab is used to remove the exudate and a second swab is used to collect the specimen. PTS: 1

DIF:

Cognitive Level: 1

REF: p. 508

48. Which of the following organisms can be routinely detected by culture and nucleic acid

testing? a. Gardnerella vaginalis b. Neisseria gonorrhoeae c. Chlamydia trachomatis d. Both b and c ANS: D


Both Neisseria gonorrhoeae and Chlamydia trachomatis can be detected by nucleic acid testing and culture. Both organisms are processed and tested concurrently in the nucleic acid procedure. In this way, two sexually transmitted diseases can be tested for in the same time frame, same analyzer, and results are available at the same time. Many laboratories have adopted these methods. Also viable organisms are not needed for this procedure. These two organisms are fastidious and must be collected, processed, and grown in culture with careful guidelines. Because it is an obligate intracellular pathogen, Chlamydia trachomatis can only be grown in cell culture (McCoy cell line typically). Neisseria gonorrhoeae can be grown in artificial culture on agar such as chocolate and modified Thayer-Martin. The latter is a selective media for pathogenic Neisseria spp. PTS: 1

DIF:

Cognitive Level: 1

REF: p. 509

49. A progressive syndrome with symptoms of fever, chills, low blood pressure, respiratory

distress, and disseminated intravascular coagulation is called: a. bacteremia. b. septic shock. c. transient bacteremia. d. exotoxin. ANS: B

Septic shock is a serious progressive syndrome in which the patient is septic (bacteria or toxin produced by the bacteria in the blood is causing harm) and the bacteria release exotoxins (toxins released to the surrounding environment) or endotoxins (part of the cell wall of gram-negative bacteria). Particularly in the case of endotoxins, septic shock can occur and progress to a life-threatening situation. PTS: 1

DIF:

TCEogSnTitiBveALNeKveSl:E1LLER.COM

REF: p. 510

50. A common gram-negative organism found in blood is: a. Staphylococcus aureus. b. Escherichia coli. c. coagulase-negative staphylococci. d. Streptococcus pneumoniae. ANS: B

Escherichia coli is a gram-negative bacillus found in blood cultures commonly. Staphylococcus aureus, Streptococcus pneumoniae, and coagulase-negative staphylococci are gram-positive organisms and can also be found in positive blood cultures. PTS: 1

DIF:

Cognitive Level: 1

REF: p. 510

51. Which of the following bacteremias can result from possible wound abscesses and are present

for a time in the blood, followed by a period with no bacteria? a. Intermittent bacteremia b. Transient bacteremia c. Septicemia d. Continuous bacteremia ANS: A


Intermittent bacteremias are present for a period of time, followed by a period of time with no bacteria. Transient bacteremia can occur incidentally after dental or colonoscopic procedures. In this case, bacteria may be present in the blood only for a brief period of time after the procedure. A continuous bacteremia is present in the blood consistently and the organisms are from an intravascular source. A septicemia indicates bacteria or toxin in the blood, and harm to the patient is a result of the infection. PTS: 1

DIF:

Cognitive Level: 1

REF: p. 511

52. Which of the following is false about blood cultures? a. Blood/medium ratio 1:2 to 1:5 b. Blood/medium ratio 1:5 to 1:10 c. Sodium polyanethol sulfonate (anticoagulant) d. Broth medium ANS: A

The ratio is 1:5 to 1:10 for most laboratories. The anticoagulant sodium polyanethol sulfonate (SPS) is used because it typically does not harm the bacteria if the percentage is kept at 0.025 to 0.05. Fastidious organisms can be affected by SPS. Broth is the medium used and can include trypticase soy or peptone supplemented types. PTS: 1

DIF:

Cognitive Level: 1

REF: p. 511

53. Automated methods of culturing blood have replaced traditional methods in most laboratories

because of: a. more rapid detection time for many pathogens. b. monitoring of growth without visual inspection or subculture of the bottles. c. the ability to use the reT poErS t iT nB coAnNjuKnS ctE ioL nLwEitR h. aC coOmMputerized laboratory management information system. d. all of the above. ANS: D

Most microbiology laboratories use the automated method of culturing blood for: 1. More rapid detection time for many pathogens 2. Monitoring of growth without visual inspection or subculture of the bottles 3. The ability to use the report in conjunction with a computerized laboratory management information system. The traditional method is cumbersome and time consuming; therefore, the automated method has made a very important part of microbiology easier and faster. PTS: 1

DIF:

Cognitive Level: 1

REF: pp. 510-511

54. The lowest concentration of an antimicrobial agent that will visibly inhibit the growth of the

organism being tested is called: a. bacteriostatic. b. bactericidal. c. minimal inhibitory concentration (MIC). d. minimum bactericidal concentration (MBC). ANS: C


The MIC is the lowest concentration of an antimicrobial agent that will visibly inhibit the growth of the organism. This is one consideration used in selecting a specific antimicrobial agent. The MBC is a 99.9% reduction in the growth of a bacterial population. The term bacteriostatic refers to inhibition of bacterial growth by an agent, and the ability of an agent to kill is bactericidal. The question is referring to lowest concentration that will visibly inhibit the growth of the organism; therefore, MIC is the correct answer. PTS: 1

DIF:

Cognitive Level: 1

REF: p. 513

55. Which of the following is an agar disk diffusion method of testing for antimicrobial

susceptibility? a. Microdilution b. Macrodilution c. Kirby-Bauer method d. MBC ANS: C

The Kirby-Bauer method is a disk diffusion method used in microbiology laboratories to test for susceptibility and resistance. It is a standardized method of inoculating agar plates with the organism in question and placing antibiotic-impregnated disks on the inoculum. Dilution methods have been adapted for automated methods and are used in most laboratories. PTS: 1

DIF:

Cognitive Level: 1

REF: p. 514

56. How do you interpret no zone of inhibition around an antibiotic disk in the disk diffusion

method? a. Susceptible b. Resistant c. Intermediate d. Repeat testing; there should always be a zone of inhibition. ANS: B

If there is no zone of inhibition, then complete resistance of the organism to the antibiotic in question is the correct interpretation. The zone of inhibition is measured in millimeters and is compared with a table of breakpoint values for each antimicrobial to determine whether the organism is susceptible, intermediate, or resistant to that particular agent. PTS: 1

DIF:

Cognitive Level: 2

REF: p. 514

57. A microbiologist is preparing a Mueller-Hinton agar for disk diffusion testing and has

inoculated the plate with the appropriate standardized inoculum. The microbiologist is interrupted by a phone call from a physician that takes 20 minutes to track the requested information. How should the technologist proceed with the disk diffusion testing? a. Continue as planned and place the disks on the agar plate. b. Continue as planned, but add quality control testing to ensure proper results. c. Discard the plate and, with the same inoculum, swab a new Mueller-Hinton agar. d. Discard the plate and the inoculum and start anew. ANS: D


The disk diffusion method is a standardized procedure. Once the inoculum is prepared it must be placed on the agar within 15 minutes. The reason for this is that bacteria can multiply and the inoculum would be too heavy and yield erroneous results. Also, once the inoculum is placed on the agar the disks must be placed on the agar within 15 minutes. Again, erroneous zones of inhibition could result because the bacteria are multiplying. Therefore, the technologist should begin anew for accurate results. PTS: 1

DIF:

Cognitive Level: 2

REF: p. 515

58. All of the following are characteristics of fungi except: a. cell wall made of peptidoglycan. b. hyphae. c. “fuzzy” or woolly appearance. d. eukaryotic. ANS: A

Fungi are eukaryotic cells and have membrane-bound organelles such as mitochondria and a nucleus. Fungi can be yeasts or molds. Most yeasts appear as creamy or pasty on Sabouraud media, and molds can have a “fuzzy” or woolly appearance on the same media. Hyphae are tubelike projections that give molds their characteristic appearance. Many hyphae are referred to as mycelia. PTS: 1

DIF:

Cognitive Level: 1

REF: p. 516

59. A fungal infection that affects the deeper layers of the skin, including muscle and connective

tissue, is called a. superficial b. cutaneous c. subcutaneous d. systemic

mycosis.

ANS: C

A superficial mycosis is confined to the outermost skin layer and hair. A cutaneous mycosis is a fungal infection of the keratinized layer of the skin, hair, or nails. A subcutaneous mycosis is a fungal infection that affects the deeper layers of the skin, including muscle and connective tissue. A systemic mycosis affects the lungs and can disseminate to internal organs or deep tissues of the body. PTS: 1

DIF:

Cognitive Level: 1

REF: p. 517

60. Which of the following is the least desirable specimen for fungal cultures? a. Hair b. Skin c. Sterile body fluids d. Swabs ANS: D

Swabs are the least desirable specimen for fungal cultures. These organisms can be easily missed if a surface swab is submitted; therefore, hair, skin, nails, urine, blood, tissue, and sterile body fluids are preferred.


PTS: 1

DIF:

Cognitive Level: 1

REF: p. 517

61. Which of the following organisms can be detected by India ink staining and appear as budding

yeast surrounded by a large clear area (capsule)? a. Nocardia spp. b. Candida albicans c. Cryptococcus neoformans d. Candida spp. ANS: C

Cryptococcus neoformans is an encapsulated yeast. Typically when India ink is added to cerebrospinal fluid with this organism present, budding yeast can be seen with a large clear area surrounding them. This clear area is due to the capsule. This is a negative staining method, because the organism does not take up the dye. PTS: 1

DIF:

Cognitive Level: 1

REF: p. 518

62. Which of the following procedures can identify Candida albicans? a. India ink stain b. Germ-tube test c. Gram stain d. Acid-fast stain ANS: B

The germ-tube test is used to identify Candida albicans. It is easily performed, and the results are available within 3 hours. Germ tubes are the beginning of true hyphae and appear as appendages that are half the width and three to four times the length of the yeast cell from which it arises. Candida alTbE icS anTsBaA lsoNK haSs EpL seL udEoRh. ypChO aeM, which are elongated buds that do not separate. They can appear similar to true hyphae. The India ink stain is used to detect Cryptococcus neoformans. The acid-fast stain can be used to aid in the identification of Nocardia spp., and the Gram stain can detect yeast but cannot aid in identifying which yeast is the cause of the infection. PTS: 1

DIF:

Cognitive Level: 1

REF: p. 519

63. Which of the following are the nematodes? a. Roundworms b. Flatworms c. Tapeworms d. Flukes ANS: A

The nematodes are roundworms and belong to the phylum Nematoda. The flatworms are the trematodes (flukes) and cestodes (tapeworms), and belong to the phylum Platyhelminthes. PTS: 1

DIF:

Cognitive Level: 1

REF: p. 519

64. Which of the following is the feeding and reproductive stage of protozoa and what

consistency of stool would you usually find them in? a. Cysts; loose or watery stool b. Cysts; formed stool


c. Trophozoites; loose or watery stool d. Trophozoites; formed stool ANS: C

Trophozoites are the actively feeding and reproductive form of protozoa. They are typically found in stools that are loose or watery. The cyst stage is the inactive stage. Finding protozoan cysts usually indicates that the infection is in an inactive state, whereas finding trophozoites usually indicates an active infectious disease. PTS: 1

DIF:

Cognitive Level: 1

REF: p. 519

65. What is the recommended protocol for collection of stool specimens for ova and parasite

(O&P) detection? a. Three stool samples 1 or 2 days apart, all within a 10-day period b. Two stool samples 1 or 2 days apart, all within a 5-day period c. Three stool samples 1 or 2 days apart, all within a 5-day period d. Two stool samples 1 or 2 days apart, all within a 10-day period ANS: A

The recommended protocol is to collect three stool samples 1 or 2 days apart, but all within a 10-day period, to provide optimal detection of intestinal parasites. PTS: 1

DIF:

Cognitive Level: 1

REF: p. 520

66. Which of the following specimens is appropriate for detection of Plasmodium spp.? a. Stool b. Wound c. Blood d. Urine ANS: C

Plasmodium spp. are malarial parasites. They invade red blood cells; therefore, blood smears, thick and thin, are appropriate. The thick smear increases the sensitivity but can be difficult to read. The thin smear is used to identify the species of the malarial parasite. PTS: 1

DIF: Cognitive Level: 1

REF: p. 520

67. Motile trophozoites found microscopically in fresh urine or fresh genital secretions is

diagnostic for which of the following parasites? a. Plasmodium spp. b. Trichomonas vaginalis c. Enterobius vermicularis d. Pneumocystis jiroveci ANS: B

Trichomonas vaginalis is a flagellate that can inhabit the urogenital system of both males and females. It is usually considered a sexually transmitted disease and the cause of vaginitis, urethritis, and prostatitis. It is easily identified in fresh urine or fresh genital secretions because it is highly motile. If the specimen is old, then the trophozoite may not be motile and appear “rounded up” and resemble a white blood cell. Therefore only fresh specimens should be analyzed.


PTS: 1

DIF:

Cognitive Level: 1

REF: p. 520

68. The cellophane tape method is used to detect which of the following? a. Plasmodium spp. b. Trichomonas vaginalis c. Enterobius vermicularis d. Pneumocystis jiroveci ANS: C

The cellophane tape (clear, not frosted) method is used to detect Enterobius vermicularis. It is a common parasite called the pinworm. It is a roundworm whose adult female migrates during the night, depositing her eggs in the perianal region; thus a fecal sample is not the optimal specimen because the eggs may not be observed in the feces. PTS: 1

DIF:

Cognitive Level: 1

REF: p. 522

CASE 1. Interpret the following case study:

A 25-year-old woman presents to the emergency department with vaginal discomfort. The physician ordered a hanging drop (wet preparation for Trichomonas vaginalis), KOH preparation, Gram stain, culture, and Gen-Probe. The results are: Hanging drop: no motile trichomonas seen, clue cells present KOH preparation: no fungal elements seen Gram stain: Many gram-variable bacilli Few squamous epithelia Rare amount gram-positive bacilli Genital culture: pending Gen-Probe: negative for Neisseria gonorrhoeae and negative for Chlamydia trachomatis ANS:

Gardnerella vaginalis causing bacterial vaginosis (BV) The hanging drop is a procedure to detect motile Trichomonas vaginalis. The procedure is performed by placing a drop of fresh genital secretions under the microscope. “Clue” cells can be detected and are squamous epithelia covered with bacteria. The edges of the squamous epithelia are obscured because of the bacteria present. This is an indicator of bacterial vaginosis. The KOH preparation is a microscopic procedure to detect fungi and in particular in vaginal specimens, yeast. The Gram stain reveals many gram-variable bacilli and squamous epithelia. Also noted was rare amount of gram-positive bacilli, reflecting a decrease in normal flora of the vaginal tract (Lactobacillus). Bacterial vaginosis is diagnosed by smears and not culture; therefore, these results all point to BV. Also the Gen-Probe was performed and was negative for Neisseria gonorrhoeae and negative for Chlamydia trachomatis, further strengthening the diagnosis of BV. PTS: 1

DIF:

Cognitive Level: 3

REF: p. 508

2. Interpret the following Gram stain of urethral discharge from a male:

Many polymorphonuclear leukocytes (PMNs) Few gram-negative intracellular diplococci


ANS:

Neisseria gonorrhoeae Gram-stained smears from urethral discharge are examined for the presence of PMNs and gram-negative intracellular diplococci (within PMNs) indicative of gonococci in men only. Vaginal flora contaminates smears from women and, if done, the positive smear is only presumptive evidence of gonorrhea. PTS: 1

DIF:

Cognitive Level: 3

REF: p. 510


Chapter 16: Immunology and Serology Turgeon: Linné & Ringsrud’s Clinical Laboratory Science, 7th Edition MULTIPLE CHOICE 1. A fundamental principle of immunology is recognition of: a. body cells. b. molecules. c. self from nonself. d. antibodies. ANS: C

Immunology is defined as the study of the molecules, cells, organs, and systems responsible for the recognition and disposal of nonself substances; the response and interaction of body components and related interactions; and the way that the immune system can be manipulated to protect against or treat diseases. The function of the immune system is to recognize self from nonself and to defend the body against nonself. PTS: 1

DIF:

Cognitive Level: 2

REF: p. 541

2. Serology is the study of: a. antigens. b. antibodies. c. chemical reactions. d. infectious diseases. ANS: A

Serology is a division of immunology that specializes in laboratory detection and measurement of specific antibodies that develop in the blood during a response to exposure to a disease-producing antigen. PTS: 1

DIF:

Cognitive Level: 2

REF: p. 541

3. The first line of body defense is: a. unbroken skin. b. normal biota. c. body fluids (e.g., tears, saliva). d. all of the above. ANS: D

The first line of defense or first barrier to infection is unbroken skin and mucosal membrane surfaces. Normal biota deter penetration or facilitate elimination of foreign microorganisms from the body. Other types of first-line defenses against microbial invasion include secretions (e.g., mucus, ear wax, lactic acid in sweat, stomach acid, saliva, and tears). Additional protection is provided to the respiratory tract by the constant motion of the ciliated epithelial cells. PTS: 1

DIF:

Cognitive Level: 2

4. Another name for innate immunity is

immunity.

REF: p. 544


a. b. c. d.

natural inborn acquired both natural and inborn

ANS: D

Innate immunity also can be called natural or inborn. PTS: 1

DIF:

Cognitive Level: 1

REF: p. 544

5. The second line of body defense consists of: a. neutrophils, tissue basophils, macrophages. b. complement and lysozyme. c. interferon. d. all of the above. ANS: D

Natural immunity, a nonspecific mechanism, is the second line of defense. It consists of particular cells (neutrophils, tissue basophils, and macrophages) and soluble substances in the blood (complement, lysozyme, and interferon). Complement proteins are the major humoral (fluid) component of natural immunity. Lysozymes and interferon are a component of the second line of body defense as well. PTS: 1

DIF:

Cognitive Level: 2

REF: p. 544

6. The third line of defense is: a. acquired. b. adaptive. c. natural. d. both a and b. ANS: D

Acquired or adaptive immunity forms a third line of defense that allows the body to recognize, remember, and respond to a specific stimulus, an antigen. PTS: 1

DIF:

Cognitive Level: 2

REF: p. 544

7. Humoral-mediated immunity is associated with: a. T lymphocytes. b. B lymphocytes. c. neutrophils. d. monocytes. ANS: B

B lymphocytes are the cells associated with humoral immunity; T lymphocytes are associated with cell-mediated immunity. PTS: 1

DIF:

Cognitive Level: 2

8. Immediate hypersensitivity reactions are associated with: a. IgM. b. IgG.

REF: p. 547


c. IgE. d. IgD. ANS: C

Immediate hypersensitivity reactions are a subset of the body’s antibody-mediated mechanisms. Immediate hypersensitivity involves the reactions of immunoglobulin E (IgE) with tissue basophils (mast cells). PTS: 1

DIF:

Cognitive Level: 1

REF: p. 547

9. Arrange the stages in an antibody response in the correct sequence.

1 = Log phase 2 = Lag phase 3 = Plateau phase 4 = Decline a. 1, 2, 3, 4 b. 2, 3, 1, 4 c. 4, 2, 1, 3 d. 2, 1, 3, 4 ANS: D

The response to an antigenic or “foreign” substance is referred to as an immune response. An antibody response has four phases: lag, log, plateau, and decline. PTS: 1

DIF:

Cognitive Level: 1

10. A typical antibody response takes about a. 24 hours b. 72 hours c. 1 week d. 2 weeks

REF: p. 543

to develop sufficiently.

ANS: D

Immunity is not immediate. When a foreign antigen is first introduced, the antibody cannot be detected immediately in the serum or plasma. It is observed about 10 to 14 days after antigenic stimulation, and the antibody titer is greatest at about 20 days, after which it gradually decreases. PTS: 1

DIF:

Cognitive Level: 2

REF: p. 543

11. The antibody subclass associated with a primary response is: a. IgM. b. IgG. c. IgD. d. IgE. ANS: A

The subclass of antibody associated with the primary response is IgM. PTS: 1

DIF:

Cognitive Level: 2

REF: p. 543

12. The antibody subclass associated with a secondary response is:


a. b. c. d.

IgM. IgG. IgD. IgE.

ANS: B

A secondary antibody response produces a higher and longer lasting titer of IgG antibody. PTS: 1

DIF:

Cognitive Level: 2

REF: p. 543

13. Agglutination is comprehensively defined as: a. aggregation of soluble test antigens. b. aggregation of particulate test antigens. c. the physical attachment of antibody molecules to antigens. d. the establishment of cross-links between sensitized particles. ANS: B

Agglutination and precipitation are the visible expression of the aggregation of antigens and antibodies through the formation of a framework in which antigen particles or molecules alternate with antibody molecules. Agglutination (clumping) is the term used to describe the aggregation of particulate test antigens. Precipitation is the term applied to aggregation of soluble test antigens. The first phase of agglutination, sensitization, represents the physical attachment of antibody molecules to antigens. Lattice formation is the second phase of an agglutination reaction, in which establishment of cross-links between sensitized particles and antibodies occur. PTS: 1

DIF:

TCEogSnTitiBveALNeKveSl:E2LLER.COM

REF: p. 551

14. A popular method for waived, over-the-counter testing is: a. chemiluminescence. b. fluorescent in situ hybridization (FISH). c. signal amplification technology. d. enzyme immunoassays (EIA). ANS: D

EIA provides an alternative to immunofluorescent assays. This is a popular method for waived, over-the-counter testing. PTS: 1

DIF:

Cognitive Level: 2

15. A convalescent blood specimen should be collected a. 24 hours b. 72 hours c. 1 week d. 2 weeks ANS: D

REF: p. 552

after the onset of symptoms.


In obtaining specimens for serologic testing, it is important to consider the phase of the disease and the condition of the patient at the time of the specimen collection. This is especially important in assays for diagnosis of infectious diseases. If serum is being tested for antibody levels for a specific infectious organism, generally the blood should be drawn during the acute phase of the illness—when the disease is first discovered or suspected—and another sample drawn during the convalescent phase, usually about 2 weeks later. Accordingly, these samples are called acute and convalescent serum. PTS: 1

DIF:

Cognitive Level: 1

REF: pp. 554-555

16. A positive reaction when using the MonoSlide rapid slide test for infectious mononucleosis

indicates the presence of: a. Epstein-Barr virus. b. heterophil antibodies. c. Forssman antibodies. d. guinea pig antibodies. ANS: B

The MonoSlide test for rapid slide testing detects the presence of heterophil antibodies. This procedure is based on agglutination of horse erythrocytes by heterophil antibody present in infectious mononucleosis. PTS: 1

DIF:

Cognitive Level: 1

REF: p. 562

17. In testing for rheumatoid factor, lipemic or hemolyzed specimens can cause a. false-positive b. false-negative c. no effect on the test d. both false positive and false negative

results.

ANS: A

False-positive results may be observed in the assay for rheumatoid factor, if serum specimens are lipemic, hemolyzed, or heavily contaminated with bacteria. Biological false-positives can be manifested by disorders such as lupus erythematosus, Sjögren’s syndrome, syphilis, and hepatitis. False biological positive reactions have been reported with cardiolipin type of antigens in the following conditions: 1. Lupus erythematosus 2. Rheumatic fever 3. Vaccinia and viral pneumonia 4. Pneumococcal pneumonia 5. Infectious mononucleosis 6. Infectious hepatitis 7. Leprosy 8. Malaria 9. RA 10. Pregnancy 11. Aging individuals PTS: 1

DIF:

Cognitive Level: 2

REF: p. 567


18. In testing for rheumatoid factor, systemic lupus, syphilis, or hepatitis can cause a. false-positive b. false-negative c. no effect on the test d. either false positive or false negative

results.

ANS: A

False-positive results may be observed in the assay for rheumatoid factor, if serum specimens are lipemic, hemolyzed, or heavily contaminated with bacteria. Biological false-positives can be manifested by disorders such as lupus erythematosus, Sjögren’s syndrome, syphilis, and hepatitis. False biological positive reactions have been reported with cardiolipin type of antigens in the following conditions: 1. Lupus erythematosus 2. Rheumatic fever 3. Vaccinia and viral pneumonia 4. Pneumococcal pneumonia 5. Infectious mononucleosis 6. Infectious hepatitis 7. Leprosy 8. Malaria 9. RA 10. Pregnancy 11. Aging individuals PTS: 1

DIF:

Cognitive Level: 2

REF: p. 567

19. In the RPR test for syphilis, the presence of conditions such as infectious mononucleosis can

cause results. a. false-positive b. false-negative c. no effect on the test d. Either false-positive or false-negative ANS: A

False-positive results may be observed in the assay for rheumatoid factor, if serum specimens are lipemic, hemolyzed, or heavily contaminated with bacteria. Biological false-positives can be manifested by disorders such as lupus erythematosus, Sjögren’s syndrome, syphilis, and hepatitis. False biological positive reactions have been reported with cardiolipin type of antigens in the following conditions: 1. Lupus erythematosus 2. Rheumatic fever 3. Vaccinia and viral pneumonia 4. Pneumococcal pneumonia 5. Infectious mononucleosis 6. Infectious hepatitis 7. Leprosy 8. Malaria 9. RA


10. Pregnancy 11. Aging individuals PTS: 1

DIF:

Cognitive Level: 2

REF: p. 567

20. A type I hypersensitivity reaction in sensitive persons is associated with agents such as: a. peanuts. b. poison ivy. c. yellow hornet insect stings. d. both a and c. ANS: D

Type I hypersensitivity reactions can range from life-threatening anaphylactic shock to milder manifestations associated with food allergies. Anaphylaxis is the clinical response to immunologic formation and fixation between a specific antigen and a tissue-fixing antibody. This reaction is usually mediated by IgE antibody. Several groups of agents cause anaphylactic reactions. The two most common agents are drugs, such as systemic penicillin, and insect stings, such as common hornet, yellow hornet, and paper wasp. IgE-mediated adverse food reactions, such as peanuts, can be fatal. PTS: 1

DIF:

Cognitive Level: 2

REF: p. 547

21. A type IV hypersensitivity reaction manifestation can include: a. contact sensitivity. b. delayed hypersensitivity. c. rejection of foreign tissue grafts. d. all of the above. ANS: D

Type IV hypersensitivity reactions, or cell-mediated immunity, consists of immune activities that differ from antibody-mediated immunity. Cell-mediated immunity is moderated by the link between T lymphocytes and phagocytic cells. Lymphocytes (T cells) do not recognize the antigens of microorganisms or other living cells but are immunologically active through various types of direct cell-to-cell contact and by the production of soluble factors. Cell-mediated immunologic events include the following: • Contact sensitivity • Delayed hypersensitivity • Immunity to viral and fungal antigens • Immunity to intracellular organisms • Rejection of foreign tissue grafts • Elimination of tumor cells bearing neoantigens PTS: 1

DIF:

Cognitive Level: 2

REF: p. 548

TRUE/FALSE 1. Factors that influence an individual’s defense against disease include age. ANS: T


Many factors (e.g., general health and the age of an individual) are important considerations in the defense against disease. The ability to respond immunologically to disease is age related. Other factors that can influence body defenses are genetic predisposition to many disorders, nutritional status, and an individual’s method of coping with stress. PTS: 1

DIF:

Cognitive Level: 1

REF: p. 541

2. Factors that influence an individual’s defense against disease include status of general health. ANS: T

Many factors (e.g., general health and the age of an individual) are important considerations in the defense against disease. The ability to respond immunologically to disease is age related. Other factors that can influence body defenses are genetic predisposition to many disorders, nutritional status, and an individual’s method of coping with stress. PTS: 1

DIF:

Cognitive Level: 1

REF: p. 541

3. Factors that influence an individual’s defense against disease include nutritional status. ANS: T

Many factors (e.g., general health and the age of an individual) are important considerations in the defense against disease. The ability to respond immunologically to disease is age related. Other factors that can influence body defenses are genetic predisposition to many disorders, nutritional status, and an individual’s method of coping with stress. PTS: 1

DIF:

Cognitive Level: 1

REF: p. 541

4. Factors that influence an inTdE ivSidTuB alA ’sNdK efSeE nsLe L agEaR in.stCdOisMease include taking mega-doses of

vitamins. ANS: F

Many factors (e.g., general health and the age of an individual) are important considerations in the defense against disease. The ability to respond immunologically to disease is age related. Other factors that can influence body defenses are genetic predisposition to many disorders, nutritional status, and an individual’s method of coping with stress. PTS: 1

DIF:

Cognitive Level: 1

REF: p. 541

5. An anamnestic response differs from a primary antibody response in several important

aspects. A secondary response has a longer lag phase, a longer plateau phase, and a more gradual decline in antibody titer. ANS: F

An anamnestic response differs from a primary antibody response in several important aspects as follows: • A secondary response has a shorter lag phase, a longer plateau phase, and a more gradual decline in antibody titer. • IgM-type antibodies are the principal class of antibody formed in a primary response. • In a secondary response, antibody concentrations reach a higher titer.


The plateau levels in a secondary response are typically 10-fold or more than the plateau levels in the primary response.

PTS: 1

DIF:

Cognitive Level: 2

REF: p. 543

6. An anamnestic response differs from a primary antibody response in several important

aspects. IgG-type antibodies are the principal class of antibody formed in a primary response. ANS: F

An anamnestic response differs from a primary antibody response in several important aspects as follows: • A secondary response has a shorter lag phase, a longer plateau phase, and a more gradual decline in antibody titer. • IgM-type antibodies are the principal class of antibody formed in a primary response. • In a secondary response, antibody concentrations reach a higher titer. • The plateau levels in a secondary response are typically 10-fold or more than the plateau levels in the primary response. PTS: 1

DIF:

Cognitive Level: 2

REF: p. 543

7. An anamnestic response differs from a primary antibody response in several important

aspects. In a secondary response, antibody concentrations reach a higher titer. ANS: T

An anamnestic response differs from a primary antibody response in several important aspects as follows: • A secondary response hTaEs S aT shBoA rtN erKlaSgEpLhL asE e,Ra.loCnOgM er plateau phase, and a more gradual decline in antibody titer. • IgM-type antibodies are the principal class of antibody formed in a primary response. • In a secondary response, antibody concentrations reach a higher titer. • The plateau levels in a secondary response are typically 10-fold or more than the plateau levels in the primary response. PTS: 1

DIF:

Cognitive Level: 2

REF: p. 543

8. An anamnestic response differs from a primary antibody response in several important

aspects. The plateau levels in a secondary response are typically 10-fold or more than the plateau levels in the primary response. ANS: T

An anamnestic response differs from a primary antibody response in several important aspects as follows: • A secondary response has a shorter lag phase, a longer plateau phase, and a more gradual decline in antibody titer. • IgM-type antibodies are the principal class of antibody formed in a primary response. • In a secondary response, antibody concentrations reach a higher titer. • The plateau levels in a secondary response are typically 10-fold or more than the plateau levels in the primary response. PTS: 1

DIF:

Cognitive Level: 2

REF: p. 544


9. The antinuclear antibody (ANA) assay is specific for systemic lupus erythematosus (SLE). ANS: F

ANAs are immunoglobulins that react with the whole cell nucleus or nuclear components such as nuclear proteins, DNA, or histones in the tissue of the host. ANAs are found in a variety of disorders, including SLE and rheumatoid arthritis (RA), certain drugs, and in aging persons without disease. Thus the assays for ANAs are not specific for SLE. Rheumatoid factor belongs to a larger family of antiglobulins usually defined as antibodies with specificity for antigen determinants on the Fc fragment of human or certain animal IgG. Rheumatoid factors have been associated with three major immunoglobulin classes: IgM, IgG, and IgA. Rheumatoid factors are autoantibodies that can be present in many persons with RA, but not all. PTS: 1

DIF:

Cognitive Level: 2

REF: p. 567

10. Rheumatoid factor has been associated with IgM, IgG, and IgA immunoglobulin classes. ANS: T

ANAs are immunoglobulins that react with the whole cell nucleus or nuclear components such as nuclear proteins, DNA, or histones in the tissue of the host. ANAs are found in a variety of disorders, including SLE and rheumatoid arthritis (RA), certain drugs, and in aging persons without disease. Thus the assays for ANAs are not specific for SLE. Rheumatoid factor belongs to a larger family of antiglobulins usually defined as antibodies with specificity for antigen determinants on the Fc fragment of human or certain animal IgG. Rheumatoid factors have been associated with three major immunoglobulin classes: IgM, IgG, and IgA. Rheumatoid factoTrsEaSreTaBuAtoNaK ntS ibE oL diL esEtR ha.t C caOnMbe present in many persons with RA, but not all. PTS: 1

DIF:

Cognitive Level: 2

REF: p. 566

11. Rheumatoid factors are autoantibodies. ANS: T

ANAs are immunoglobulins that react with the whole cell nucleus or nuclear components such as nuclear proteins, DNA, or histones in the tissue of the host. ANAs are found in a variety of disorders, including SLE and rheumatoid arthritis (RA), certain drugs, and in aging persons without disease. Thus the assays for ANAs are not specific for SLE. Rheumatoid factor belongs to a larger family of antiglobulins usually defined as antibodies with specificity for antigen determinants on the Fc fragment of human or certain animal IgG. Rheumatoid factors have been associated with three major immunoglobulin classes: IgM, IgG, and IgA. Rheumatoid factors are autoantibodies that can be present in many persons with RA, but not all. PTS: 1

DIF:

Cognitive Level: 2

MATCHING

Match the term with the appropriate definition. a. vaccination

REF: p. 566


b. infusion of plasma containing antibodies c. obtained from colostrum d. as a result of suffering from the disease 1. 2. 3. 4.

active natural immunity active artificial immunity passive natural immunity passive artificial immunity

1. ANS: D PTS: REF: p. 544 | p. 547 2. ANS: A PTS: REF: p. 544 | p. 547 3. ANS: C PTS: REF: p. 544 | p. 547 4. ANS: B PTS: REF: p. 544 | p. 547

1

DIF:

Cognitive Level: 2

1

DIF:

Cognitive Level: 2

1

DIF:

Cognitive Level: 2

1

DIF:

Cognitive Level: 2

Match the leukocyte type with a specific function (use an answer only once). a. recognition of foreign antigen and antibody production b. antigen-presentation c. phagocytosis 5. granulocytes 6. monocytes-macrophages 7. lymphocytes-plasma cells 5. ANS: C REF: pp. 549-550 6. ANS: B REF: pp. 549-550 7. ANS: A REF: pp. 549-550

PTS: T1ESTBANKS DE IFL: LE CoRg. niC tivOeMLevel: 2 PTS: 1

DIF:

Cognitive Level: 2

PTS: 1

DIF:

Cognitive Level: 2

Match the antibody response term with the correct description. a. no detectable antibody b. antibody titer stabilizes c. antibody is catabolized d. antibody titer increases 8. 9. 10. 11.

lag decline log plateau

8. ANS: A REF: p. 543 9. ANS: C REF: p. 543 10. ANS: D REF: p. 543

PTS: 1

DIF:

Cognitive Level: 2

PTS: 1

DIF:

Cognitive Level: 2

PTS: 1

DIF:

Cognitive Level: 2


11. ANS: B REF: p. 543

PTS: 1

DIF:

Cognitive Level: 2

Match each statement with correct item below. a. A conjugated antibody is used to detect antigen-antibody reactions. b. Antibodies can act as antigens and react with antiimmunoglobulins. 12. Direct immunofluorescent assay (DFA) 13. Indirect immunofluorescent assay (IFA) 12. ANS: A REF: p. 552 13. ANS: B REF: p. 552

PTS: 1

DIF: Cognitive Level: 2

PTS: 1

DIF: Cognitive Level: 2


Chapter 17: Immunohematology and Transfusion Medicine Turgeon: Linné & Ringsrud’s Clinical Laboratory Science, 7th Edition MULTIPLE CHOICE 1. The major categories for transfusion of red blood cells (RBCs) and blood components include

all of the following except transfusion: a. may be used to restore or maintain oxygen-carrying capacity or hemoglobin. b. can be used to restore or maintain blood volume. c. can replace coagulation factors to maintain hemostasis. d. can be to change a patient’s blood type. ANS: D

There are many indications for the transfusion of RBCs and blood components. In general, these can be divided into four major categories, and the component to be transfused will depend on which category applies: 1. Transfusion may be used to restore or maintain oxygen-carrying capacity or hemoglobin. This is best done by the transfusion of RBCs with plasma removed. The transfusion of whole blood is both unnecessary and contraindicated, because the inclusion of plasma will increase blood volume with possible circulatory overload. 2. Transfusion can be used to restore or maintain blood volume. This is necessary in cases of acute blood loss, as seen with massive bleeding, to prevent shock. Although whole blood may sometimes be indicated, as in actively bleeding patients who have lost more than 25% to 30% of their blood volume, it is usually preferable to use RBCs and a volume expander (e.g., crystalloid [electrolyte] solutions such as 0.9% sodium chloride [isotonic saline]) or plasma substitutes. AddTiE tioSnTalBnAeN edKfS orEhLeL mE ogRl. obCinOcMan be replaced later with packed RBCs, although in most patients, about 20% of blood volume can be replaced with crystalloid solutions alone. 3. Transfusion can replace coagulation factors to maintain hemostasis. This is done with a variety of blood components, which vary with the particular situation. Components include platelet concentrates and fresh frozen plasma. 4. Transfusion may be indicated to restore or maintain leukocyte function. Although rare, this may be necessary for severely granulocytopenic patients with infections that do not respond to antibiotics. PTS: 1

DIF:

Cognitive Level: 2

REF: p. 577

2. The latest standards for blood donor selection can be found at all of the following sources

except: a. www.arc.com. b. AABB Standards for Blood Banks and Transfusion. c. Federal Register. d. Code of Federal Regulations. ANS: C

Medical guidelines and requirements for donor selection have been developed by the FDA, the AABB, and the American Red Cross (ARC). It is essential that guidelines and requirements be established for each transfusion service and be codified in its own standard operating procedures manual.


PTS: 1

DIF:

Cognitive Level: 2

3. Eligibility for autologous donation is a. more strict than for b. less strict than for c. the same as d. variable depending on the donor

REF: p. 581

regular volunteer donations.

ANS: B

Making donations for your own use during surgery (autologous blood donation) is considered a medical procedure that requires a written prescription; the rules for eligibility are less strict than for regular volunteer donations. PTS: 1

DIF:

Cognitive Level: 2

REF: p. 584

4. The anticoagulant of choice for a unit of donor blood is: a. dextrose. b. citrate. c. inorganic phosphate buffer. d. heparin. ANS: B

Citrate is used as an anticoagulant, which binds calcium, thus preventing activation of the coagulation cascade. PTS: 1

DIF:

Cognitive Level: 2

REF: p. 581

5. The presence of A, B, and H antigenic material in body secretions is regulated by: a. precursor of A and B RBC antigens. b. H gene. c. Se gene. d. A and B genes. ANS: C

The Se and se genes regulate the presence of A, B, and H antigenic material in the body secretions. About 78% of the population has inherited the Se gene (SeSe or Sese). These persons are secretors whose secretory cells produce H, A, or B substance. Thus, corresponding H, A, or B substance will be found in the saliva of these persons. PTS: 1

DIF:

Cognitive Level: 2

REF: p. 595

6. The optimal temperature of reactivity for anti-A and anti-B is: a. 0° C. b. 4° C. c. 37° C. d. any temperature. ANS: B

Natural ABO antibodies react best if the red cells are suspended in saline solution and the test is carried out at room temperature or 4° C. Immune antibodies differ in that they react better if cells are suspended in albumin or serum and incubated at 37° C.


PTS: 1

DIF:

Cognitive Level: 2

7. Anti-A and anti-B belong to the a. IgM b. IgG c. IgA d. IgD

REF: p. 595

subclass of antibodies.

ANS: A

Natural or environmentally stimulated antibodies, anti-A and anti-B are IgM antibodies. PTS: 1

DIF:

Cognitive Level: 1

REF: p. 595

8. An Rh-negative person has the genotype of: a. cde/cde. b. cDe/cDe. c. Cde/cde. d. CDE/CDE. ANS: A

Persons whose RBCs contain D antigen either as D/D or D/d are called Rh positive. They represent approximately 85% of the population. Persons whose RBCs lack the D (Rho) antigen are termed Rh negative. Rh-negative persons are then d/d. They represent about 15% of the population. The great majority of Rh-negative persons are cde/cde. This genotype is what is meant by a truly Rh-negative person. PTS: 1

DIF:

TCEogSnTitiBveALNeKveSl:E2LLER.COM

9. Rh antibodies belong to the a. IgM b. IgG c. IgD d. IgE

REF: p. 596

subclass of antibodies.

ANS: B

Rh antibodies below to the IgG subclass of antibodies. They are immune antibodies produced by exposure to the Rh antigen on RBCs. PTS: 1

DIF:

Cognitive Level: 2

REF: p. 597

10. All of the following are criteria for postpartum eligibility for Rh immune globulin

administration except: a. administered to the mother within 72 hours of delivery. b. administered to the baby within 72 hours of delivery. c. baby must be RhO (D) positive. d. mother must not demonstrate anti-D in her blood. ANS: B

Rh immune globulin (RhIG) can prevent immunization to the D antigen during and immediately following pregnancy.


RhIG is injected intramuscularly within 72 hours of delivery in mothers (1) who are D-negative, (2) who have no detectable anti-D antibody, and (3) whose newborn infant is D-positive. PTS: 1

DIF:

Cognitive Level: 2

REF: pp. 607-608

11. Transfusion reactions that are the most life threatening are: a. hemolytic. b. febrile. c. nonimmune. d. iron-overload. ANS: A

The most life-threatening transfusion reaction is the hemolytic reaction that occurs with the destruction of incompatible RBCs by antibodies in the patient’s serum (usually ABO incompatibility). PTS: 1

DIF:

Cognitive Level: 2

REF: p. 604

TRUE/FALSE 1. The first stage of red cell agglutination is sensitization. ANS: T

Many factors affect red cell agglutination, which is thought to occur in two stages, sensitization and agglutination. The first stage involves tT heEpShTyB siA caN lK atS taE chLmLeE ntRo.f C anOtiMbody to red cells and is referred to as sensitization. The second stage of agglutination involves the formation of bridges between sensitized red cells to form the lattice that is seen as agglutination. Correct conditions are essential for reliable tests. Many factors affect the reactions that are used to detect an antigen-antibody reaction. These include: • Use of adequate serum and red cells • Correct concentration of cell suspensions • The testing medium • Proper temperature and duration of incubation • Proper use of centrifugation • The condition and correct use of reagents • Accurate reading and interpretation of agglutination reactions PTS: 1

DIF:

Cognitive Level: 2

REF: p. 589

2. The proper temperature and duration of incubation are not critical to antigen-antibody

reactions. ANS: F

Many factors affect red cell agglutination, which is thought to occur in two stages, sensitization and agglutination.


The first stage involves the physical attachment of antibody to red cells and is referred to as sensitization. The second stage of agglutination involves the formation of bridges between sensitized red cells to form the lattice that is seen as agglutination. Correct conditions are essential for reliable tests. Many factors affect the reactions that are used to detect an antigen-antibody reaction. These include: • Use of adequate serum and red cells • Correct concentration of cell suspensions • The testing medium • Proper temperature and duration of incubation • Proper use of centrifugation • The condition and correct use of reagents • Accurate reading and interpretation of agglutination reactions PTS: 1

DIF:

Cognitive Level: 2

REF: p. 589

3. The concentration of RBCs used in blood banking testing is important. ANS: T

Many factors affect red cell agglutination, which is thought to occur in two stages, sensitization and agglutination. The first stage involves the physical attachment of antibody to red cells and is referred to as sensitization. The second stage of agglutination involves the formation of bridges between sensitized red cells to form the lattice that is seen as agglutination. Correct conditions are essential for reliable tests. Many factors affect the reactions that are used to detect an antigen-antibody reaction. These include: • Use of adequate serumTaE ndSrTeB dA ceNllK s SELLER.COM • Correct concentration of cell suspensions • The testing medium • Proper temperature and duration of incubation • Proper use of centrifugation • The condition and correct use of reagents • Accurate reading and interpretation of agglutination reactions PTS: 1

DIF:

Cognitive Level: 2

REF: p. 589

MATCHING

Match the term with the appropriate description. a. used to detect antibody b. used to detect antibody-coated RBCs 1. direct antihuman globulin test 2. indirect antihuman globulin test 1. ANS: B REF: p. 600 2. ANS: A REF: p. 600

PTS: 1

DIF: Cognitive Level: 2

PTS: 1

DIF: Cognitive Level: 2


Match the blood product with the most appropriate use (use an answer only once). a. Best used when oxygen-carrying capacity is needed b. Rarely used but can meet a patient’s need for volume replacement, oxygen-carrying capacity, and heat-labile coagulation factors c. Used to treat multiple coagulation deficiencies d. Used to treat hemophilia 3. 4. 5. 6.

Whole blood Packed RBCs Fresh frozen plasma Monoclonal factor VIII

3. ANS: B PTS: REF: pp. 577-578 | p. 580 4. ANS: A PTS: REF: pp. 577-578 | p. 580 5. ANS: C PTS: REF: pp. 577-578 | p. 580 6. ANS: D PTS: REF: pp. 577-578 | p. 580

1

DIF:

Cognitive Level: 2

1

DIF:

Cognitive Level: 2

1

DIF:

Cognitive Level: 2

1

DIF:

Cognitive Level: 2

ESSAY 1. Fill in the blanks using the following key for antigens on the RBCs or antibodies in the

serum/plasma. A. A B. Anti-A C. B D. Anti-B E. Anti-A, anti-B F. None Blood Group A O B

Antigen 1. 3. 5.

Antibody 2. 4. 6.

Blood Group A O B

Antigen 1. A 3. F 5. C

Antibody 2. D 4. E 6. B

PTS: 1

DIF:

ANS:

Cognitive Level: 2

REF: pp. 590-591


Turn static files into dynamic content formats.

Create a flipbook
Issuu converts static files into: digital portfolios, online yearbooks, online catalogs, digital photo albums and more. Sign up and create your flipbook.